Vous êtes sur la page 1sur 218

1.

If tan A =3/4 then sinA cosA


Ans- sina=3/5 cosa =4/5
Tan A=3/4. So it is sinA/cosA=3/4 so sinA=3 and cosA=4 so ans is 12. Where ois
the fault
12/25
2.

(i)A ladder 15 metres long just reaches the top of a vertical wall. If the ladder
makes
an angle of 60 with the wall, find the height of the wall
(ii) An observer 1.5 metres tall is 20.5 metres away from a tower 22 metres high.
Determine the angle of elevation of the top of the tower from the eye of the
observer.
Ans- yes 7.5 and 45 right ans

3. If tan + sec = L , then Sec in terms of L..????


Ans- sec^2 - tan^2 = 1 (sec+tan)(sec-tan) = 1
Add the two eq to get 2sec = L+1/L
l^2+1/2l...right ans
4. In a right angled triangel the hypotenuse is four times as long as the perpendicular
drawn to it from the opposite vertex .One of the acute angle is???
Ans - Let the triangle be ACB, CP be the perpendicular to AB AC = c, CB=b,
CP=h,PB = x and AP = a-x c2 = h2 + (a-x)2
b2 = h2 + x2
on adding, we have 2h2 + x2 + (a-x)2 = a2 ..(1)
also a = 4h and cos B = b/a = x/b
so x = b2/a
on solving (1), we have h2 + x2 -4hx = 0
substituting above gives
b4 + a4/16 -a2.b2 =0 .. (2)
as cos B = b/a
on solving eqn. (2) we have , Cos2B = 0.933 or 0.067
Cos B = 0.966 or 0.259
B = 15 or 75 degrees

5. (sin^4 cos^4 +1) cosec^2 = ??


1
-1
0
2
Ans- 2 is right
6. sin^6 + cos^6 + 3sin^2 cos^2 = ??
1
-1
0
none of these
Ans- 1. (sin^2@ + cos^2@)^3 = given expression = 1

7. 2sin@ + 15cos^2@ = 7 then cot @ =?


5/4
3/4
1/4

Ans- uadritic hai bhai solve kar lo


change cos^@ into sin^2
eq vl b 15sin^2 - 12sin -8 = 0
2sin@ + 15(1-sin^2@) = 7 ........now its quad eq

yesterday glady da told me one thing...it can b done easily by option...cot@=3/4,


then sin@=4/5 n cos@=3/5....so again desi method by checking option
8. Sin x + sin. ^2 x=1
Find value of
Cos^12 x + 3cos^10 x + 3cos^8 x + cos^6 x??
1
2
3
0

Ans- (cos^2@ + cos^4@)^3 ...........from given we can deduct sin@ = cos^2@ ...........so
ans is 1

9. tan@ + sin@ = m and tan@ -sin@ = n then the value of m^2 - n^2 is a) 2rtmn b) 4mn
c) 4rtmn d) 2rt(m/n)
Ans- 4rtmn
bhai put values of m and n ..... 4rt(tan^2-sin^2) vl give 4sin@tan@
10. Given that sin + 2cos = 1, then 2sin cos=??
Ans- @=30
15/4 haan yaar 15/4
11. if cot @ = 8/15, then wat is the value of rt(1-cos@/1+cos@)???
Ans - cos = 8/hypotenus will come. cot me hypotenus nahi rehta
cot=adj side/opp side, so here 8/15, and u get hyp=rt(8^2+15^2),
then u ll get cos=8/hyp now apply this in equ. u ll get answer
b = 8 p = 15 h = 17
Ans will be 3/5....:)
welcum shashi bhai
12. SinA= -3/5 and pie<A<3pie/2
Find cos(A/2)
1/rt10
-1/rt10
-3/rt10
3/rt10
Ans:- -1/rt10
cosA = 2cos^2(a/2) - 1....we have sinA. so cosA= -4/5. substituting cos(a/2)= 1/rt10
its - 1/rt10 as A will lie in third quadrant
13. 4 tan = 3,then
(4sinq -cosq)/(4sin+ cosq)
Ans :- 1/2
14. if cos 43=x/underroot x^+y^,
then tan47 is
Ans- cos 43 = x / under root x^2+y^2 ..........then for 43* base = x , perpedicular=y hyp =
under root x^2 + Y^2 .............now for for 47* it would be opposite mean base and
perpendi interchange
cos43 equal to sin47 equal to l/h
n tan47 equal to l/b so by phaythagoras theorem we get b equal to y
so x/y

15. if a,b,c r non zero real no. Such dat a+b+c=0 and b^ ca,then d value of a^+b^+c^/b^ca is
Ans- again a typo error if it is b^2 = ca then it would be infinity
ans options r 3,2,0,1
16. If ABC is right angled at C, then the value of cos (A+B) is
Ans- 0
agar C = 90* then A+B to 90* hoga hi na
17. The angle of elevation of the top of a tower 30 m high, from two points on the level
ground on its opposite sides are 45 degrees and 60 degrees. What is the distance
between the two points? (1) 30
(2) 51.96
(3) 47.32
(4) 81.96
Ans- tan45=30/x
X=30
Tan60 = 30/y
Y=10root3
Reqd ans =30+10root3
=30+17.3
=47.3..:)
18. cos^4x/cos^2y + sin^4x/sin^2y = 1 then what is the value of
cos^4y/cos^2x + sin^4y/sin^2x
a.4
b.0
c.1/8
d.1
plz expain also
Ans - 1 put x=y=45
19. The sides of a triangle are a,b, rt( a^2 + b^2 + ab), the greatest angle is?
Ans - angle opposite to rt(a^2 + b^2 +ab ) will be greatest .............i think it may denote
as C

20. If cosA-sinA=rt2*sinA,then cosA+sinA=?


Options:
rt2*sinA
1/2
rt2*cosA
1
Ans- rt2 cosA.
(cosA-sinA)^2 + (cosA+sinA)^2 = (root2*sinA)^2 + x^2
Find d value of x, thats ans.
21. the minimum value of cos 2A + cos A is ?
ans- cos2A + cosA = (2cos^2A - 1) + cosA
= 2(cosA + 1/4)^2 - 9/8
so minimum value=-9/8
2 cos ^2 A -1 + cos A = 2 ( cos ^2 A +1/2* cos A ) - 1..
= 2 ( cos ^2 A + 1/2* cos A + 1/16) - 1 - 2*1/16 ( add and substract 1/16)
= 2 ( cos A + 1/4) ^2 - 9/8...
so - 9/8 is the minimum value..
Thanks for a detailed solution. Not of 10th stand, but of ssc stand
22. The angle of elevation of the sun, when the length of the shadow of a tree 3 times
the height of the tree, is:
Ans- iss bar ssc ne hadh hii kar di 10th class mein jo 10-12 min leke bade aaram se
karte thai speed nikalne ke height and distance mein woh dal diye :)
shadow means base =3
Perpendicular = 1
Tan = 1/3 bhai to angle 60 kaise aaya tan^-1(1/3) aana chahiye na..:p
wrong ques i guess
30 degre
off course 30 degre

23. if m ( tan A-30) = n tan ( A + 120)....then, (m+n)/ (m-n) =


a. 2 cos 2 A, cos 2 A, 2 sin 2A,none of these..
ans- m/n = tan(A+120)/tan(A-30)
By C and D
m+n/m-n = tan(A+120)+tan(A-30) /tan(A+120) - tan (A-30 )
Lamba ja raha hai...:p
Ans- convert tan in to sin / cos .....then apply sin ( A+b ) formulae..
convert tan in to sin / cos .....then apply sin ( A+b ) formulae..
is it 2cos2A..i did the same steps upto shashi and then put A=60 in equations and then
in options..only option a satisfies it...
24. if y = 2 sin A / ( 1+cos A +sin A), then ( 1-cos A + sin A ) / ( 1+ sin A ) =? in terms of
Y?
Ans- multiplying by conjugate
ans is y
25.If sin^2 A = cos^3 A,then find the value
of ( cot^6 A - cot^ 2 A ) is?
1
0
-1
2
Ans- 1...simply convert cot to cos/sin and put cos^6 = sin ^4...then substract
didnt get sin^4@/sin^6@-cos^2@/cos^3@ then?
1/ sin ^2 A - cos ^2 A/ sin ^2 A = ( 1- cos ^2 A)/ sin ^2 A = sin ^2 A/ sn ^2 A
26.If A=sin^4Q+cos^4Q then ( Q-thetha)
1. 0<A<1/2
2.1<A<=3/2
3.1/2<=A<=1
4.3/2<=A<=2
5.None
Ans- A=sin^4Q+cos^4Q = (sin ^2 Q + cos ^2 Q) ^2 - 2 sin ^2 Q cos ^2 Q
= 1- 2 sin ^2 Q cos ^2 Q = 1 - sin ^2 2 Q /2...has max value when sin 2 Q = 1 and has
min value when sin 2 Q = 0....so
A lies between 1- 1/2 and 1-0....b/w 1/2 and 1
A = (sin^2Q + cos^2Q)^2 - 2 sin^2Qcos^2Q....the second term has min value = 0 so the
max value will be 1...option 3
Thanks bro..

27. The chord AB of a circle of centre O subtends an angle theeta with the tangent
at A to the circle. ang ABO is..?
a) theeta
b) 90- theeta
c) 90+ theeta
d) 2(180- theeta)
Q2--- The sum of angles in thr four
segments exterior to a cyclic quadrilateral
is equal to ( in degres)..?
a) 450
b) 360
c) 540
d) 720
Q3--- Triangle ABC is an isosceles triangle
in which AB = AC. A circle through B
touches the side AC at D and intersects
the side AB at P. If D is the midpoint of the
side AC. Then AP is..?
a) AB/4
b) AD^2
c) AP = AD
answers....1 . 90 theta 2. 540 3. AB/4
28.find the min. value of
4Cot^2 A+9tan^2 A
my ans is -12
sandes, why not -12?
As we can also write d expression as ( A+ B )^2..
here 4Cot^2 A+9tan^2 A = ( 2 cot A +3
tan A )^2 12
to get -12, u need to equate 2 cot A + 3 tan A =0...so 2 cot A = -3 tan A ..or cot ^2 A = -3,
which will become complex...so u can not, i think , equate 2 cot A + 3 tan A to zero..
use AM >= GM...one line u can get min value as 12
for any two positinv nos..... arithmetic mean >= geometric mean....let the nos. be a and b
then (a+b)/2 >= under root of a*b...here a = 4cot^2A and b = 9tan^2A now use this

29.The area of the traingle formed by the lines 5x+7y=35, 4x+3y=12 and a-axis is
a) 160/13sq units
b)150/13 sq units
c)140/13sq unit
d) 10 sq unit
Ans- get the intersection point of the two lines by solving them. The Y cordinate would
give the ht of the triangle and at y-o for each line would give x get the base. Then area of
triangle= 1/2 b*h
how do u solve this type of sums? subst. values to all 3 eqns, form triangle or any fig and
then apply mensuration formulas?
first put y=0 in both eqns for 1st it wud give 7 for other x=3 these pts give intersection pt
on x axis and make base the distance is 7-3=4. now solving bothe eqns by subst. or
elimn get y=80/13 . this is the perpendicular ht of third pt. Now applying formula = 1/2
base8 ht. gives 160/13
30. First ques here
find the value of 4sin50-rt3tan50
Ans- dont think ssc will go to this extant ......we hv to leave this question in exam if it
comes
Ssc ka koi bharosa nhi, kaise v ques aa sakte h
may be 1////right?
31. The area of the triangle formed by the three graphs of the equations x=4, y=3 and
3x+4y=12, is?
Ans- 6 shud b the ans
the equation 3x+4y itself makes x and y intercept as 4 and 3...so answer ...6*4/2=12
how 12 bro?
sorry 3*4/2 =6....
A=b*h/2=4*3/2=6
32. The radius of the circumcircle of the triangle made by x axis, y axis and 4x+3y=12
is?
Ans- abc/4 A......3*4*5/4*6
or simply for a right angled triangle its half the hypotneus......so 5/2 = 2.5....
Yes. abc/(4*Area)
33. 4sin50-rt3tan50
=2sin100-rt3sin50
=2cos10-rt3sin50
=2cos(60-50)-rt3sin50
open the bracket of cos n ans is 1
=4sin50cos50-rt3sin50
very gud explanation....i liked it very
A good Question..
there is no pt. in lyking or disliking a ques..ssc wont ask us while setting paper
I dont understand it and its beyond my ambit.

34. The number of solutions of the equation sin 5 A cos 3 A = sin 6 A cos 2 A in the
interval [ 0, pi]
Ans- yaar yeh sab engg k questions hai nahi poochenge....do lines ka solution hai y =
first line and y = second line intersection ka

35. The greatest number of 5 digits to be added to 8321, so that the sum will be exactly
divisible by 15, 20, 24, 27, 32, and 36 is?
Ans- 99679
CGL qustn..
36. sin 2 A + sin 2 B = 1/2, cos 2 A + cos 2 B = 3/2, then cos ^2 ( A -B ) = ?
Ans- simple apply the formula of sin ( c+d) and cos ( c+d)...square and add...will get the
answer as 5/8....
is there any direct formula for cos ^2 ( A -B ) ?
CosA+B*cosA-B =
divide the two eq u vl get tan A+B = 1/3
obtain cos(A+B) from this which vl be 3/rt10
now put this value in above eq u vl get cos A-B = rt10/4
Suare to get the answer
37. if 2cos^2@ + 3sin@ = 0 then sin@= ?
4
2
3
none of the above
Ans-value of sin@ always lies between -1 and 1
so none of the above
ya its none of the above but i wanted to trick you guys by putting this kind of option
.....................ans is -1/2
38. Find the value of theeta for maximum value of
sin 3 theeta + cos 3 theeta..?
Ans- 15
3q=45 q15

39. If A+B+C=180degree then what is the value of cotA.cotB+cotB.cotC+cotC.cotA


Ans-A= B = C = 60
Ans will be 1....
tanA+tanB+tanC=tanA.tanB.tanC
Cot(a+b+c) ka
40. 5cos@ + 12sin@ = 13 then tan@ =
13/12
12/13
12/5
5/12
Ans- 5cos@+12sin@=13
5+12tan@=13sec@ (divide both side by cos@)
25+144tan^2@+120tan@=169(1+tan^2@) [squaring both]
25tan^2@ - 120tan@ + 144 = 0
(5tan@ - 12)^2=0
tan@=12/5
cos^2 A + sin ^2 A =1...for that 5/13 = cos A and 12/13 = sin A ( same hyp)....
so tan A = 12/5
41. Find the value of (cos18-sin18)/ (cos18+sin18)
Ans-45-18=27 take tan both d sides n use d formula 4 tan(a+b)
Maveric sir's changla(best) way.... Sin45cos18cos45sin18/cos45cos18+sin45sin18=sin(45-18)/cos(45-18)=sin27/cos27=tan27
In this type que s divide all by cos...here cos 18...
which will give, ( 1 - tan 18) / ( 1+ tan 18) = tan ( 45-18) = tan 27 = cot 63
42. If cos(x-y)= -1, find the min +ve value of sinx+siny
Ans- 0
x=180 y=0
x-y is 180
x is 360
y is 180
therefore sin360 plus sin180 is zero
ek bar saare trigo functions ke graph dekh lo kaafi help milti hai

43. One more of same type find d value of (cos17+sin17)/ (cos17-sin17)


Ans-hey divide by cos 17, u will get ( 1+ tan 17) / ( 1- tan 17) = tan ( 45+17) = tan 62...=
cot 28
44. If tan A = ( root 2 - 1), then, sin A cos A is..?
Draw a right angled triangle
opposite side rt2 -1
adj side 1
find the hyp and solve it.
1/2rt2 is the right ans
45. Find d max value of 5cos@ +cos(@+60)+3
Ans- At first open the bracket 4 cos,put the value of cos60 n u'l get 13/2*cos@3rt3/2*sin@ +3
=sqrt of sqre of coef of cos n sin+3
=7+3
=10
46. if MN is 10 meters in length and is tangent to one of the concentric circles at point A.
If it is given that the raddi of the both the circles are integers, then what is the radius of
the inner circle.
Ans- bhai circle draw karo... with chord MN as tangent to the inner circle... use
pytahogorean triplet
Ans- 12
47. tan75-tan30-tan75tan30 is..?
Ans-use tan(A-B) formula where A = 75 and B =30
48. (sin x +sin 3x +sin5x +sin7x) / (cos x +cos 3x +cos5x + cos7x) = ?
Ans- put x zero
1) tan x
2) tan 2x
3) tan 4x
4) tan 8x
tan4x

49. cos 55 + cos 65 + cos 175 is..?


Ans- 0
50. Find tan 20 +2tan50..?
Ans- tan(50+20) formula use karlo tan(A+B) wala u vl get the answer
Yes n d term tan70tan20= cot20tan20=1 ye concept v use hoga
Yes manish wo type err tha nw edited
tan 70 * tan 20 = 1 ??
I think short cut for this sum isDo addition & ans is tan70
51. Ram bought 6 mangoes, 8 oranges and 10 bananas for a certain amount of money.
Ramesh bought 15 mangoes, 20 oranges and 15 bananas for the twice the amount that
hari spent. What percentage of money spent by hari for buying bananas ?
Ans- 6m+8o+10b =x .......multiply 2.5 both side it will be 15m+20o+25b =2.5x where as it
is given 15m+20o+15b = 2x................now 10b= 0.5x so hari spent 50%

bhai i think ram ke place pe hari ayega


typo error
hehe.. :-P
CID !!
daya darwazaa tod do :)

52. x^1/3+y^1/3=z^1/3
than x+y+z+27xyz
-1
0
1
27
Ans- 0
how?
if a + b + c = 0,,, then a^3+b^3+c^3 = 3abc...
But I think the problem will be -z^(1/3)
Then we can go by cubing both sides......
cubing both sides but that process is slightly time taking
Yes thats ok. But z + or -?
+z
hahaha................
Sorry guys mere toh ans nahi aa raha hai :(
The problem will be
If x^1/3+y^1/3=z^1/3, then the value
of (x+y-z)^3 + 27xyz ?
time wast type q of ssc
thanks for pointing out error ..+2 for all and who have wasted tym sorry
Cubing both we get,
x+y-z= -3*x^1/3*y^1/3*z^1/3
So (x+y-z)^3 + 27xyz= -27xyz+27xyz=0

53. X+Y = 45, Then (1+tanX) (1+tanY)=?


1)1
2)0
3)2
4)-1
Ans- put x =0 and y=45 or x=45 and y=0...answer either 2 or 1/2...jugaadu method hai
54. sin 20 - sin 70 + sin 10 = ?
1) 1
2) 0
3) -1
4) 2
Ans-use sin C + sin D, for sin 50 & sin 10, and convert sin 70 into cos 20.
bhai tumne 70 likha hai 50 nahi
sorry typo error its sin 50 not sin 20 in Ques.
55. in triangle ABC ,angle A,B,C are in A.p then (SinA - SinC)/(CosC-CosA) is equal to
a)Sin B
b)Tan B
c)Cot B
d)Tan(A+B/2)
Ans- 2cos (a+c/2) sin (a-c/2) /2sin (a+c/2) sin (a-c/2) =cos(a+c/2)/sin(a+c/2)=cos b/sin b
= cot b
AP bola hai na.. so take 30 60 and 90,, u cn still get

56.Cos20cos40cos60cos80=? a. 3/80 b. 2/16 c. 5/16 d.1/16


Ans- Ya its 1/16, aise ques ssc me ayenge to mai to gyi :(
shortcut for this tye of que....4 Sin(60-A).SinA.Sin(60+A)=Sin(3A)
57. The length of the portion of the straight line 8x+15y=120 intercepted between the
axes is ans- 14unit,15unit,16unit,17unit, plz with some explanation.
Ans-17..
x/15+y/8 =1
rt(15^2+8^2)

58. The no 1,2,3,4,.....1000 are multiplied together.The no of Zero at the end of product
will be (kindly explain)
Ans-1000! it is ........no of zero would be equal to factor of 5 .......so 1000/5 + 1000/25
+1000/125 +1000/625 = 249
59. If cos theeta - sin theeta = rt2 sin theeta, then cos theeta + sin theeta is..?
Ans-root 2 cos theta

60. tan20 + tan 40+ rt3 tan20tan40 is..?


Ans- rt3
61. (1-1/3)(1-1/4)(1-1/5)----(1-1/25) = ??
Ans- 2/25
62.cot x + cosec x = 5, then cos x= ?
a) 5/12
b) 0
c) 1
d) 12/13
Ans- 5 ,12 ,13 triplet ...........always rremember
1+cosx/sinx= 2cos squrx/2/2 sinx/2cosx/2 we get cotx/2=5 and tanx/21/5 and now apply
the formula of cos x= 1-tan^2x/2/ 1+ tan^x/2

63.If (sin(x+y))/(sin(x-y)) = (a+b)/(a-b),then


a tany is..?
Ans- b tanx apply componendo and div idendo

64. Cos20cos40cos60cos80=? a. 3/80 b. 2/16 c. 5/16 d.1/16


Ans- Cos20Cos40Cos80=1/8 and Cos60=1/2 so....1/16
Cos20cos40cos80 ? 1/8 kaise ??
1/2sin(20)][2sin(20)Cos(20) Cos(40) Cos(80)]
[1/4sin(20)][2sin(40)Cos(40) Cos(80)]
[1/8sin(20)][2sin(80)Cos(80)]
[1/sin(20)](1/8)sin(160)
1/8 as sin(160) = sin(20)

65.tan x = 5/6 & tan y = 1/11, then x+y=?


Ans-use tan(x +y ) = (tan x + tan y)/1-tanx tany and then take inverse
x+y=tan( inverse) 1=45

66. 5^3+6^3----10^3 = ??
Ans- yaar calculation karwaeoge......use cubic series expansion frm 1-10 and subtract 14 ...formula [n(n+1)/2]^2
67. find the value of
sin20sin40sin80?
Ans- rt3/8is right
short cut formula...sinAsin2Asin4A = (1/4)*(sin3A)
68.If sin x + sin y = a and cos x + cos y = b, then tan ( x+y)/2 =?
Ans-it is a/b
69.cos ^2 ( (@-$)/2 ) - sin ^2 ( (@+$)/2) is..?
a) cos$sin@
b) cos 2@sin$
c) cos@cos$
d) sin@sin$
Ans-c....cos @ cos $
convert cos ^2 in to (1+cos 2A )/2 and sin ^2 in to (1-cos 2A)/2
simply cos(A+B) cos(A-B)

70.The value of rt(2+rt(2+2cos4theeta))..?


Ans -2cos theta
71.if 6 multiplied 15 = x10 then value of x=?
1)3 2) 3 3)3 4) 6
Ans-it should be +3 or 3
yahan + and - wala concept nahi aayega so only 3 will be the ans
bhai ye bhi batao k kyu nahi aayega?? we need explanation
simple h yar rt90 is not equal to -rt90

nope.... rt(9) = rt(3) * rt(3)..this is cool... but if we take it as rt(-3) * rt (-3) = rt.3. i * rt 3 .i..
where i = rt(-1).. then LhS not equal RHS

72.if 7x + 3 y + 9=0 and y = kx +7 are two parellel lines, then k =?


Ans- k= --7/3
for perpendicular m1x m2=-1 and for parallel m1=m2
73.Rd sharma's sum
in triang ABC, <B=35, <C=65 and bisector of <BAC meets BC in P.arrange AP,BP,CP in
descending order
Ans- BP>AP>CP

74. Find the sqrt of rt27-rt15


Ans- Its 4th rt of 3 *(rt5+1)/rt2

75. a=3+2rt2 then the value of a^6+a^4+a^2+1/a^3 is


Ans- 204
a=3+2rt3 and 1/a = 3-2rt2 and the equation is like-- {a^3+(1/a^3)}+{a+1/a}.............now
put the values of a and 1/a....:)
76.if 2cos x = x+(1/x) then 2cos 3x =
a)x^3+(1/x^3)
b)x^2+(1/x^2)
c)x^2/(1+x^2)
d)none of these
Ans- a
Using cos3x=4cos"3x-3cosx
Sorry mera ans nahi aaya??
expand (x+1/x)^3..u will get it..
Guys,kindly put questions carefully without any typo error.....Typo errors causing loss of
time & making dimaag ka dahi... :)

77. If min hand of a clock is 10cm long,how far the hand wil move in 20min?
Ans- 1/3 * 2* pi* 10
2pie r *120/360 = 20pi/3

78. 1/20 + 1/30 +1/42 +1/56 --------1/132 = ???


Ans- expand as 1/4*5 + 1/5*6 +1/6*7 and so on.....separate the numerator and
denominator by lcm and cancel all the terms....the one left wud be the answer
1/4-1/5+1/5-1/6+1/6-1/7+......1/11-1/12 = 1/4-1/12 = 1/6

79. Value of sin36 ?


Ans- 2 sin 18 cos 18....sin 18 = ( rt 5-1)/4 and cos 18 = rt( 10 + 2 rt 5 )/4....I think this
kind of sum will not be asked
Just posted ..to highlight the importance of these value....sin36 cos36 sin18 cos18
80. Find d value of tan75
Ans- tan(45+30)= 2+rt3
(rt3+1)/(rt3-1)=2+rt3

81. given the set of numbers, n>1, of which one is 1-1/n and all others are 1. The
arithmetic mean of the n numbers.
Ans- answer is 1-1/n^2

82. The Graph of x^2 -4y^2=1


parabola elipse or hyperbole
Ans-hyperbola.... out of sylllabus, i tink..
Yar aise quesn na put kro jo syllabus me nhi h
Guys you people are solving sums with so much speed........i am getting inferiority
complex... :P

83. If tan A = 1/2, tan B = 1/3, then cos 2A is


a) sin B
b) sin 2B
c) sin 3 B
d) cot 3 B
Ans- sin 2B
cos2a=1-tan^a / 1+tan^2a = 1-1/4 / 1+1/4
= 3/5
sin 2b = 2/3 / 1+1/9= 2/3 * 9/10 =3/5
84.if (x+1) and (x+2) be the factors of x^3+(a+1)x^2-(b-2)x-6 then the values of a and b
will be
Ans- ans is 2 and 8
Sandes bro m also getn a=-1 b=9
there is no option -1 and 9
please then check the ques
thik hai yar its (x-2)
Aj k liye bahut trigno ho gyi,kuch eng ka post karen
My humble requst to all puys plz give at least 1 discriptiv answr fr each questn.so ppl
finding dificulty can undrstand
Those who post question please give answer with explanation yaar .
Puys....fb pe wo feel ni aa rha jo PG par aata tha....PG pr new puys post kiye ja rhe h bt
sare senior puys yaha h....apna group 2 parts me divide ho gya h yr....
85. The value of cos 52 + cos 68 + cos 172 is..?
Ans-0
cosa+cosb=2cos(a+b/2)cos(a-b/2)
cos52+cos68=2cos60cos8 + cos 172
cos8+cos172 = 2 cos90cos82=0
@godbles 2cos(52+68/2)cos(6852/2)+cos172=2cos60cos8+cos172=cos8+cos172=2cos(172+8/2)cos(1728/2)=2cos90cos82=0

86. Base of triangle is 80, n one of base angle is 60,the sum of length of othr two sides is 90.the
shortest side is?
Ans- Draw it..
b+c =90
By cosine rule,
b^2 = 80^2+c^2-2*80*cos 60
b = 90- c.
will get 17.
87. if x+y=90 degree and sinx:siny= root 3:1 then x:y is equal to
Ans- y = 90-x
---> sinx /cosx = root 3 : 1
---> tanx = root 3
x = 60 hence y = 30 ratio 2:1
88. if (x/y)=(z/w) then (xy+zw)^2 is equal to
1.(x^2+z^2)(y^2+w^2)
2.x^2y^2+z^2w^2
3.x^2w^2+y^2z^2
4.(x^2+w^2)(y^2+z^2)
Ans-1
take the value of x=1, y=2, z=2 , w=4 now put and solve
x^2w^2 = z^2y^2... thts why... 2 xywz can be written as 2 * x^2w^2 or 2* z^2y^2
89. sin + cos = rt3 , then tan + cot = ??
Ans- 1
sq both side u will get sin2 + cos2 + 2sin cos = 3
2sin cos = 2,,,,,, sin cos = 1 = sin^2+ cos^2
divide by sincos on both side we get tan+cot=1
off course 1 no doubt
sin^2@ + Cos ^2@ + 2 sin@cos@=3 sin@cos@=1

Me along with Kaltak is suffering from inferiority complex due to level of sum u pagals r doing
with speed and accuracy, so math experts who are putting this sums kindly give detail answer.

90. if 1/x+1 + 2/y+2 + 1009/z+1009 =1 then d value of x/x+1 + y/y+2 +z/z+1009


options r 0,2,3,4
Ans- Take x=-1/3 y=2 z=-1
answer is 2 (Y) ..
91. At Daifu university, 40% of all students are both members of a student organization and
want to reduce their tuition costs. 20% of those students who want to reduce tuition are not
members of the student organization. What percentage of all Daifu students want to reduce
tuition?
(A) 20%
(B) 30%
(C) 40%
(D) 50%
(E) 60%
Ans- we need to find out (40+x)
and its given that (40+x) * 20/100 = x
x = 10
answer 40+10 = 50
40% common student is 80% of the student who want to reduce tution so all the student who
want to reduce tution is 50% of whole student

92. The max and min values of (1+cos 2x) are


a) -1 and 1
b) 1 and 2
c) -1/2 and 1/2
d) 0 and 2
Ans- d) 0 and 2
cos x >= -1
cos 2x>= -1
1+cos 2x>= 0
cos x<=1
cos 2x<=1
1+cos 2x<=2
Cos min value is -1 and max is 1 so I just added 1 in both so I got 0 and 2 is this ok to do like
this??

93. 2sin^2 cos^2 = 2, then find the value of .


Ans- 90..wahi desi method
2sin^2 - Cos^2= 2 ,,,,,, put Cos^2=1-Sin^2
we get, 3sin^2=3 ,,,,,,,sin^2=1 ,,, = pie/2 or pie

94. Length l of a tangent drawn frm a point A to a circle is 4/3 of radius.the shortest distance
from A to the circle is?
Ans- 5r/3-r = 2r/3
figure bana pdega .........waise (4r/3)^2+r^2 will give the length of the A to centre .............now
minus r frm this
95. there are three points on a circle a,b,c distance of a to b and b to c is 24cm . find the
distance between a and c, radius of circle is 20 cm.
Ans- the main point is we have to use sin 2 A... see if u draw the diagram u will get a triangle..
and sin of half tht angle is 4/5 ..but for the other side we need the whole angle tht is sin 2A= 2
sin A cos A
so as per the formula
20 =( 24 * 24 * x ) / ((4 * 24 *24 * 2 *(3/5) * (4/5)) /2)
bounce ho gaya bhai
where x is the lenght of the 3rd side
area of isosceles traingle.. (side^2 * sin of the angle )/ 2
you can also use b * rt(4a^2-b^2)/4
96. guys this is a ques frm pg... no 1 cud solve thr....,i solved dis ques ,,,need to verify my
answer....so solve it,,
A and B run in opposite directions on a circular track, starting at diametrically opposite points.
They first meet after A has run 100 meters. They next meet after B has run 150 meters past
their first meeting point. Each person runs at a constant speed. What is the length of the track in
meters?>>
Ans-Imagine that A and B start at the same place and run in opposite direction. When A and B
meet for the first time what is the distance covered by A and B together? Its nothing but the
length of the track. So all we need to find out is the Distance covered by A + the distance
covered by B
Now they actually start diametrically opposite points, so when they meet for the first time they
cover just half of the track and A covers 100 mtrs. So what distance would A have covered if

they ran the full track distance? 2 * 100 = 200


then they again meet after B covers 150 mtrs. Now both start at the same point, so they cover
the entire track distance. So B's distance is 150
So total is 200 + 150 = 350.
draw acircle put A n B on the opp.ends of diamtr..now they start moving in opps dirn(towards
each othr)..now A covers 100mtr n thus B covers x/2-100mtr(x=circumference)..nowfor 2nd
meating B move futher 150mtr to meat A that is A have to cover( x/2-100)+(x/2-50)=x-150...if
they takeT tym for 1st meating they wud take 2T for 2nd meating (frm fig).so for A -->100/T=(x150)/2T.......=>x=350..
100/x = (x+x+50)/150... if x is distance travelled by b ... and track =2(100+x)... x comes as 75.

97.If sinB = 3sin(2A+B), then the value of 2 tanA + tan(A+B) is?


Ans- Taking a=b=0 ans. 0 jugar tech. Jindabad.

98.if tan x + sin x = m and tan x - sin x = n. then (m^2 - n^2) ^2=
1.16mn
2.4mn
3.9mn
4.8mn
Ans-tan x = (m + n)/ 2 ............sin x = (m-n)/2.........(.m^2 - n^2)^2 = 16 tan^2 x sin^2 x..... =16
sin^2x /cos^2x *(1-cos^2x)........= 16*tan^2x - sin^x = 16mn
sinx=(m-n)/2and cosx=(m-n)/(m+n) then sq.and add.
m+n=2tanx=2tan45=2
m-n=2sinx=2sin45=root2
(m^2 - n^2)^2
{(m+n) (m-n)}^2
(2root2)^2=8

16mn=8
Then its ok.
@Sourav paul
wow great method

99.If P, R, T are the area of a parallelogram, a rhombus and a triangle standing on the same
base and between the same parallels, which of the following is true.
1. P>R>T 2. T>P>R 3. R= P= T and 4. R= P= 2T
Ans- well parallelogram and rhombus i think have the same area whereas paralellogram has
two triangles T so 4
100.sin A+ sin^2 A+ sin^3 A = 1. then,
cos ^6A- 4cos ^4A+ 8cos^2 A is?
Ans- Easy one sin A=1 put
A= 90 degrees to obtain th answer
Ans- 4.
I hope the above eqs has square,cubes etc
Otherwise its wrng on my part as interpretation
can anyone explain?
SinA+SIN^3A=COS^2A
SINA(1+SIN^2A)=COS^2A
sq both sides n make al d terms in cos.u wil get ans.its lengthy though
101. Two circles with same radius r intersect each other and passed through the centre of the
other. then the length of the common chord.
a. r b. root3r c root 3/2 r and d root 5 r
Ans- rt3r
Its simple use pythagorus theorm.half of length of chord=rt(r^2-r^2/4)
102. If circles are of two different radii then?...they dont pass through centres but intersect each
other. Circles with radii 3cm and 4cm. Find the length of common chord.Distance between their
centres is 6cm. then how to solve?
Ans- Ye v vaise hi hoga,yaha distance bw centre diya h.suppose one is x so another wil b 6-x
nw use the eqn 3^2-x^2=4^2-(6-x)^2 this eqn wil gv u x
Use this x and use pyathagorus theorm again to find length of chord

103.ABC is a triangle . The medians CD and BE intersect each other at O. Then triangle
ODE:Triangle ABC.
a. 1:3 b. 1:4 c 1:6 d 1:12
answer with explanation please wanna clear concept
Ans- 1:12
explain bro your method don't hesitate it will benefit us.
i think when the medians intersect it divides ABC triangle into 6 equal parts?
Median devides area also.
let AF is the median to BC.
join DEF.
Area of DEF is ABC/4
by the centroid O. DEF further devided to 3
so DEO is ABC/12
wah saab thanx maneesh got it

104.A wheel rotates 3.5 times in one second. What time in seconds does the wheel take to
rotate 55 radian of angle?
1. 1.5 b. 2.5 c 3.5 d 4.5
Ans- 2.5
1 sec m vo ghumta hai 7pie radian,to hume nikalna hai ki 55 kitne m ghumega
2pie radian= 360 .........so in 3.5times it make 2pie*3.5 radian angle in center .........so 55 radian
will make in time = 55/ 2pie*3.5
2pie radian= 360 .........so in 3.5times it make 2pie*3.5 radian angle in center .........so 55 radian
will make in time = 55/ 2pie*3.5

105. A and B are centres of the two circles whose radius are 5 cm and 2 cm respectively. The
direct common tangents to the circles meet AB extended at P. The P divides AB in the ratio?
a. externally in the ratio 5:2
b. internally in the ratio 2:5
c. internally in the ratio 5:2
d. externally in the ratio 7:2
Ans- externally in the ratio 5:2

106.The radius of circumcircle of triangle made by x- axis, y axis and 4x+3y=12


a. 2 unit b. 2.5 c 3 d 4
easy one.
Ans- 2.5
107. A number is when divided by 5, 6 , 8 and 12 it gives reminder 1 in each case, but when
divided by 13 leavs no reminder. Find the smallest such no.
Ans- 481
108. Two equal circles pass through each other's centre .if the radius of each circle is 5cm,what
is the length of the common chord?
5, 5rt3, 10rt3, (5rt3)/2
Ans- 5rt3
equation please in pythgoaras
Ok got it 2x2.5root3
109. sin(a+b)=4/5 & sin(a-b)=5/13 then tan2a=
a)16/36
b)63/16
c)36/16
d)none of these
Ans- 63/16 Tan[(a+b)+(a-b)]
110. the bisector of angle A of triangle ABC cuts BC at D and the circumcircle of the triangle at
E.then
A) AB:AC=BD:DC
B) AD:AC=AE:AB
C) AB:AD=AC:AE
D) AB:AD=AE:AC
give explanation too
Ans- A is correct option...its angle bisector theorem which states bisector of any angle devides
oposite side devides in the same ratio in which sides forming it r there.
ssc said D
dis.ques.was..also..inmy.set.r.u.representing..dis

111.A & B cab separately do a piece of work in 20 & 15 days. They worked together for 6 days
after which B was replaced by C. The work was finished in next 4 days.The number of days in
which C alone could do the work?
35, 30, 45, 40
Ans- a+b-6days work= 42/60
remaining work= 3/10 in 4 days
total will be in =10*4/3=40/3 days
c will comlete it in
3/40-1/20=1/40 or 40 days
112.If x,y satisfy eqn. atan@+bsec@=c then tan(x+y)= ?
Ans- are bhaiya tumhari given eqn mein x,y kahan hain jo satisfy karein ???
113.Easy one for relax of mind
If cosec - cot = 1/2, then cos is equal to?
Ans- Eq wil be (1-cos@)/sin@=1/2 sq both side n replace sin^2@ by cos^2@ u wil get
quad.eqn of cos@.solve it

114.Find the area of quadrilateral whose


vertices taken in order are A(-3,2) B(5,3) C
(7,-6) and D (-5,-4)
Ans- 75 Yes diavide quadiltrl in 2 two tringl then calculate area using tringl formula..u can use
matrix methd as well.
---------formula for area of triangle 1/2{x1(y2-y3)+x2(y3-y1)+x3(y1-y2)}
115. cos24+cos55+cos125+cos300+cos204 =?
Ans- .5
300 = 270 + 30... so 4th quadrant
cos(270+30)=sin30
IV quadrant
T-ratio change and cos s positive
APPLY Cos(C+D)
1/2[2cos (125+55/2)cos35+2cos114cos (224-24/2)]+COS 60=1/2
yaar woh cyclic qaudrilateral ka sum se kuch yaad nahi aa raha.. cos 55 + cos 125 = 0.
similarly,,, cos 204 = - cos 24... bacha sirf cos 300 which is 0.5

116.X cot(90+@) + tan(90+@) sin@ + cosec(90+@) = 0, then value of X is?


@=theta
Ans- yes it is sin@ need a lot of practice but i will do it anyway.
118.[cos(90+@) sec(-@) tan(180-@)] / [sec(360-@) sin(180+@) cot(90-@)] =?
@=theeta
Ans- Phle jugad k chakar ma 0 ans. Aa gya tha to propr methd lagana pada.
yes -1 sec (360-@)= - sec @
119.If cot (a+b) = 0, then the value of sin (a+2b) ?
Ans- cot ( a+b) = cot A cotB - 1 / cota + cot b >> which means.. cota . cot b = 1.. so a and b can
take 45 45 respt ..or 30 60.. vice versa :-(
Iske toh 3 ans possible ...:-)
another one is sin150=1/2
a+ b= 90 so sin(a+2b)= Sin(a+b+b)= Sin(90 +b)= cos b so we have to just find the value of
Angle b which can be found from option i think so
so finally iske infinite answer possible hai ........
120.What is the value of cos1 + cos2 + cos3 + ...........+ cos180 ?
(1,2,3,......,180 all r in degree)
Please explain the method.
Ans- cos179=-cos1...etc...cos90=0,,,cos180=-1
so ans is -1
cos 90 + cos 180 will only remain so ans must be -1
121.If A, B, C, D are angles of a cyclic quadrilateral , then value of cosA + cosB + cosC + cosD
is?
Ans- sinc Cos(A+C)/2= Cos 180/2= cos 90=0 as sum of opposite angle in cyclic quadrilateral is
180
opposite angles are supplementry in cyclic quadrilateral thic concept is used hr..cosA+cos(180A)=0.
Bhai ye to property h jiski mathematics k market ma value=0 h.
ok then u better dont use this property in maths..........use sm valuable property instead.

122.What is the value of sin(-420) (cos390) + cos(-660) (sin330) ?


Ans- -1
-3/4-1/4=-1
123.What is the value of cos510*cos330 + cos390*cos120 ?
Ans- -3 -root3 /4 is it right
Yes
-(rt3+3)/4
124.A circular wire of radius 15 cm is cut and bent so as to lie along the circumference of a
loop of radius 120 cm. what is the measure of the angle subtended by it at the centre of the
loop?
Ans- 2*pi*r=120
r=60/pi
@=l/r
@=15/60/pi=pi/4
125.the diagonals of AC and BD of a parallelogram ABCD intersect each other at the point O
such that angle DAC=30 degree and angle AOB =70 Degree then angle DBC ???
Ans- 40( alternate angle prop. And v.o. Angles)
126.when a heap of pebbles is arranged into groups of 32 each 10 pebbles are left over .when
they are arranged in heaps of 40 each ,18 pebbles are left over and when in groups of 72 each
,50 are left over .the least no. Of pebbles in the heap is
options..
1450
1440
1418
1412
Ans- 1418 desi
32*44=1408+10..40*35=1400+18..72*19=1368 +50
1418 satisfy all , brother ye 44 35 and 19 kaha se aya
127.At the foot of a mountain the elevation of its summit is 45' after ascending one kilometer
towards the mountain upon an incline of 30' the elevation changes to 60' find the height of the
mountain.
Ans- (Rt3+1)/2= 1.36km is it?

128.if N is the smallest perfect square which is exactly divisible by 10,12 and 25 then the
quotient obtained when N is divided by 25 is
Ans- 36
900/25
129.What is the value of sin10 + sin20 + sin30 + ...........+ sin360 ?
(10,20,30,......,360 all r in degree)
Please explain the method.
Ans- 0
sin values are from -1 to +1. so all get cancelled.
Use sin(180+@) =-sin@
So sin190= -sin10 n thus all get canceld
130.A number of light bulbs were purchased to illuminate a gym. However, only of them were
needed. The extra 160 light bulbs were returned. 60% percent of their cost, or $96, was
reimbursed. How much money was spent on illuminating the gym?
(A) 360
(B) 320
(C) 384
(D) 364
(E) 160
Ans- bhai A number of light bulbs were purchased to illuminate a gym. However, only 2/3 of
them were needed. The extra 160 light bulbs were returned. 60% percent of their cost, or $96,
was reimbursed. How much money was spent on illuminating the gym?
only 2/3 of them were needed....ye line kahan se aa gayi....???
actually i follow gmatclub to waha mene solve kia tha same ques.
2/3 dia hai to bht
simple ho gaya
320+64=384
131. if cos@ + sec @= rt 3 then cos^3@ + sec^3@ is equal to : a] -1 b] rt3 c] 0 d] 1
Ans- 0 first que mein take cube of both sides ..u will get the ans

132. max value of 24sin@ + 7 cos@ is : 1] 24 2] 25 3] 7 4] 17


Ans- 25 second que mein there is one direct formula for maximum value of
asinx+bcosx=rt(a^2+b^2)=rt(24^2+7^2)=25
cos^3@+sec^3@+3rt=3rt....so ans zero
rt((24)^2+(7)^2)=25
133.If sinx + siny +sinz = 0,cosx + cosy + cosz = 0...............then sin2x + sin2y + sin2z = ?
A) -1
B) 0
C) rt3
D) 1
E) NONE
Ans- x=90,y=180,z=270...sin 180 + sin 360 + sin 540= 0+0+0=0
x=30, y=150, z=270...they are satisfying both the equations...after putting the values in sin2x +
sin2y + sin2z...ans is 0
If x+y+z=180 then sinycosz+coszsiny= sinx ,multiply d given eqn by sinx and put these values.u
wil easily get d ans.where i m wrong? Checkn option upto 270 is quite tedious
What vaues are u putting in that??? It's not mentioned that x+y+z = 180....u don't have to check
the options till 270....3 values 0f sin or cos should be added up to make 0..this can be done as
1/2+1/2+(-1) or x+(-x)+0....so now from this you can assume the values.....Anyone having
shorter method to do this can explain.....

134.A litre of water is evaporated from 6 litres of glucose solution containing 4% glucose. The
new concentration of glucose is ?
Ans- yes getting 4.8..5000*x/100=240..so x=4.8
4% means 4g in 1 litre 6l contains 24g now 1l evaporates so (24/5000)*100=x s=4.8

135.There are three diffenrent type of wine each costing Rs.48, Rs.55 and Rs.75 per bag. The
bartender wants to make a wine that will cost Rs.67 per bag. In what ratio should he mix all the
three ??????????
Ans- ans is 4:1:11.....................you are right
There could be different value too but we always take the lowest one
48 and 75 will be mixed in the ratio 8:19.......and 55 and 75 will be mixed in ratio 2:3 ............so
the ratio of 48:55:75 = 8 : 2 : (19+3) =8:2:22 = 4:1:11 ..........this is the way we solve this ..........bt
the other possibility just becoz ......... we can write 2:3 above as 8:12 ........so now the three ratio
is 8: 8:31 ..........we can also write 2:3 as 6:9 so the three ratio now will be 8 :6:28 = 4 :3:14
mere paas hsortcut hai usse 8:8:31 aya. aise qs mein all three prices likh lo.then jis price ka
mixture banana hai wo in price ke uper likh do. likhna is way mein hai ki 48, 55 ,75 likhe hue
hain. new mixture 67 rs wala hai to use 55 ke baad uper ki taraf likho. then 75-67=8 fist and
second ka ratio hoga. and 67-48 plus 67-55 =31 lat wale ka ratio hoga.ye shortcut hai. aur qs
post karo and discuus akrne se ye shortcut clear ho jayega
136. ABCE is an isosceles trapezoid and ACDE is a rectangle.
AB= 10, EC= 20, AE= ??
Ans- 10
137.the sum of 4 consecutive two digit number when dived by 10becomes a perfect sqaure
.which of the followingcan be one of thoses number?
21
25
41
67
73
Ans- 21+22+23+24= 90 /10 = 9 a perfect square of 3
take four no as n, n+1, n+2, n+3 .........now (4n+6)/10 = k .....4n = 10k-6 ...........as "K" is the
perfect square so put value of K = 4, 9, 16......so tht 10K-6 is multiple of 4 ............so ans is 21
138. Three vessels having the volumes in the ratio 1:2:3 full of a mixture of coke and lemon
soda. In the first vessels ratio of the coke and the lemon soda is 2:3 , in second 3:7 and in third
1:4 . If the liquid in all three vessels were mixed in a big container , then what is the resulting
ratio of the coke and lemon soda ?
Ans- 4:11 asume quntatity 50 100 and 150 calc becomes easy. Coke:Soda=20+30+30/300-2030-30=80/220=4:11

139.Two circles touch each other internally. The sum of their area is 116 pi . And the distance
between their centre is 6 cm. Find the radii of the circles...
Ans 4 & 10
140. A room 5.44m*3.74m is to be paved with square tiles. The least number of tiles required to
cover the floor is ?
Ans- 176
find hcf of 5.44 and 3.74 it comes to be 3.4 so 5.44 x 3.74/3.4x3.4

141. (a,b) & (a+3,b+k) are on x=3y-7... K=??


Ans- put a,b into the eq.......a=3b-7
and now (a+3.b+k) in the same eq.,

you will get k=1


u may also say that slope of line would be equal to k/3=1/3..so k=1
142. In circle a chord xy subtends angle 90 with centre.& ox=oy=Radius..if Area of Triangle=32,
Area of circle??
Ans- It wud b 64pi. Triangle formed is right angled.let r b the radius of circle.den 1/2 *r*r=32 it
gives r=8 hence ar.of circle is 64pi

143.Area of circle inscribed in an eq.triangle is 154 units.find the perimeter of the triangle
Ans- 7=side/2rt3 ..side=14rt3...P=42 rt3
Pi*a*a\2rt3*2rt3=154 so a=14rt3 perimetr=42rt3
144. if sin@ =cos(@-45)where @ and @-45degree are acute angles find the degree measure
of @
Ans- @=90-22.5=67.5
135/2 equal to 67.5
cos(90-@)= cos (@-45) equate 90-@= @-45

145. If three circles(equal) of radius 3 cm each touch each other then the area of the shaded
portion is:
a)rt3/2*(2-pie) cm^2
b)9/2 *(2rt3 - pie)cm^2
c)9/2*(2rt3 +pie) cm^2
d)2/9 * (2rt3 -pie) cm^2
Ans- Rt3*6*6/4-3pi*3*3*60/360=9/2(2rt3-pi)cm"2 ans. B
rt3/4 * (6)^2 - 3pi*(3)^2*60/360=9rt3-9/2pi=9/2(2rt3-pi)
146. {1 - (1/2^2)}*{1 - (1/3^2)}*{1 - (1/4^2)}...........= ?
1 , 2 , (2/pie) , 0.5 , none.
Ans- 0.5
15/24~2/pie
147. ABC is a triangle. The internal bisector of the angles angle A, angle B and angle C
intersect the circumcirlce at X,Y and Z respectively. If angle A=50 degree, angle CZY=30
degree then angle BYZ will be
a] 35 degree b] 30 degree c]45 degree d]55 degree
Ans- a) 35
for this you can find angle <B as we have angle<CZY(GIVEN).after that you can easily find
angle C .AND so half of the angle C is your ans.
148. Circumcentre of triangle ABC is O . If angle BAC=85 degree, angle BCA=80 degree, then
angle OAC is:
a] 60 degree b] 75 degree c] 80 degree d] 30 degree
( got till angle AOC=30 degree but how to get angle OAC ?)
Ans- 75
Bhai iske liye tumne wahan tak thik hi kiya hai..........after that in the same triangle you can
easily find out the other two angles as both are same(radius).
149. In a right-angled triangle ABC, AB=2.5 cm, cosB= 0.5 , angle ACB=90 degree. The length
of side AC, in cm is : 1] 5/4*rt3 2] 5/16*rt3 3] 5rt3 4] 5/2*rt3
Ans- IT MUST BE OPTION A ...AS COS B =.5 MEANS B=60 ...NOW USE SIN B N OBTAIN AC

150. In triangle ABC , AD is drawn perpendicular from A on BC. If AD^2=BD.CD then angle
BAC is 1] 60 degree 2] 90 degree 3] 30 degree 4]45 degree
Ans- IT SHOULD BE A RIGHT ANGLE....BECAUSE GIVEN CONDITION IS ONLY TRUE
WHEN TRIANGLE ABC,TRIANGLE ABD N TRIANGLE ADC R SIMILAR SO....ABC MUST BE
RIGHT ANGLED AT A
151. The moon's distance from tge earth is 36000 km and its diameter subtends an angle of 31
min at the eye of the observer. Find the diameter of the moon.
Ans- 3470 km
I took 30 min,instead of 31,to ease d calculation
100pi..
152. If x+y+z+w = 17.then find the maximum possible value of (x-1)(y+3)(z-1)(w-2)
(x,y,z and w are real nos. greater than 2)
a)288
b)232
c)186
d)256
e)300
Ans- 256. Use d concept am>gm.
I dnt think questions on AM,GM will come in SSC
x=5,,y=1,,z=5,,w=6
4*4*4*4=256
how y=1??????????
all values are greater than 2
2*6*4*4 can be never be 256
153. If a^2*b*c^3*c^2 =256 and the value of 2a+b+3c+2d is minimum possible,then find
a+2b+3c+4d.
a)16
b)18
c)24
d)20
e)22
Ans- correct ans is 20............for getting a minimum value of 2a+b+3c+2d a=b=c=d= 2...........so
a+2b+3c+4d = 20
Oh purely desi question

154. If x=cos^4a+sin^2a, the range in which x lies is


1. 13/16<_x<_1
2. 1/2<_x<_2
3. 3/4<_x<_1
4. 3/4<_x<_13/15
Ans- 3
155. The length of the portion of the straight line 8x+15y=120 intercepted between the axes is :
1] 16 units 2] 17 units 3] 14 units 4] 15 units
Ans- 17
change to x/a +y/b = 1
rt(a^2+b^2) is the length.
156. The perimeter of a rhombus is 146 cm and one of its diagonals is 55 cm. The other
diagonal is
1]48 cm 2] 72 cm 3] 92 cm 4] 73 cm
Ans- 48 cm
a^2 = (d1/2)^^2 +(d2/2)^2

157. A can do a piece of work in 24 days, b in 52 days and c in 64 days . All begin to do it
together , but A leaves after 6 days and B leaves 6 days before the completion of the work how
many days did the work last??
15
20
18
30
Ans- 6(1/24+1/52+1/64) + 6/64 ..1 se subtract karo jo bachega thats the work done by B n C
together..nikal lo..:)
work =2496
2x2x2x3x13x2x2x2
A104
B48
C39
624 +48(x-6) +39 x
87x+336=2496
x=2160/87=720/29

158. if 3cos@-4sin@=2cos@+sin@ find tan@


Ans- 1/5
Cos@ ya sin@ se divide kr do both side

159. Pawan and Qureshi working together can do a piece of work in 10 days whereas Qureshi
and Rohit
working together can do the same work in 12 days. All three work together to do job for which
they
are paid Rs. 300. If Qureshis share is Rs. 140, then what is Pawans share?
(a) Rs. 100 (b) Rs. 60 (c) Rs. 80 (d) Cannot be determined
Ans- as in rest of 160 ,,,,,,,,,, pawan is more efficient than rohit so amoumnt of pawan is more
thats 100 here isgud option
pay ratio is proportional to work efficiency...
so work efficency is
7:5:3
7=140 then 5=100
160.What would be reminder when 2"256 is divided by 17 and 18? P.s. " means Power
Ans- 2^256 = 16^64 =(-1)^64=1
K....ab to SSC wale CAT ko fod skte h
161.A circle of radius 1 is inscribed in a sector of radius 3.the area of the sector will be??
Ans- bhai diagram bnao..ek arc bnao..aur uske andar ek circle fit kar do...then it would become
easier
@mizan
1.5pi ans aaya...
Circle ko fit karo..aur 2 radius draw karo perpendiclar to both sides of arc at A& B...abhi beech
ka angle AOB=120,so saamne wala angle of aob=60..
So pi*r^2*60/360
162. A trader makes a profit equal to the selling price of 75 articles when he sold 100 articles.
What was his % profit ?
Ans- 75X = 100X - Y
Y = 25X.............PROFIT% =300

163.The sum of the third and the ninth term of an A.P is 8. Find the sum of the first eleven terms
of the progression.
Ans- 44
11/2(2a+ 10d)
Is it in syllabus??
164. Cost of diamonds direcltly propotional to Weight^2
Cost of a Diamond=5184,we broke into 3 pieces in ratio 1:2:3 of their weights.. Loss in
cutting=??
Ans- let cost = k*w^2. let weight be 1gm, 2gm, 3gm. total weight = 6gm. now total cost = 5184.
5184=k*36. k=144. now cost of each piece = 144*(1)^2 + 144*(2)^2 + 144*(3)^2 = 2016. so loss
= 5184 - 2016 = 3186
chupchaap sq root kar aaya 72...ab use 1:2:3 mein baat aaya 12,24,36...teeno k square kar k
jod...2016 aaya ghata de 5184 se
165. 3-(3+root5)/4- 1/(3+root5)=??
Ans- bhau....second and third term ko solve karo pehle....denominator same karke....and then
subtract it from 3....solve karte raho....3/2 aa jayega
166.In triangle abc,ad-median,ad=(bc)/2,angle bad=30,angle acb=??
Ans- As ad = bc/2. and d is midpoint of bc ad=bd=dc.ADB and ADC are two isosceles triangles
formed. <bad = 30 so <abd = 30 (isosceles traingle) so <adb = 120. so <adc = 60. now in
triangle adc...as it is isosceles....rest two angles must be equal. so 2x + 60 = 180. so x = 60. so
<acd = 60
bd=ad so abc=30.....also dc=ad so...angle bcd=angle dac ..say x...now 2x+60=180 so x=60

Many puys are solving maxima minima questions using some concept "AM>GM" What is this
concept? If anyone knows please explain. If explanation not possible then give a link or tell me a
book (eg NCERT) where i can find it.

167. X+1/x=3,x^5+1/x^5=??
Ans- find x^2 + 1/(x^2). and find x^3 + 1/(x^3). multiply both to get ans
x^2+1/x^2=7.....x^3+1/x^3=18... multiply
x^5+1/x^5=123

168. A gives B a start of 10m in a 100m race and still beats him by 1.25sec. How long does B
takes to complete the 100m race if A runs at the rate of 10m/s ????????
1-8
2-10
3-16.67
4-12.5
169. A ship is moving at a speed of 30km/hr. To know the depth of the ocean
beneath it, it sends a signal which travels at a speed of 200 m/s. The ship receives
the signal after it has moved 500 m. The depth of the ocean is
170. AB and CD are two chords of a circle with centre o intersect each other at P. if <AOD =100
degree and <BOC = 70 degree find the <APC
85; 95; 55; 105
Ans- 95
171. A circular wire of radius 7 cm is cut and bent again into an arc of a circle of radius 12 cm.
the angle subtended by the arc at the centre is?
Ans- 210
2pi*12* angle/360=2pi*7
172. The ratio of land to water on the whole of earth is 1:2 and it is 2:3 on the northern
hemisphere. the ratio of land to water on the southern hemisphere is
a. 11:4
b. 4:11
c. 15:4
d. 4:15
Ans- Lets land part of earth=2x & watr part=x so total area of earth=3x
Totl Area of north hemisph.=3x/2 land partof n. Hemisph.=2*3x/5*2=3x/5=3x/5 so land part
ofsuthrn hemisph=x-3x/5=2x/5
Watr part of north hemsph=3*3x/5*2=9x/10 so watr in suthrnhem.=2x-9x/10=11x/10 so in
sthnhemi land:watr=2x/5:11x/10=4:11
173.1/rt2(cosA-sinA) find d vallue
Ans- 1/2(cos45cosa-sin45sina)=1/2cos(45+a)=sec(45+a)/2
174. find d vallue Cos0+cos1+..... cos180
Ans- 0
Cos 0 to 90 all values are positive after that upto 180 all are negative, sab cancel ho gaye

175. (x^2- x+1)/ (x^2+x+1)= 2/3, then the value of x+ 1/x is


a. 4
b. 5
c. 6
d. 8
Ans- 5 componendo- dividendo ,very easy
176. plz solve it...yar pg pe itne fad ques dalte hai ki solve nhi hota..
1.cot6. cot42.cot66.cot78=?
2.tan12. tan 24. tan 48 . tan 84=?
3. cos @+cos$=1/3 and sin@+sin$=1/4 then cos(@-$)/2=?
4. if sin@=nsin(@+2$) then tan(@+$)=?
5.sin20 . sin 40 . sin 80 =?
6.if @+$=90 , find the maximum value of sin @ sin $
7.cos 55+cos65+cos175=?
8.find the max. and min. value of 7cos@+24sin@
9.value of sin100-sin 10 is positive or negative
Ans- 1,2 type ke ques kaafi tym consuming hai inhe chodne mein hi bhalai hai.(yaa 1 maar do
option mein ho agar 1/2 marks ka risk lenaa ho).:) shayad hi ssc aise ques de....trigo 10th ke
concept pe de rakhi thi tier1 mein ....pehle thode bacho wale ques puch lete thai jinko dekhte hi
desi yaad atta tha....ab thoda traditional de rahe hai..alzebra mein trigo ghussa di coordinate
mein mensuration...trigo mein quad..
1.1
2.1
3.5/24
4.q ek baar fir se dekh lo ki n likha hai kya rhs mein ?
5.root3/8
6.1/2
7.0
8.max 25 min -25
9.positive
Ans- 3 multiply both equation, u will get the value of cos ( A-B) i.e, cos A cos B + sin A sin B as 263/288
u can write cos ( A-B ) = 2 cos ^2 ( A-B)/2 -1....
from this equation u can find cos ( A-B )/2 as 5/24
aap first n second eqn mein cosa+cos b vala n sina |+sinb ka formula lagao...dono eqn ko
devide karo...u will get cot (a+b)/2=4/3...
find cos (a+b)/2=4/5
now put that in eqn 1 u will get ans

177. is type ke ques bhi aa sakte hai kya ssc me......?


A group of 10 workers can plough a field in 20 days. This group starts the work and after every
2 days , 2 additional workers join the group. The capacity of each worker is the same. in how
many days will the field be ploughed ?
(A) 11 (B) 12 (C) 13 (D) 14
Ans- 13
10*2 + 12* 2 + 14 *2 + 16 *2 + 18*2.. til the count goes to (20 *10 = 200)
ok got it ..20+24+28+....+n=200 ...,we have to find the value of n..

178. If Sec - Tan = 4/3, then Sin= ??


Ans- 1 -ve ka kya pangaa hai ??
bhai eqn se solve karoge to smajh jaoge.. dont use desi :-P
sina/cos=4/3==>3(1-sina)=4*rt(1-sin^a)..nw taking sqr on both sides we get 99sina=16+16sina..so sina=-7/25
179. If sinx+sin^2 x=1 find cos^2 x+cos^4 x
Ans- 1 sinx=cos^2x
cos^2x+cos^4x
sinx+sin^2x=1
Bwhahahaha issme bhi Desi Method dhoondh nikal hi li=
Sinx=x
X(x+1)=0
so x=0
Now
Cosx=root(1-x^2)
Put in eq & put x=0
Then ans=1
(Though its long cut but exam me formula strike nahi hua to yahi kaam aayega)

180. A circular road runs around a circular garden. If the sum of the circumferences of the inner
and outer circles is 88 metres and the radius of the inner circle is one-third of the outer circle,
then the width of
the road is :
(A) 4 metres (B) 5 metres
(C) 6 metres (D) 7 metres
Ans- 7

181. Easy one


The radius of a circle is 30 cm. find the length of an arc of this circle, if the length of the chord of
the arc is 30 cm.
Ans- 2pi*30*60*360=10pi
chord will form equilateral triangle with centre. very easy
182. In PQR, QR = 10, RP = 11 and PQ = 12, D is the midpoint
of PR, DE in drawn parallel to PQ meeting QR in E. EF is
drawn parallel to RP meeting PQ in F. What is the length of
DF?
(a) 11/2
(b) 6
(c) 33/4
(d) 5
Ans- 5
183. Easy one
If the arcs of the same length in two circles subtend angles 65 and 110 at the centre, find the
ratio of their radii.
Ans- r inv prop q and q in radians
22/13
184. The perimeter of a certain sector of a circle is equal to the length of the arc of a semi circle
having the same radius. What is the angle of the sector in degree?
Ans- Length of the arc of a semi circle of radius r= pie*r
As (180/360)*2*pie*r
So 2r+(2pier*q/360)=pier
q=720/11
185. A horse is tied to a post by a rope. If the horse moves along a circular path always keeping
the rope tight and describes 88 meters when it has traced out 72 degree at the centre, find the
length of the rope.
Ans- 2pi*r*72/360=88 so r=70

186. if x+1/x= root 3 , then the value of


x^18 + x^12 + x^6 + 1 is
Ans- x+(1/x)=root3
X^2+1=x*root3
Cubing both
x^6+1+3x^2(x^2+1)=x^3*3*root3
x^6+1+3x^2*x*root3=x^3*3*root3
x^6+1=0
x^18+x^12+x^6+1
x^12(x^6+1)+(x^6+1)
x^12(0)+(0)
0 ans
187. Find d min value of 3cosx+4sinx+8
Ans- Val is rt(3^2+4^2)+8
=+-5+8 if u hv to find max take + n it wud b 13.for min take - and its 3
-3sinx+4cosx=0 (diffrentiate)
tanx=4/3
cosx=3/5
sinx=4/5
3*3/5+4*4/5+8=25/5+8=13
but by desi one..x=0
gives 11

188.

Ans- sin@=cos^2@
cube ( sin@+sin^2@=1)
sin^3@+sin^6@+3sin^3@=1
sin^6@+3sin^5@+3sin^4@+sin^3@-1
1-3sin^3@+3sin^5@+3sin^4@-1
1+3sin^3@(sin^2@-1)+3sin^4@-1
1-3sin^4@+3sin^4@-1
=1 - 1 =0
is ans
189. A pole broken by storm of wind & its Top stuck the ground at angle 30 & at dist 20 from
foot of pole.Ht of pole before it was broken??
Ssc ans-100root3/3
paramount ans-60root3
Ans- Sorry mera ans to 60/root3 aa raha hai...
Hyp=x,Ht=Y..
Sin30=1/2=y/x
& cos 30=root3/2=20/x
So 40/root3 + 20/root3=60/root3...is my ans correc??
yupss correct 60/rt3 or 20rt3

190. There is a work to be done by three friends A,B,C. Three of them


together take 5 hours less than A alone would have taken,one-third that B
alone would have taken and two-ninths the time C alone would have taken. How
long does the three them take to finish the work?
(a) 3 hrs
(b) 4 hrs
(c) 5 hrs
(d) None of these
suppose A took "x" hour so A+B+C will take x+5 hour ........so B will take 3(x-5) and c will take
9/2(x-5) hour ...........now we can solve 1/x + 1/3(x-5) + 2/9(x-5) = 1/(x-5)...........so ans is 4h
Take A=x+5 ,so A+B+C=x...ye jyada simple hoga
But anyway i think such sums should be solved in 2nd round at exams
191. Perimeter of triangle formed by d pts (0,0), (1,0) and (0,1) is
Ans- 2+rt2

192. Find d value of (1+tanx+secx)*(1+cotx-cosecx)


Ans- 1+sin2x-1/sinx cosx

yes 2 it is..take x=45


Yaar desi lagane ki jarurat nhi h isme,ye bina desi k jyada easy h
193. (sin4A + sin2A)/(1+cos2A+cos4A) = ?
a. secA
b. sin2A
c. tan2A
d. cot2A
Ans- traditional method : put sin4A = 2sin2A.cos2A, put 1+cos4A = 2cos^2(2A).....take sin2A
common from numrtr. and cos2A common from denom....cancel remaining terms to get tan2A
Desi Rocks A=15

194. AC and BC are 2 equal chords of a


circle. BA is produced to any point P and
CP, when joined cuts the circle at T. Then
(A) CT*TP=AB*CA
(B) CT*TP=CA*AB
(C) CT*CB=CA*CP
(D) CT*CB=CP*CA
Ans- I thik PT x PC = PA PB
Bhai ye to option hi ni hai :/
Muskil sawaal..no desi 4 this
GEOMETRY ME DESI LAGANA MUSHKIL HI NAHI NAMUMNKIN HAI
195. If tanx=b/a find d value of acos2x+bsin2x
196. If sinx=8/17 find tanx+secx
197. x is a natural number which is a perfect square...then the number x + rt x will end in
a. 0 or 5..b. 0 or 1 or 9....c. 0 or 2 or 6 d. 0 or 4 or 8
Ans- c
i started taking squares....and answer was clear by square of 3....9 + rt9 ends in 2....did u guys
use any other method?
Hahaha ....HEIGHT OF DESI METHOD
c ? assuming 4 , then 4+rt4=6
198. 19 ^n - 1 is
1. always divisible by 9 2. always divisible by 20 3. is never divisible by 19 4. only a and c are
true
Ans- 4 th option
19 ^n is divisible by 19...so 19^ n -1 will not be divisible by 19..
Put n=1 & solve
AGAIN DESI METHOD!

199. if x = 2 ^ (1/3) + 2 ^ ( - 1/3), then 2 x^3 - 6x = ?


Ans- 5
cube the given expression...x^3 = 2 + 1/2 + 3x. multiply by 2. 2x^3 = 4 + 1 + 6x. so x^3-6x = 5
try sachislyf's method.. cb. rt.2 ~ 1.25..
x=2^1/3+1/2^1/3
x*2^1/3=2^2/3+1
cube& substract 6x
2x^3-6x=4+1+3.2^2/3(1+2^2/3)-6.2^1/3-6.2^1/3
=5+6.2^1/3+6+6.2^1/3-6.2^1/3-6.2^1/3
=5
200. Rahul bhai...kro..:)

Ans- 22 (3)
23 (3)
24 (4)
25 (2)
26 (1)
27 (2)
28 (2)
201. Which is not the factor of 4^ (6n) - 6 ^ (4n) for any positive intiger n ?
a. 5 b. 25 c. 7 d.none
Ans- 2^4n ( 256^n - 243^n ) .... so apart from 2^n .. the other odd factor is 13 only ... 5 , 25 , 7
doesnt fit the bill .... answer is none
Desi Method4^6-6^4=2800 ...which was div by all 3..so none

202. sin^2C = sin^2A + sin^2B. then triangle ABC is


right angled, acute, obtuse
Ans- right angled triangel.
C=90 A=B=45
Or Put 30,60,90
Again Desi method! :P
203. Find the length of an arc in terms of pie of a circle of radius 5 cm subtending a central
angle measuring 15 degree.
Ans- 15/360*2*pie*5 = 5/12*pie
formula for finding the arc of circle is= @/360 x 2 pie r
204. a ques from pg site..
A balloon of radius x makes an angle y at the eye of an observer and the angle of elevation of
its center is z. The height of its center from ground level is
a)x cos(z/2) sec y
b)x cos z sec (y/2)
c)x sin (z/2) cosec y
d)x sin z cosec (y/2)
Ans- cat..level..bro..i..'d'..i..think..will..not..attempt..if.comes.in.paper
x cos z secy/2 i had done this question when i am preparing for engineering entrance. so don't
give them
205. what is the least multiple of 7,which when divided by2,3,4,5 and 6,leaves the reminders
1,2,3,4,5 respectively?
Ans- 7a=60b-1
119
when the divisor and there remainder have same diff like 1 then u have to take lcm of 2,3,4,5,6
=60 and then multiply it with least dividor given thats 2 .......120 and as remainder difference is 1
then -1 from this 119
206. find the least number which upon being divided by 2,3,4,5,6 leaves in each case a
reminder of 1,but when divided by 7 leaves no reminder?
Ans- for this Lcm of 2,3,4,5,6 =60 ,, and as 1 is remainder in each case so jst take max.
divisor6- remainder(1) = 5 and multiply 60*5 for complete divisibilty and add 1 for remainder
1....301

207. A train travelling with a speed of 60 km/hr catches another train travelling in the same
direction and then leaves it 120 m behind in 18 seconds. The speed of the second train is
(a) 26 km/hr (b) 35 km/hr
(c) 36 km/hr (d) 63 km/hr
Ans- use shrt cut,,,,,,,,,,,, as difference of speed = (120/18)(18/5)=24 and hence the other train
speed is 60-24=36
208. find the vallue of 1/2sin10*-2sin70*
Ans- arrange them...then use -2sinAsinB=cos(A+B)-cos(A-B) ...finally get cos80/sin10 =1
209. A man covered a certain distance at some speed.Had he moved 3 kmph faster ,he
would have taken 40 minutes less.If he would had moved 2 kmph slower,he would taken 40
minutes more.the distance(in km )is.
Ans- (3*2*40+40/60)/3-2=8 h then 8*3+8*2=40
210. An aeroplane leaves an airport and flies due north at a speed of 1000 km per hour. At the
same time, another aeroplane leaves the same airport and flies due west at a speed of1200 km
per hour. How far apart will be the two planes after
3/2hours?
Ans- Solv 100rt549=100rt(9*61)=300rt61 same ans
rt (1500sqr+1800sqr)......
211. If sin 3A + sin 3B + sin 3C = 1, then one angle must be equal to ?
a.90 b. 60 c. 120 d. none
Ans- D. none
sin 180 = sin 360...so try for just 90 and 60 by putting values, it will not satisfy the equation
212. A garden is fenced in the shape of a hexagon with each side 10m in length. A dog is tied
with a 14m leash to one of the post at one of the vertices of the garden. What are can the dog
cover outside the garden??
1-527
2-227
3-427
4-327
Ans- area of sector with 14cm radius = 240/360 * 22/7 * 14^2=392/3*pie ............area of sector
with 4cm radius = 2 * 60/360 * 22/7 * 4^2 = 16/3*pie............total sum =136*pie = 427.42

213. In a triangle ABC, if Sin A + Sin B + Sin C = 1+rt 2, and cos A + cos b + cos C = rt 2, then
the triangle is
a. equilateral b. isosceles c. right d. right isosceles
Ans- d.
as one angle is 90 deg and other two are 45 each ,,,,,, then we get above values
214. Easy one
find d max value of tanx*tan(90-x) -2sin^2 x
Ans- Eqn = tanxcotx- 2sin^2x
=cos2x whose max value is 1
215. If a+b=10 a,b>0 find d min value of (a-2)*(b-4)
Ans- instead of a=1 and b=9, take a=9 and b=1 then u ll get -21
216. In an obtuse-angled triangle ABC, angle A is the obtuse angle and O is the orthocentre. If
angle BOC= 54 degree, then angle BAC is
(a) 108
(b) 126
(c) 136
(d) 116
Ans- 126
216. In a square shaped hostel, there are 8 rooms in on each side in every floor.Room No. 101
is the first room facing north in the first floor.The room numbers are continued clockwise.Which
direction does Room No.125 face?
a)South b)North C)West D)East
Ans- simple ... 8*3+1 .... start frm (facing north means situated in south)
south..west...north...east..
so as room situated in east ,,it will face west...ans
217. A Candle Burns in 6 Hrs..Another Candle of the same height and width burns in 8
Hrs...After How Many Hours the height of the first candle will be half of the second Candle.
Ans- 2*(6-x)/6 = (8-x)/8.................calculating it we will get X= 4.8 hrs.
Kaunse book se sum hai ye??

218. Krishnamurthy earns Rs. 15000 per month and spends 80% of it. Due to pay revision,his
monthly income has increased by 20%,but due to price rise, he has to spend 20%more. His new
savings are
(a) Rs. 3400 (b) Rs. 3000
(c) Rs. 4600 (d) Rs. 4000
Ans- wrong option given by ssc.......may be it was misprinting....
3600
MR.KRISHNMURTHY WORKS AS TA IN CBEC

219. Let a, b, c be distinct such that a + 1/b = b + 1/c = c + 1/a, then abc
equals
a) 1 b) 0 c) -1 d) greater than 1 e) Less than -1
Ans- ok if
a=b=c=-1 ,ans- -1
a=b=c=1. ans- 1
a=b=c=2 , ans - 8
so a=b=c shld be always there
this is a futile question , time consuming no one is giving this type of question in tier 2, if assume
its present leave it. and ankit its clearly given in question u cannot take a=b=c and u r assuming
what not to be assumed it. chill it guys the questions will not be of IIT level !!!

220. If a + b + c = 1 and ab + bc + ca =1/ 3then a : b :c is


(a) 1 : 2 : 2 (b) 2 : 1 : 2
(c) 1 : 1 : 1 (d) 1 : 2 : 1
Ans- c

221. a^2 + b^2 + c^2 = ac + ab rt3, then the triangle is


1. equilateral 2. isosceles 3. right 4. none
Ans- 3
Bhai cosine law de rakha h decode kar lo...

222. What least number must be subtracted from each of the numbers 17, 17, 34, 42 so that the
ratio of first two is the same as the ratio of the next two?
(A) 0 (B) 1
(C) 2 (D) 7
Ans- wrng Q
it should be 18,,,,22,,,,34,,,,42 nearest possibilty ,,,,,,,,,,,,, then minus 2 from each will get the
same ratio of first two and last two numbers

223. (a+b):(a-b) = 1:5,then (a^2-b^2) : (a^2+b^2) = ?


Ans- 5/13
224. sherly started from a fixed point goes 15 m towards north and then after turning to his right
goes 15 m . then he goes 10 , 15 and 15 m after turning to his left each tym .hw far is he from
starting point ??/
Ans- 10
bhai 15 m north banao phir 15 m east phir 10 m north phir 15 m west phir 15 m south..phir
batao answer aa rha h ya nhi
225. Percentage increase in the surface area of cube when each side is increased to 3/2 times
the original length
Ans- 125
{(3/2)^2(a^2+b^2+c^2)} - (a^2+b^2+c^)
226. If x2 + y2 2x + 6y + 10 = 0, then the value of
(x2 + y2) is
(a)4 (b)6
(c)8 (d) 10
Ans- ye circle ka eqn hai centre is (1,-3)...so 1sqr+(-3)sqr=10...
227. If the square of the radii of three concentric circles are in A.P., then the square of the
lengths of the tangents from any point
to these circles are in
(1) G.P. (2) A.P. (3) H.P. (4) None of these
Ans- it forms 3 circle with same base but thr sqr of hieghts in ap so hypotan.. is in ap..

228. If the angles A, B , C of a triangle are in AP and sides a,b,c are in GP, then a^2, b^2, c^2
are in...
1. AP, 2. GP, 3. HP, d. Not in Ap or in Gp
Ans- A.P is the correct answer.
gp
one thing when it is given that bsqr =ac ,then why not b^4=a^2*c^2...thus a^2,b^2,c^2 is in gp
have to consider angles also...they are in AP..for your value, i think cos B wil not be equal to
1/2
229. .A three digit number which on being substracted from a another 3 digit number consisting
of the same digits n the reverse order gives 594. The minimum possible value of the sum off the
digits of the number is ?
Ans- (100z+10y+x)- (100x+10y+z) = 99(Z-x) =594 so z-x =6 ......consider the value of value of z
and x so that x become min ....z= 7 , x= 1 ........so no is 107 ...........digit sum is 8..............yup i
mistaken earlier
one short trick for this sum is.... 594/99 = 6... so the difference between the face values of the
nos in hundreds place and units place is 6... look in the options... u shud get,,,
230. If tanx=3 and x is in third quadrant find sinx
Ans- ASTC yaad rakho
p=3 b=1 h=rt10 and sin will be -veP/H
231. . Two pipes A and B and can fill a cistern in 12 minutes and 15 minutes respectively but a
third pipe c can empty the full tank in 6 mintues. A and B are kept open for 5 minutes in the
beginning and then c is also opened. In what time is the cistern emptied.
40,,,,,,,,45,,,,,,,50,,,,,55
Ans- tank empty= 1/12+1/15-1/6=60min
a & b kept open for 5 min = 45/60 = 3/4 part full
remaining 3/4 will empty in = 45 min

232. Find the maximum and minimum values of 6 sinx cos x + 4xcos2x..
Ans- 6sinxcox=3sin2x..ab lgalo wo asinx+bcox wala fromula..max and min value of asinx+bcox
is rt(a^2+b^2) and -rt(a^2+b^2)
4 x multiply wala hian ki alphabet wala confuse ho gaya
3 sin 2x + 4 cos 2x so min and max= +-root (3^2+4^2)
ye x mat use kiya kare
233. If sinA+sinB = a and cosA+cosB = b, then cos(A+B)
(a)a^2+b^2/b^2-a^2
(b)2ab/a^2+b^2
(c)b^2-a^2/a^2+b^2
(d)a^2-b^2/a^2+b^2
Ans- The answer given in the book is (c)...They have solved b^2-a^2 and then a^2+b^2 and
then divide and it will be equal to cos(A+B)...but don't know why they have calculated only this
and not 2ab or a^2-b^2....FF bhai solution samjhao.
Only c option wil give d formula cos(A+B)
Cos(A+B)*cos(A-B)=COS^2 A-SIN^2 B= cos^2 B-Sin^2 A
Use this formula alongwid cos(a+b) and cos(a-b), rest term would cancel n only cos(a+b) wil
remain
If we put A=B=30 then we dont get proper ans..in C we get 11/13 ..& where as cos60=1/2
Whereas if we put A=B=45 then
Options C & D comes correct
at A=B=30 ..u will get ans as c option only..as a becomes 1 and b becomes rt 3..put in option c
now...it comes 1/2
234. Distance b/w d points (acos 25,0) and (0, acos65) is ?
Ans- a
rt[(x2-x1)^2+(y2-y1)^2]

235. CGL tier I


if X = 2rt6 /(rt3 + rt2) then the value of (x + rt2)/(x-rt2) + (x+ rt3) /(x-rt3) is
1- rt3
2- rt6
3- 2
4- rt2
Ans- 2
Divide first by root 2 and take comp-div
again divide by root 3 and take comp-div
then add will take hardly 40 secs
X/ rt2 ko comp and div karo to wo (x+rt2)/(x-rt2) hoga na which is the first term X/rt2 ho jayega
........same way . second term X/rt3 ho jayega
aisa karo tum dada ka method try karo ..........it is given X=2rt6/(rt3+rt2) nw both side rt3 se
divide karo it will be X/rt3 = 2rt2/(rt3+rt2) ...........ab ispe com& div try karo ye tumhara equation
ka first term ho jayega .........phir rt2 se divide kar com & div karo wo second term ho jayega
.............nw solve
236. A number, when divided successively by 4, 5 and 6, leaves remainders 2, 3 and 4
respectively. The least such number is
(a) 50 (b) 53
(c) 19 (d) 214
i think its a wrong ques..asked last yr in tier2
Ans- ans is 94 which is not in option
why not ........214??
214
214/4 -- 53 rem 2 53/5--10 rem 3 10/6 rem 4...

237. time for tough maths..:)


1.A person starts from the origin of the coordinate axis . He travels in
this pattern . 1 unit to right , (1/2) units up , (1/4) units right, (1/8)
units down , and continues the above pattern . At what point will he
ultimately come to stop?
(a) (4/3,2/5) (b) (3/2,4/3) (c) (2/5,4/3) (d) (4/5,4/3)
Ans- for X axis take term 1 , 1/4, 1/16.............this is in GP with a=1 r= 1/4
so for infinite term its a/(1-r) = 4/3...............for Y axis it is a= 1/2 r= -1/4 ....so its 2/5
ya it should be A (4/3,2/5)......X axis--1/(1-1/4)=4/3.........Y axis----(1/2)/(1+1/4)= 2/5.
238. if a^2+ b^2 =2 and c^2+ d^2=1 then
the value of (ad-bc)^2 + (ac+bd)^2 is
Ans- desi 2
a=b=c=1 d=0
a=1,b=1,c=1/rt2,d=1/rt2
239. (2a)-(2/a)+3=0 then [(a^3)-(1/a^3)+2]=??
Ans- i think a = -2 and ans is -47/8
240. A man inversts 15680 rs in names of A,B,C in such a way that they get same amount after
2,3,4 yrs respecvly..R=5%..then ratio of amount invested at start??
Ans- (x*2*5)/100 = (y*3*5)/100 = (z*4*5)/100....2x = 3y = 4z.....let it be equal to k. so x = k/2, y=
k/3, z= k/4.....so ratio = k/2:k/3:k/4 = 6:4:3
L.C.M OF 2,3 and 4------12.....so ratio--6:4:3
022MM4 waale,
Represent karo isse.,,,,1 mark milega

241. Rohan buys 150 articles on which he has to pay Rs.50 on carriage. The
articles were marked for sale at Rs.12.50 each. Rohan sells 90 of them at this
price and the remaining after allowing a discount of 20% on the marked
price. Altogether he finds that he makes a profit of 38% on his outlay.
Calculate the amount he pays for each article.
(a) Rs.10 (b) Rs.9 (c) Rs.8 (d) Rs.7Jul 15
Ans- SP=90x12.5 + 60x10 =1725... 1725=1.38(CP)..CP=1250..subtract
50(tranporatation)...1200/150=8per piece
total SP = (12.50*90) + (80/100)*12.50*60 - 50 = 1675......CP of each article = 100/138 * 1675
* 1/150...m i right?
242. A lotus is seen 5 cm above the water level of a lake. With the onset of the wind it sinks in
the water 10 cm away from its place. How deep is the water in that place?
1) 5 cm 2) 5Sqrt5 cm 3) 7.5 cm 4)10 cm
Ans- x^2 + 10^2 = (x+5)^2
assume a lotus that is in water upto x cm and above water 5 cm.
when it sinks a right aangled triangle is formed with its dimensions
as x cm, (x+5) cm, 10 cm. using pythogorous again we get
(x+5)^2 = x^2 + 10^2
or 25 + 10 x = 100
or x = 7.5 cm....courtsey orkut community
243. ==Desi==
X^2+Y^2+2X+1=0 Than X^239+Y^478=??
0
-1
1
NONE OF THESE
Ans- Eqn can b written as (x+1)^2+y^2 thus x=-1 and y=0
244. In a right angled isoceles triangle, the ratio of the circumradius and inradius is ?
Ans- let sides be x, so hypotenuse is rt2x. circumradius of right angles tr. is hypo/2. so R =
x/rt2. Area = inradius * s. so we get r = x/2+rt2. ratio = (x/rt2)/(x/2+rt2). solve to get rt2 + 1. this
is the method i used.....did u use any short-cut
245. the least positive integer that should be subtracted from 3011*3012 so that the difference
is a perfect square..how to solve ths type of ques..
Ans- its like a(a+1)===>a^2+a..so if we subtract a from it..it will bcom prfct square..means
3011

246. Values of a & b for which (8x^3-ax^2+54x+b) is perfec cube??


Ans- for the given sum to be a cube , it should be in the form ( c-d )^3
==> c = 2x....and d = cube root b
3 c^2 d = ax^2 and 3 c d^2 = 54 x
12 x^2 d = a x^2 ==>> 12 d = a
3 * 2x * d^2 = 54 x ==>> d^2 = 9, d =3...
a = 12* 3 = 36.........b = 27...
247. P,Q,R are employed to do a work for rs 5750. P and Q together finished 19/23 of work and
Q and R together finished 8/23 of work. wage of Q in rupees is..a. 2850 b. 3750 c. 2750 d. 1000
Ans- p & Q did 19/23 work. so R did 4/23 work. R & Q did 8/23 work. so Q did 4/23 work. so
work ratio is P:Q:R = 15/23:4/23:4/23....:)
4/23*5750 = 1000
248. HEY WHAT IS THE ANSWER TO THE QUES RT 6 * RT 15 = X RT 10, THEN X = ?
ACCORDING TO SSC KEY...IS IT 3 OR +/- 3..
Ans- 3
249. If one side of the triangle is double the other, and the angles on opposite sides differ by 60
degree, then the triangle is ?
1. equi....2. obtuse 3. right 4. acute..
Right
2
Ans- again 30 60 90
250. A pipe of 2 inch diameter fills the water tank in one hour. If the diameter of the pipe is 4
inch in what time will the pipe fill the same tank?
(1) 10 minutes (2) 15 minutes
(3) 30 minutes (4) 45 minutes
Ans- r^2 = k/t.............1=k/60........k=60
now,when r=2.......(r)^2 =k/t.......4=60/t.......t=15min

251. In climbing a round pole of 80 metres height, a monkey climbs 5 metres in a minute and
slips 2 metres in the alternate minute. To get to the top of the pole, the monkey would take :
(A) 51 minutes (B) 54 minutes
(C) 58 minutes (D) 61 minutes
Ans- after 50 mins---75metres in next 1 min +5 metres..........so in 51th min. he would be on top.
in 2 min - 3 m
2*25 min i.e, 50 min - 75 m
1 min + 50 min = 5 m + 75 m
so in 51 min 80 m
252. In right handed Triangle,AD is perpendicular to Hypotenuse BC.if AC=2AB...Then BD=??
1.bc/2 2.bc/3 3.bc/5
Ans- OK fresh expalnation first find out BC...........then use the similar triangle proprty to find out
the BD =AB/rt5.........(1)...............AC^2
=BC*CD..................DC=4/rt5*AB...............BC=BD+CD...........BC=rt5*AB or AB
=bc/rt5...............NOW PUTTING THS IN (1)
BC/rt5*(1/rt5) = BD .........BD =BC/5
253. P, Q and S are moving on a circular stadium of circumference 2100 m. When P completes
one round, Q is still 700 m behind. When S completes one round, Q is 300 m ahead of him.
How far from the starting point three of them will meet for the first time ?
1) 168 2) 2568 3) 2100 4) None of these
Ans- when P covers 2100 Q covers 1400........so P:Q = 3:2...........when S cover 2100 Q cover
2400 so S:Q = 7:8..........so ratio of P:Q:S= 12:8:7.................so they will meet the starting point
in 168th round ..............so total distance covered till then is 168*2100 ..............so this will be the
point 168 m away from the starting point ..................ANS is right 168
254.Two pipes can separately fill a tank in 20hr & 30hr respectively. Both pipes are opened to
fill the tank bt when the tank is 1/3 full a leak develops in the tank through which 1/3 of water
supplied by both the pipes leak out. What is the total time taken to fill the tank?
Ans- Without leak its 12, bt after 4hr (1/3rd) its only 2/3 efficient so 4+12=16.
since leak develop after 4hr so 8hr's work finished in 12hr due to efficiency decreases by 1/3rd.
Hence total time taken by pipes is 4+12, instead of 4+8.

255. A constant distance is covered by a man at 40kmph. The person rides back the same
distance at 30kmph. Find his avg speed during whole journey.
Ans- constant dist avg speed -- 2ab/a+b , constant time avg speed--a+b/2 whre a and b r speed
using 2xy/(x+y)................2*40*30/(40+30)=240/7 kmph
256. Two pipes A and B fill a tank in 15 hours and 20 hours respectively while a third pipe c
can empty the full tank in 25 hours. All the three pipes are opened in the beginning, after 10
hours, c is closed. In how much time, will the tank be full ?
10,,,,,,,,,12,,,,,,,,,14,,,,,,16
Ans- 12
10 +2 ..
257. A man makes a proft of 20% on d sale by selling 20 articls for Re1. The numbr of articles
he bought by Re 1 is???
Ans- 20*120/100--rule of frac.
20 articles for 1....means SP of 1 article = 5 paise...
on selling it by 5 paise, he makes a profit of 20 %....so CP of 1 article = 5 * 100 /120...=
500/120...
so articles bought for 1 rupee = 100/500/120 = 100*120/500 =24
258. If m and n are natural numbers such that 2^m 2^n = 960, what
is the value of m ?
(a) 10 (b) 12
(c) 16 (d) Cannot be determined
Ans- m=10, n=6
259. 2p +1/p=4 find the value of p^3 + 1/8p^3 same as the ssc paper
Ans- (2p + 1/p) = 4 .....multiply both side by 1/2 it will be p+ 1/2p = 2.......now (p+1/2p)^3 =
2^3.........p^3+1/8P^3 + 3*p*1/2p(p+1/2p)=8..........p^3+1/2p^3 + 3/2*2 = 8 so p^3+1/2p^3 = 5
(2)'3-3(2) ANS IS 2

260. Three grades of milk are 1%, 2%, 3% fat by volume. If x gallons of the 1% grade, y gallons
of the 2% grade, and z gallons of the 3% grade are mixed to give x+y+z gallons of a 1.5%
grade, what is x in terms of y and z?
1) y + 3z
2) y + z/4
3) 2y + 3z
4) 3y + 2
5) 3y + 4.5z
Ans- x/100+y*2/100 + z*3/100 =3/2*(x+y+z)/100).............2x+4y+6z =3x+3y+3z.........y+3z = x..:)
x+2Y+3Z=1.5(X+2Y+3Z)

X=Y+3Z
261. Ram started his journey at 9.00 a.m. at 8 km/hour. Hamid started from the same spot in
the same direction at 9.30 a.m. at 10 km/hour. Hamid overtakes Ram at :
(A) 11.00 a.m. (B) 12.30 p.m.
(C) 12.00 noon (D) 11.30 a.m
Ans- 4/2=2 9.30+2=11.30 D
262. (X^4)+(1/x^4)=119 then x-(1/x)=?
Ans- My desi methodTrace X^4 =119
3^4=81
4^4=256
So x must be between 3 & 4 = 3.5
Put x=3.5 in reqd Q...ans is approx 3....& thats correct ans
X^4 + {1/(X^4)} = 119........{X^+1/X^2}^2 = 121..........X^2+1/X^2 =11
X^2+1/X^2 - 2 = 11-2............{X-(1/X)}^2 = 3^2
SO,X-(1/X) = 3
263. X=(root3)/2 then [root(1+x)+root(1-x)]=??
Ans- let [rt(1+x) + rt(1-x)] = y squaring both sides....1+x + 1-x + 2 rt[(1+x)(1-x)]
= 2 + 2rt(1-x^2) substituting value of x
= 2+ 2rt(1-3/4) = 2+ 2rt(1/4) = 2+2*1/2 = 2+1 = 3
so y^2 = 3. i.e. y=rt3

Note down My theorem( one`s attepmts & score in CGL tier2 maths paper) will be directional propotional to (No of
sums in which one successfully able to apply DESI METHODS)

264. Greatest no of 5 digits to b added to 8321,so that sum will b exactly divisible by 15, 20 ,24,
27 ,32 ,&36??
Ans- 99679+8321=108000
108000 is divisible by all digits
dekhte hi samajh agaya 108 hi to dekhna tha
start with biggest no opt b) ye 15 se hi divide nahi ho raha, other nos se check akrne ki need
nahi then check for second biggest opt a) ye bhi 15 se divide nahi ho raha then check for c)
easily 15 se divide, 20se bhi hoga ek look dek ke hi pata lag rha hai. 24 se bhi divide, 27 and 32
se bhi divide.calcualtion practice ho to 1 min ka qs hai.
265. (X^4)+(1/x^4)=119 then x-(1/x)=?
Ans- 3
266. Remainder when (17^37+ 29^37) is divided by 23 is=??
Ans- 0
in a^n+b^n if n is odd and postive integer then it will have a+b as one factor, so 17+29=46 will
be one factor.
267. (3+Root6)/(5*root3-2*root12-root32+root50)=?
Ans- Rt3(rt3+rt2)/(5rt3-4rt3-4rt2+5rt2)=rt3
268. Sunil has 4/5th of the number of berries that Rahul has. If Sunil sells the
berries at 2/3rd the price per kg at which Rahul sells, and Rahul has 20%
profit, what is the profit or loss percentage of Sunil?
(a) 10% profit (b) 20% loss (c) 20% profit (d) 10% loss
(e) neither profit nor loss
Ans- 20% loss

269. Mr.Khanna earns 1/3rd of his total income from his salary while 1/5th of the
rest by working for an office on weekends. He earns of the remaining from
royalty payment as the author of a best seller he had written some time back
and the remaining amount from investments in stocks. If he earns Rs.1200
by working on the weekends, what is the interest he gets from the
investments?
(a) 2400 (b) 1200 (c) 3300 (d) 2000
Ans- na re.. 1/3 + (2/3 * 1/5) = 1/3 + 2/15 = 7/15... bacha 8/15..which he divides into equal half
so 4/15 and 4/15.. now its given 2/15 = 1200 so 4/15 = 2400
Goru bhai gaur frmaye.assume erning 300 so salry 100..weeknd=200*1/5=40 rest 160 soby
invstmnt 80..w/i=1/2 so invst=2400
270. Three spherical balls of 1 cm, 2 cm and 3 cm are melted to form a single
spherical ball. In the process, the loss of material is 25%. The radius of the new ball
is
(a) 6 cm (b) 5 cm
(c) 3 cm (d) 2 cm
Ans- 75/100* (1^3+ 2^3 + 3^3)= 27..aftr tht take cube root of 27
271. if the length of rectangle is increased by 10 percent and breadth decreased by 10% what
will be change in area?
Ans- 1% decrease
hooori baba.........1% decrease
272. a very easy question.
1. the copper wire of length 36 m and diameter 2 mm is melted to form a sphere. the radius of
the sphere in cm?
Ans- pie r^2h= n x 4/3 pie r^3
aab toh geometry,trigo,alzebra ki itni aadat hogayi hai ki aise ques dhudne padte hai paper
mein..:) , seriously..!
karne se jyadda dhudne mein tym lagta hai..:)
273. If sinx=1/rt 10 and siny=1/rt5 find x+y
Ans- sin(x+y) nikal lo..usse aa jayega
bt thn hw v vill get the value of cos x and cos
cos^2 x = 1- sin^2x
cos x = 3/root 10 and cos y= 2/ root 5 by pythogoras theorm

274. Aj k liye bas easy quesn so dat evry1 can do it


if 35cm long line consist of x no. of pts. then no of pts in a line of length 15cm?
Ans- Infinity is d right ans
i guess it is based on the definition 'a line consists of infinite no. of points' correct me if my
theory is wrong :-)
Rssj is playing with our mathematical emotions
Ye to clas 5 ka quesn h.hw can u count no of pts on a given line
275. ABC is an equilateral triangle of side 4 cm. If R, r, and h are the circum radius, inradius
and altitude, resp. then ( R+r)/h = ?
a. 4 b. 2 c. 1 d. 3
Ans-1
R=a/rt3 , r= a/2 rt3, altitute is 4^2=2^2=x^2 (x= altitude) put values
by pythagoras theorem x=2 rt3
276. Two teams participating in a competition had to take a test in a given time.
Team B chose the easier test with 300 questions, and team A the difficult test
with 10% less questions. Team A completed the test 3 hours before schedule
while team B completed it 6 hours before schedule. If team B answered 7
questions more than team A per hour, how many questions did team A
answer per hour ?
(a) 15 (b) 18 (c) 21 (d) 24
Ans- 18
Group mein total 100+ members h par answers dene k wale Sirf 15-20 log...aur sab kaha h
yaar.??..Jo log jawab de rhe h wo toh phod k aye h aur aage b phodenge..nthn new for them..:)
277. Triangle ABC has Bc = 1 and AC =2, then the maximum possible value of angle A?
Ans- 30
i think when triangle is isoceles i.e third side is 2 angle will be greatest

278. Sin x sin (60-x) sin (60+x ) = 1/4 *sin 3 X


===>> cos x cos ( 60 -x) cos ( 60 + x) = 1/4 * cos 3 X
====>> tan x tan (60-x) ( 60 + x) = tan 3X
This seems right,....whats ur opinion..please post....
Ans- this is the way how to create sortcuts
279. The bisector of angle A of triangle ABC cuts BC at D and the circumcircle of the triangle at
E then
1- AB:AC = BD:DC
2-AD:AC = AE:AB
3-AB:AD = AC:AE
4-AB:AD = AE:AC
Ans- 1 & 4 both are possible...
280. A traveler walks a certain distance. Had he gone half a kilometer an hour
faster, he would have walked it in4\5th of the time, and had he gone half a
km an hour slower, he would have traveled5\2 hours longer. What is the
distance?
(a) 10 km (b)15 km (c) 30 km (d) Data insufficient.
Ans- Since the traveler takes4\5
th of the time he makes his speed 5/4 of original
speed. He increases his speed by 1/4.
1/4 = 1/2 km/hr.
speed = 2km/hr.
In second case he goes 1/2 km/hr slower.
3/4 of original speed
he takes 4/3 of time.
1/3 of time = 5/2 hrs.
time = 15/2hrs
distance = 2 x 15/2= 15 km
281. Find tan 15* tan 30* tan 60?
Ans- 2-3
somebody given yesterday that tan A * tan 2A * tan 4A = tan 3 a.
have to change formula....right?

282. A triangle has sides 6,7, and 8. The line through its incentre parellel to the shortest side is
drawn to meet the other two sides at P and Q. Find PQ?
a. 25/7, b. 30, c. 40/7, d. 30/7
Ans 30/7
draw the figure.........
A/S = r.....A = 1/2*base*h
find the ratio of r and h....
u will find 2 similar triangles, APQ and ABC....find PQ by equating 2 sides...
( height of triangle APQ will be h-r)
283. If A lies in the third quadrant and 3tanA -4 = 0 then,
5sin2A+3sinA+4cosA is..?
Ans- 0, yep zero sin and cos are negative in thir d quadrant
only tan and cot are positive in third quadrant.
284. ABC is an acute angled triangle with circumcentre O, orthocentre H. If AO = AH, then
angle A = ?
Ans- It means O n H are the same point..
Angle A+ Angle BOC= 180
&
Angle A= 1/2 Angle BOC
Dono se angle 60 degree aa jayega

only in case of equilateral triangle orthocentre, circumcentre, incentre & centroid will b same so
A= 60
285. If in a triangle sin A cos B = ( rt 2 -1 ) / rt 2....and sin B cos A = 1/ rt 2, then the triangle is..
1. equilateral....2. isosceles...3. right ...4. right angled isosceles
Ans- right angled very- very easy apply Sin(A+B)
286. If A, B, C are angles of a triangle such that angle A is obtuse, then tan B tan C will be less
than...
a. 1/ rt3 ...b. rt3/2...c. 1....d.none
Ans- 1

287. Easy one


A 100 ml flask contains 30% acid solution. What quantity of the solution should be replaced with
12% acid solution so that the resultant solution contains 21% acid?
(a) 50 ml (b) 44.44 ml (c) 33.33 ml (d) 64 m
Ans- Let x ml of the solution be replaced with 12% acid
solution. Therefore,
30/100*(100-X) + 12/100 *X = 21/100
X=50
288. Find d value of cot(45+x)*cot(45-x)
Ans- 1
289. If secx+tanx= k then secx-tanx =?
Ans- SECX-TANX)(SECX+TANX) = K(SECX-TANX).........SECX-TANX =1/K
289. In a km race, A gives B a start of 20 seconds and beats him by 40m. However, when he
gives B a start of 25 seconds they finish in a dead heat. What is As speed in m/sec?
Ans-10
let A in t secs complete 1000m and B in t+20 sec 960m... eqn1
also B in t+25 sec completes 1000 m... eqn2... use them to find ans

290. Find d Value of k for which (x+2) is a factor of (x+1)^7 +(3x+k)^3


Ans- put x=-2 ... 5^7 +(k-6)^3=0
(k-6)^3=- 5^7
(k-6)=(-5)^(7/3)
k=(-5)^7/3+6 =25.(-5)^1/3+6
291. In an examination, A attempted 80% of questions with 50% accuracy and B attempted
60% of questions with 80% accuracy. The negative marking is 25% for wrong answer, there is
no negative marks for un-attempted questions.
What is the difference of marks obtained by A and B?
Ans- it can be 1, 2 or anything
hmm... the marks of each question is not given..
Thx god cgl12 ka paper aap nhi bna rahe ho

292. 2 cm of rain has fallen on a square km of land.Assuming that 50% of raindrops have been
collected and contained in a pool having area of base 1000m^2,by what level would the water
level in pool have inbcreased.
Options:10m,8m,15m,17m
Ans- 10m
293. The chord RS of length 8 cm , of a circle with center C,cuts one of the
diameter PQ in a point T such that CT=TQ, If RT = 6,then the diameter of
the circle is
(a) 14 cm (b) 8 cm (c) 16 cm (d) None of these
PS-Same concept as last ques from maneesh
Ans- Let radius of the circle be r PT = 3r/2 and TQ = r/2.
(3r/2) (r/2) = 6 x 2 = 12. 3r2/4 = 12 r2 = 16. r = + 4. Hence the
diameter of the circle is 8 cm. But it is impossible to draw a chord of length 8
cm (other than the diameter) in a circle of diameter 8 cm. The circle must be
imaginary.
arre yaar ye imaginary circle kya hota hai.
i don't know bhai!!
294. What must be added to 1/x to make it x.
Ans- x^2-1/x
295. a^2+b^2+2b+4a+5=0 than a-b/a+b
3
-3
1/3
-1/3
Ans- a=-2,b=-1..so ans 1/3
296. If a+b+c = 0, then
a^2/bc + b^2/ca + c^2/ab = ?
Ans- 3
go desi a=2 b=c=-1
If a+b+c=0 then a^3+ b^3 + c^3 = 3abc

2976. If x = (16^3 + 17^3 + 18^3 + 19^3 ), then x divided by 7 leaves a remainder of


options:1,3,7,9,none of these
Ans- 0
(16+17+18+19)/7=10 so rem=0 my take
298. 3^x - 3^(x-1) = 18, then the value of x^x=?
Ans- 3^3 - 3^2 = 18.....so x= 3.......and the required ans= 27
299. (a-1)^2+(b+2)^2+(c+1)^2=0 than 2a-3b+7c = ??
Easy ques but very impt. concept wrt to Tier 2
Ans- 1, whats the concept yaar...how do u people do in split second?!
sum of square is 0 hence all of them are zero
Put a=1,B=-2,c-1...
300. In an increasing arithmetic progression, the product of the 5th term and the 6th term is
300. When the 9th term of this a.p. Is divided by the 5th term, the quotient is 5 and the
remainder is 4. What is the first term of the a.p.?
(a) 12 (b) 40 (c) 16 (d) 5
Ans- (a+4d)(a+5d) = 300..................(1)
a+8d = a+4d)*5 + 4
a = -(1+3d)
putting it in (1).
(d-1)(2d-1) = 300
d = 13
a = -40
301. What must b added to x^3-3x^2-12x+19 so dat result is exactly divisible by x^2+x-6
Ans- Na,ans is in algebric term, rd sharma clas 9th sum
x^ + 3x - 1 :-(
Its 2x+5
ha,, -2x -5 is the remainder .... yes tht shud be added to it.. didnt know wht to do with it.. :-(,,

302. AB and CD are two parallel chords drawn on two opposite sides of their parallel diameter
such that AB = 6cm, CD = 8cm.If the radius of the circle is 5cm, the distance between the
chords, in cm, is
Ans- I am also getting 7 but it is not in the options...ye question bhi galat tha exam mein..have
to represent.
whats the formula for this?
303. If x = 2+rt3, the value of (x^6 + x^4 + x^2 + 1)/x^3 is
Ans- Given eqn can b written as x^3+1/x^3 +x+1/x
x=2+rt3, 1/x =2-rt3
put it in d eqn
@rssj cgl..pehli baar kisi ladki ka maths itna acha dekha hai...
Hehe mom ne kaha tha maths aaye na aaye roti banana aana chahiye,maine advice ulta le liya
:)
304. Let C and K be constants. If x^2 + Kx + 5 factors into (x + 1)(x + C), the value of K is
(A) 0
(B) 5
(C) 6
(D) 8
(E) none
Ans- X^2+6x+5=(x+1)(x+5
on x= -1 k=6 yahi method na? short explanation bhi de diya karo ans ke saath bhai logo.
305. If sin@ + sin^2@ + sin^3@ = 1, then cos^6@ - 4cos^4@ + 8cos^2@is equal to
Ans- sin@ + sin^2@ + sin^3@ = 1
sin@ + sin^3@ = 1 - sin^2@
sin@(1 + sin^2@) = cos^2@
Sq both sides
(1-cos^2@)(2-cos^2@)^2 = cos^4@
expand kar lo u vl get the answer
306. If secx + cosx = 3, than tan^2x-sin^2x is:
Ans- 5
sqaure karo,,, scene solve

307The ratio of the angles in TringABC is 2 : 3 : 4. Which one of the following triangles is
similar to ABC ?
(A) TringDEF has angles in the ratio 4 : 3 : 2.
(B) TringPQR has angles in the ratio 1 : 2 : 3.
(C)Tring LMN has angles in the ratio 1 : 1 : 1.
(D)Tring STW has sides in the ratio 1 : 1 : 1.
(E)Tring XYZ has sides in the ratio 4 : 3 : 2.
Ans- (A) TringDEF has angles in the ratio 4 : 3 : 2.
308. Max n min value of 5cosx + 3cos(x+pi/3) + 3
Solutions any1??
Ans- Open d bracket of cos usin cos(a+b) u wil get 13/2 cos@-rt3/2sin@+3
Lagta h padhai shuru karni padegi..ppl here r very intelligent..!
= +- sqrt (sq of 13/2+sq 3rt3/2)+3
=+-7+3
thus max 10 min -4

Term vl b 13/2 cos@-3rt3/2sin@+3


Ha wo typo ho gya,bt dat doesnt matter solve krne ka idea to mil gya
309. If sec A= (x+1/4x) then sec A+ tan A ??
Ans- 2x or 1/2x
why 1/2x
sir u can write tan x = x - 1/4x or 1/4x -x
310. A and B are centres of the two circles whose radii are 5 cn and 2 cm respectively.The
direct common tangents to the circle meet AB extended at P.Then P divides AB in..
1.externally in ratio 5:2
2.internally ratio 2:5
3.internally 5:2
4.externally 7:2
Ans- 1?
the points of intersection of direct common tangents and indirect common tangents to two
circles divide the line segment joining the two centres externally and internally rsepectively in
the ratio of their rad

311. Two spheres of radii 6 cm and 1 cm are inscribed in a right circular cone. The bigger
sphere touches the smaller one and also the base of the cone. What is the height of the cone?
(a) 14 cm (b) 15 cm
(c) 85/6 cm (d) 72/5 cm
Ans- using similarity of triangles..
6AB = AC..........So,BC = 5AB........and BC = 7.
so AB = 7/5
SO HEIGHT OF CONE = 7/5 + 13 = 72/5 :)
312. easy one
Solve this
if sinx+siny+sinz = 3, then cosx + cosy + cosz is equal to:
A. 0
B. 1
C. 2
D. 3
Ans- 0
jugad..
Take x=90
313. A solid sphere is cut into four equal parts. If the total surface area of each part now is x
times that of the sphere, then what is the value of x?
(a) 1/4
(b) 1/2
(c) 3/4
(d) 3/8
Ans-
4*pie*r^2 = 4(2*pie*r^2)...
314. If total length of diagonals of a cube is 12 cm, what is the total length of edges of the cube?
(a) 15 cm (b) 12 cm
(c) 6rt3 cm (d) 12rt3 cm
Ans- D.

315. AB is a chord of a circle. AB = 5 cm. A tangent parallel to AB touches the minor arc AB at
E. What is the radius of the circle?
A. AB is not a diameter of the circle.
B. The distance between AB and the tangent at E is 5 cm.
Ans- i got (x+5)^2 = x^2 + 2.5^2 which is not possible
as x= - 1.875
yup...not possible..didnt thought that time..gud 1 ..:),Questn galat hai
316. A cone of radius 14 cm height 15 cm is cut in a plane parallel to its base. If
the area of the circle at the intersection is 154 sqm then what is the height
from the base at which the cone is cut?
(a) 10cm (b) 5 cm (c) 15 cm (d) 7.5 cm
Ans- 7.5cm
Since area of circle with BC as radius is 154 sqcm., (BC)2 = 154, so, BC = 7
cm. Triangles ABC and ADE are similar triangles. Therefore, AB/AD =
BC/DE. As BC = 7 and DE is 14 cm, BC/DE = , so AB/AD must also be 1\2.So, as AD = 15,
AB must be 7.5 cm.
So, as AD = 15, AB must be 7.5 cm. Thus Ans.d
317. Find the remainder when 1^39 + 2^39 + 3^39 + 4^39 + ... + 12^39 is divided by 39
options:0,1,12,38
Ans- 0
a^n + b^n when "n" is odd always divided by a+b .......yahan (1^39+12^39 ) +
(2^39+11^39)...............these term will be divided by 13 .........and also the equation will be the
sum of the 6 odd no so it will be divisible by 3 ...........when divided by 39 it will give reminder 0
when a^n + b^n + c^n + ......is divisible by a+b+c+.....if n is odd and a,b,c......are in AP...
here both the condition satisfies ans a+ b+ c+.....= 1+2+3+....+12 = 78.....
so the given sum is divisible by 78 and hence by 39
318. A thirsty crow stops by a spherical pot containing water. But unfortunately
the water level in the pot is too low. The smart crow put in 576 round
pebbles in the pot and the water level rises upto the top thus bringing the
water into the reach of the crow. If the radius of each peeble is 1 cm and the
pot was initially 2/3rds full, what is the radius of the pot?
(a) 12 (b) 12(3/2)^1/3 (c) 12/(3)^1/3 (d) 576
Ans- sol.)2/3rd of the post is initially filled with water therefore the total volume of the pebbles is
equal to 1/3rd of the volume of the pot. Thus if r is the radius of
the pot then 1/3 x 4r3/3 = 576 x 4/3R3. Subtituting R = 1, we get r = 12.
Thus answer a.

319. One day i started from A to B exactly at 12 noon. My friend started from B to A at 2.00pm.
We met on the way at five past four and reached our destination at exactly at the same time.
What time it was ??????
Ans- have traveled 245 min whereas my friend 125 min. Coz our meeting time and destination
time is same also total distance travel is same so ..........think it in reverse order like we have
started at the same time and meet at same time ......so from meeting time i will take 245min to
reach the starting point and my friend will 125min.......now v1/v2 = rt( t2/t1) wala concept use
karke.....v1/v2=5/7......so the distance i travel in 245min my friend will travel in 245*5/7 = 175 so
total time for travel for my friend is 125+175 = 300......so we will reach 7.00pm
320.In the triangle ABC, AB = 6, BC = 8 and AC = 10. A perpendicular dropped from B, meets
the side AC at D. A circle of radius BD (with center B) is drawn. If the circle cuts AB and BC at P
and Q respectively, the AP:QC is equal to
1. 1:1
2. 3:2
3. 4:1
4. 3:8
Ans- 3.8
Mayank bhai..use Ab*bc=bd*ac..bas bd Jo radius wo mil jygi Uske baad toh..Apke liye majak
h...:)
321. 43^444 + 34^333 is divisible by?
options:2,5,9,11
Ans- unit digit of 1st term = 1. unit digit of 2nd term = 4. additon will have unit digit 5. so divisible
by 5
waise bhi isme 2 hi options consider ho sakte hai 2 and 5. 9 and 11 se divisible karne ke liye full
term maluum honi chahiye....

322. If x, y and z are the sizes of a triangle satisfying the condition that the sum
of the products of two sides is 5, then the sum of the squares of it's sides lies
in the interval
(a) [5, 7.5] (b) [5, 10] (c) [10, 15] (d) [5, 15]
P.S-cant understand the solution given...
Ans- Given that xy + yz + zx = 5. Since (x y)2 > 0 x2 + y2 > 2xy. Similarly we
get x2 + z2 > 2xz and z2 + y2 > 2 zy, adding them we get x2 + y2 + z2 > xy + yz
+ zx = 5, (x + y + z)2 = x2 + y2 + z2 + 2 (xy + yz+ zx). As x, y and z are sides of
a triangle satisfying xy + yz + zx = 5; Maximum value of x + y + z is 5. Using
this inequation, we get x2 + y2 + z2 < 15.
5 < x2 + y2 + z2 < 15.
haan bhai as per sloution: minimum value will be 5 as it is said and proved that (x2 + y2 + z2 >
xy + yz+ zx = 5)... now for max value: put max value of (x+y+x) which is 5 in eqn x2 + y2 +
z2=(x + y + z)2 - 2 (xy + yz+ zx) \... what i feel is that this is some kind of property for a triangle
"" As x, y and z are sides of
a triangle satisfying xy + yz + zx = 5; Maximum value of x + y+ z = 5"
a direct formula for this as per the solution.. if xy + yz + zx = A than range will be (A , A*A-2A)...
use with caution i derived it from solution
323. if 2^a=4^b=8^c and abc=288 then 1/2a + 1/4b + 1/6c
Ans- 2^a = 4^b = 8^c.........2^a = 2^2b = 2^3c...........so
a=2b=3c...............b=6....a=12........c=4..............so the required ans=1/8
324. The maximum value of 3- 2cosX
Ans- 5
325. Let A and B be two solid spheres such that the surface area of B is 300% higher than the
surface area of A. The volume of A is found to be k% lower than the volume of B. The value of k
must be
1. 85.5 2. 92.5 3. 90.5 4. 87.5
Ans- 300 % higher
S2/S1 = 4/1
R2/R1 = 2/1
so
V2/V1 = 8/1
ie A is only 12.5% of B
so 87.5% less.
326. 4^61 + 4^62 + 4^63 + 4^64 +4^65 is divisible by ??????
Ans- 4^61(1+4+16+64+256)==4^61(341)...and 341 is divisible by 11 only

327. An isosceles trapezoid has a mid segment measuring 13cm and an area of 52cmsqr. If
one base has length 10cm, find the perimeter of the trapezoid ???????
Ans- bhai mid segment means its dividing the unparalleled side.mid segment =13
(given)...........so 1.5 on ri8 and 1.5 on left is the extra. so when you will draw perpendicular from
A and b to CD.you will find a similar triangle..........so using the similarity......we get CE = FD =
3cm. so we get CD = 6cm....................now we can easily find out the unparalleled sides as 5 cm
so adding all sides we get perimeter = 36cm.
Let ABCD be the trapezium, AB and DC parallel and AD and BC non parallel sides. Mid point
segment is MN. MN=13. Let DC=10. We know MN=(DC+AB)/2.
AB=16.
Draw two points E & F on AB from D & C.
AE=BF=(16 - 10)/2=3.
Area of the trapezium=(height * sum of parallel sides)/2.
52={DE * (AB+DC)}/2.
52={DE * (10+16)}/2
DE=4.
DE=CF=4.
AE=BF=3.
By pythagorus,
AD=BC=5.
So perimeter=AB+BC+CD+DA=16+5+10+5=36.

328. When 25^25 + 26 is divided by 24 remainder will be ??????


1-1
2-2
3-3
4-none
Ans- 3
329. Sum of n terms of following series will be :
(2n 1) + 2(2n 3) + 3(2n 5) +
(a) 1/6 n2 (n+1)(2n+1) (b) 1/6 n (n+1) (2n+1) (c) 1/6n (n+1) (2n+1)2
(d) None of these
Ans- Isme desi lagega:-)
bhai dessi hi lahaya hai... n k values put karlo.. it will be 1,2,3,4...
ansB)....value dalo n ki smallest...ques khtm.
Bhai n ki value 2 rakho..2terms ka sum 5 ayga..phir options check karo..b will satisfy..n agar 1
lete h toh Kao bar panga pad jata h so I take 2 or more

330. the length of the common chord of two ciecles of raddi 15cm and 20cm, whose centers
are 25cm apart, is
1-24
2-25
3-15
4-20
Ans- pythagoras triplet---7,24,25
331. 6a^-2bc^-3/4ab^-3c^2 divided by 5a^-3b^2c^-1/3ab^-2c^3
options
9/10ac
9/10ac^-1
9/10ac^2
9/10ac^-3
Ans- 9/10ac^-1
koi easy method ho to plzz batao...
332. The numerator of a fraction is a multiple of two nos. One of the nos. is greater
than other by 2. The greater no. is smaller than the denominator by 1. If the
denominator is given as 5 + c (c is a constant), then the minimum value of the
fraction is
(a) 2 (b) 2 (c) 1/2 (d)
Ans- k...when t =2, the fraction is ( 2-3) ( 2-1) / 2 = - 1/2...
{(4+c)*(2+c)} / (5+C).........for this to be minium C should be negative ..........onlly -3 satisfy this
.........next method.........(5+C)>=(4+c)*(2+C)..........it will lead to C^2+5C+3<= 0..........only four
value will satisfy -1 ,-2 ,-3 , -4 ........so C= -3.........minimum value of fraction -1/2

333. If cosec A - sin A = a^3,


sec A- cosA = b^3 then,
a^2b^2(a^2+b^2) is..?
Ans- Guys sorry for late explntn..
cosec A - sin A = a^3
=
cos^2 A = a^3 sin A ------ (1)
sec A- cosA = b^3
=
sin^2A = b^3cosA
cosA= sin^2A/b^3
put thos in equation 1.
will get,
sinA =ab^2
so
cosA = a^2b
then square and add these two equations. will get 1.
334.The fraction of the volume of the cone left after cutting it by a plane parallel to its base at
half the height is ?
1- 7/8
2- 1/8
3- 1/4
4- 1/2
Ans- uper wala 1/8 niche wala 7/8 kaunsa pucha hai
7/8
sir height decrease by half so radius decrease by 1/2
335. If sec A = x + 1/4x, then sec A + tan A = ?
a. 2 x, b. 1/2x, c. 2x or 1/2x, d. none
Ans- 2x or 1/2x
hint : find tan ^2 A using sec ^2 A - tan ^2 A =1...find tan A....u will get + or - ( x - 1/4x)....add sec
A and tan A..
336. 1/2*4+1/4*6+1/6*8+..........+1/18*20=?
Ans- 1/2{(1/2-1/4)+(1/4-1/6)+...,
=1/2(1/2-1/20)
=9/40
Shortcut= total number of terms/2*20 =9/40=0.225

337.--Platform Dose-A^2=B+C, B^2=C+A, C^2=A+B THAN


1/(A+1) + 1/(B+1) +1/(C+1) = ??
1
2
-1
3
Ans- isme ek condt. aur lagegi ques ke saath ki a,b,c non zero hai
a=b=c=2
Ans shud be 1.
bhai condition should be a,b,c not equal to -1.
338. If 3 sin 2 A = 2 sin 3 A , then sin A = ?....no options...
Ans- rt 15/4 is right.
6 sin A cos A = 2 ( 3 sin A - 4 sin ^3 A )...
divide by 2 sin A = > 3 cos A = 3 - 4 sin ^2 A...
convert all to cos, find the value of cos...u will get it as - 1/4...find then sin.
339. In a triangle ABC, the lengths of the sides AB and AC equal 17.5 cm and 9 cm
respectively. Let D be a point on the line segment BC such that AD is perpendicular to BC. If AD
= 3 cm, then what is the radius (in cm) of the circle circumscribing the triangle ABC?
Ans- abc/ 4 A = 17.5*9* BC /( 4* 1/2 * BC * 3)
Ye kya kiye bhai
area of triangle = abc / 4R... where R is the circumradius ab karo.
340. What is the number whose square is equal to the sum of the squares of 4683
and 4460.
(a) 6467 (b) 5442 (c) 1170 (d) 7863
Ans- dekh bhai.. u need to make some approximations... 1. eiither it has to be a or d (tht to u
cud made ot out urself ).. 2. this is where the trcik lies.. 4683 ^2 .. is anything but much less than
5000^ 2... which has starting digits 25.. and simliarly for 4460... the starting two digits must be
lvery less than 25.. now the option D has 7863..whose squARE CAN AHVE STARTING DIgits ..
which are obviously greater than 50. (why ?? becasue tht no is closer to 8000..whose square
will have starting sigits as 64...).... so it has to be A
gem of an explanation sir..!

341. If x = sec A - tan A , y = cosec A + cot A, then xy + 1 ?


a. x +y, b. x-y, c. 2x + y, d. y-x
Ans- find xy + 1...i.e, (sec A - tan A ) (cosec A + cot A) + 1....convert al terms to sin and
cos.......then expand....
divide each term of the numerator by denominator.....ten -1 and +1 will cancell each
other....result willbe y-x
Put theeta=45
342. The minimum value of sin ^6 A + cos ^6 A is ?
Ans- (sin^2a+cos^2a)^3-3sin^acos^2= 1-3sin^2acos^2a=1/4 (a=45)
Put A=45....& then matter khatam...
Get 1/4...
But no idea why put A=45 only..
Remembering what anuj has saidWhen sin & cos r there put A=45 only
sin ^6 A + cos ^6 A = ( sin ^2 A + cos ^2 A ) ^3 - 3 sin ^2 A cos ^2 A ( sin ^2 A + cos ^2 A ).....1 3 sin ^2 A cos ^2 A...= 1 - 3/4 (sin 2A)^2
The minimum value of the function is then 1 - 3/4 * 1...as max value of sin 2 A = 1
343.A no N is a positive three digit number.If x is in its hundred's place and y is in its unit's
place ,then the number
N-100x-y is always divisible by.
A.8 b.9 c.10 d.11
Ans- Assume any three digit number like 113 113-100*1-3=10 ans. Divisible by 10
344. A man purchased 40 fruits: Apples and oranges for Rs.17. Had he
purchased as many oranges as apples and as many apples as oranges, he
would have paid Rs.15. Find the cost of one pair of an apple and an orange.
(a) 70 paise (b) 60 paise (c) 80 paise (d) 1 rupee
Ans- c 80 paise
345. If tan ^2 A = 2 tan ^2 B + 1, then co2 A + sin ^2 B = ?
1. 0, 2. 2 sin ^2 B, 3. - cos 2 B, 4. None
Ans- woh cos^2 A hai na... to sin^2 A.. or none...

346. Cos A + cos B + cos C < = 3/2


Sin A + sin B + Sin c < = 3rt3/8
tan A + tan b = tan C > = 3 rt 3
sec^2 A + cosec ^2 A > = 4
Ans- I gues equality for first three holds 4 equilatrl triangle only
yes...equality holds for equilateral only..
347. Evaluate the expression 4x^3 + 2x^2 - 8x + 7 when x = (rt3 + 1)\2
(a) 5 (b) 10 (c) 20 (d) 25
Ans- take 2x^2 common n simplify..2x^2(2x+1) - 8x+ 7..!
x = (rt3 + 1)\2=
(2x - 1)^2 = 3 or 4x^2 - 4x - 2 = 0 or 2x^2 - 2x - 1 = 0.
Now, 4x^3 + 2x^2 - 8x + 7 = (2x^2 - 2x - 1)(2x + 3) + 10. Since 2x^2 - 2x - 1 is zero,
the given expression 4x^3 + 2x^2 - 8x + 7 = 10
348.Find d minimum value of (x^2-6x+13)
Ans- (x^2-6x+13) = x^2 - 6x + 9 + 4 = ( x-3 ) ^2 + 4...the min value of (x-3)^2 is 0....so minium
value of the function is 4
After differentiation, 2x-6=0 comes ...to usse min /max value kaise milegi
2x-6=0..so x = 3..put it in the function..ul gt minimum value..!
349. H is the orthocentre of a triangle ABC. What will be the value of AH ?
1. a tan A, 2. a cot A, 3. b cot A 4. c cot A
Ans- a cot A
350. If x/a * cos A + y/b * sin A + 1 = 0 and x/a * sin A - y/b * cos A -1 = 0, then ( x^2 / a^2 ) + (
y^2 / b^2 ) = ?
2,0,-2,1
Ans- method is given by rssj....arite the constants of both equations ( i.e, 1 and -1 ) in one side
....square and add....it wil be like . A + cos ^2 A ..
Put A=0 x=-a,y=-b & ans=2
bro itne dangerous question kaun si book se dete ho

351. A test has 50 questions. A student scores 1 mark for a correct answer, 1/3 for a wrong
answer, and 1/6 for not attempting a question. If the net score of a student is 32, the number of
questions answered wrongly by that student cannot be less than
1. 6
2. 12
3. 3
4. 9
Ans- Yes..
x + y + z = 50
And
The second equation can be written as,
6x 2y z = 192
Adding the two equations we get,
7x-y=242
Since, x and y are both integers, y cannot be 1 or 2. The minimum value that y can have is 3.
This Q was asked in CAT :)
352. Maximum value of function sinx(1+cosx)--- 3, 4, 3rt3, 3rt3/4
plz thoda xplain bhi kar diyo bhaio
Ans- Usin diffrn we get x=60 put it in d eqn to get 3rt3/4. I didnt find any other method :(
ssc me aise quesn nhi ayenge i.e diffrn based ssc aur JEE me kuchh to fark hona chahiye :)
Iss que ko options se bhi kar sakte hain as 1+cosx ki value hamesh 2 se kam hi hogi nd phir ise
sinx se multiply kiya toh wo bhi 2 se less hi hoga... all options are greater than 2 except lasst
one 3rt3/4.
353. In a 500m race Runman beats Bhagtaram by 100m. In 800m race Bhagtaram
beats Padtabhau by 200m. By how much did Runman beat Padtabhau in a
race of 1000 m?
(a) 300m (b) 500m (c) 400m (d) 600m
Ans- 400
when R cover 500m than B cover 400 ........when B cover 800m then C cover 600 ........so when
B cover 400m then C will cover 300............B is equal now in both condition so now compare R
and C ........you will get the ans
second method ........ratio hai ..........ratio of the speed of R and B is 5:4 .........B and P is 4:3
...........now find R:P .........

354. The sides of the triangular piece of ground measure 15547, 17647, 3521 feet
respectively. Find the length of the largest hurdle that can be used to fence it
exactly without bending or cutting a hurdle.
(a) 6m (b) 6.5 m (c) 7 m (d) 7.5 m
Ans- 7m
i guess answers are wrong. and i think ssc wont ask conversion problems. it may be ibps quest.
if u know ans pls tell
Length of the largest hurdle = HCF of 15547, 17647, 3521 which is 7.
355. on the same logic,
how many times the minute and hour hand will coincide in a span of 3 days?
a)22 b)33 c)44 d)66
Ans- 22*3
it is same for 180 degree or straight line by hour and minute angle.

356. Find the maximum value of Sinx.Cosx


Ans- hi bro...to get max value u substititue same values sin 45= 1/rt2 cos45= 1/rt2
sin45cos45=1/rt2*1/rt2=1/2\
multiple divide by 2.....u will get 1/2 * sin2x.....sin2x has max value as 1....so we get answer 1/2
357. Two vessels A and B contain milk and water mixed in the ratio 5:2 and 8:5
respectively.Find the ratio in which these mixtures are to be mixed to get a new mixture
containing milk and water in the ratio 9:4.
Ans- yaar 7:2 hi ayega..use aligation..milk in first is 5/7..milk in second is 8/13..milk in mean is
9/13..so resultant raio is (9/13-8/13):(5/7-9/13)==1/13:2/91==7:2
first me milk=5/7 .water=2/7.........
second me milk=8/13...water=5/13
x quantity of first mix with y quanity of y,will make
milk : water ratio
so
(5/7 x+8/13 y) : ( 2/7 x + 5/13 y)= 9/4
20x/7+32y/13=18x/7+45y/13
2x/7=13y/13
or x / y = 7 /2
so 7:2 is answer
ye un logo ke lie jinhe allegation smjh nhi baithti.

358.How many times are the minute hand and the hour hand at right angle in a
week?
(a) 156 (b) 308 (c) 168 (d) 161
Ans- In one day the minute hand and the hour hand are at right angles 44 times.Therefore in a
week they will be 44 x 7 = 308 times at right angle position.
relative speed of the minute and hour hand is 11/2 per minute. 11/2*X =90 ==> x=180/11. and
12 hrs = 360 so 1 day= 720. so 720/(180/11) = 44 times
and for a week it would be 44*7 = 308 :)
Sincere request to Question putters
1.plz avoid typo errors
2.dont put IIT-JEE/IES level Qs
3.plz try to give possible desi solutions
Otherswise our brain`s mother & sisters get united
359. easyone..
Aashish writes letters to four of his friends. He asks each of them to copy
the letter and mail to four different persons with the request that they
continue the chain similarly. Assuming that the chain is not broken, and
that it costs 25 paise to mail one letter; what will be the total amount spent
on postage (in rupees) till the 8th set of letters is mailed?
(a) 17640 (b) 21845 (c) 12880 (d) None of these
Ans- Ye shankutla devi se utha k patak rhe ho kya.
AAP dono ke level ka nhi h yaar ye..:P..
Muru bhai ye toh ek PDF h maths ki usme Jo thik thak se ques lgte h wo dal deta hu..
Total no. of letters = 4 + 4^2 + 4^3 + + 4^8 = 4 (4^8 1)/(4-1) = 87380
Total money spend = (25/100)x 87380 = Rs.21845
360. 2^3^4^5 - 2^3^5^4
Find the unit place????
No options for this just give me ur ans..:p
Ans- 4
bhai pehle wale ka unit digit 2 hoga and dusre wale ka 8
bhai first wale ka unit digit 2 kaise aya..?mera 6 aa rha hai..2^3^4^5==8^1024=8^4*256..and in
8^4 has unit digit 6..so 8^4*256 will also have unit digit 6....P.S m week in such type of
questions..
In first digit 2^3^4^5 ; 4^5 has unit digit 4, now 3^4 has unit digit 1 so 2^1 =2 similary second bhi
solve karlo

361. From the money that Ajay has, he gives as much to Bunti as much as Bunti
has with him. Bunti then gives as much as money to Chintu as much as
Chintu has with him. Finally Chintu gives as much money to Ajay as much
as Ajay had before Chintu gave hime the money. How much money did
each of the three friends Ajay, Bunti and Chintu have with them initially if
between them they have totally Rs.48 and after the transactions each of
them have equal amounts?
(a) 24, 14, 10 (b) 20, 16, 12
(c) 22, 14, 12 (d) 26, 12, 10
Ans- c
Exam 2 ghante toh ye Q padhte padhte hi nikal jaayenge....
Koi solution batao..
bhai...in ques me go from last to first...and also desi way is to use options ..no points of using
complex equations..
I request all the puys to give solutions, or atleast hints to find answers for those puys who are
not much strong in maths...it will be helpful...other wise, i think, they may feel inferiority complex
and it will affect their performance in tier 2......please....post ur solutions also..
362.if [(1-sin^2@)sec^2@+ tan^@]*(cos^@+1) when 0<@<90* ?
2
>2
<2
>=2
Ans- above equation has two typo error,,,,,, tan^2@ and Cos^2@ instead of tan^@ and Cos^@,
after putting solve this and get 1+Sec^2@ which is always greater than 2 for any value of @ in
between 0 and 90
Put theeta=45...ans =infinity>2...baat khatam
363.What is the smallest number which has divisors ending in every digit from 0 to 9?
Ans- my explanation = 2^3*3^2*5*7yes ryt ans is 2520 as it is divisble by 0-9 and also for 0-10
we have to take the LCM upto 9 or 10 whcih comes out to be 2520
Exam me char options rahenge...smalles option se chaalu karo
364.If P9 = 1 + 22 + 333 + 4444 + + 999999999, then P9 P8 is
(a) 10^9 (b) 10^9 - 1 (c) 10^9 + 1 (d) 10^8 1
Ans- P9-P8 = 9999 (9 times)
= 9 + 90 + 900 + + 900000000
= 9 (1 + 10 + + 108) = 109 1
Bhai intelligent wahi h jo exam me score kar gaya... yaha to koi b kar leta h...

365.Here comes..
1. tan ^6 20 - 33 tan ^4 20 + 27 tan ^2 20 = ?
1. 1 2. 2 3. 3 4. None
Ans-t an 3 A = ( 3 tan A - tan ^3 A ) / ( 1 - 3 tan ^2 A)....
tan 3 A * ( 1 - 3 tan ^2 A) = ( 3 tan A - tan ^3 A )
square both sides....and put A = 20....tan 3A = tan 60...put its value....and expand..ur answer
will come as 3.
366.The value of 3/4 + 5/36 +7/144 +..........+ 17/5184 + 19/8100 is ??????
1- 0.99
2- 0.98
3- 0.95
4- none
Ans- 1
Can a written as (1-1/4+(1/4-1/9)+....
Bich k term cancel ho jayenge.
367.(a^2-b^2)sin@+2ab cos@=a^2+b^2 find tan@

1/2 (a^2-b^2),
1/2ab (a^2-b^2),
1/2 (a^2+b^2),
1/2ab (a^2+b^2)
Ans- Divide both by a^2+b^2, then sin^2+cos^2=1
Best by going through option. If RHS is 1, then find tan and then other trigo ratio
368. cos 2 A + 2 cos A is
a. > -3/2, b. >= -3/2, c. < = 3/2, d. < 3/2.
Ans- 2cos^2a-1+2cosa= 2cosa(cosa+1)-1... -1 to 3... ans shld be a
369.Find d value of sin^6x+cos^6x+3sin^2x+cos^x
Ans- 1
sin^6x+cos^6x+3sin^2xcos^2x ye to nhi
370.Two vertices of a rectangle lie on the line y= 2X + C and cordinate of the rest two vertices
opposite to each other are (1, 3) and (5, 1). find the value of C ?
Ans- mid point is 3,2.....2=6+c .... c=-4
its right ans is -4

371.If P and Q are points on sides AB and AC respectively of trian. ABC and AP=3cm , PB
=6cm AQ= 5cm and QC=10cm then-- which one is correct1)BC=3PQ
2)BC=2PQ
3)BC=PQ
4)None
p.s: TYPING QUESTIONS SO PLS IGNORE SPELLING ERRORS (IF ANY)
Ans- bc=3*pq
372. If d larger base of d trapezium equals a diagonal and d smaller base equals d altitude, find
d ratio of smaller base to the larger base
Ans- Rectangle ma 4"2+3"2 ka set hoga to diagonal 5 aayga to larger side=5(4+1) aayegi and
altitude 3 so small base=3 so 3/5
Let smaller side= s, diagonal=t nw the larger side wil b s+a+a=t ,a=(t-s)/2 nw use pythagorus
th. To get reln bw s and t
Here larg base=5=diagonal large base ka 4unit diagonal wale tringl ma hoga and 1 unit side
wale triangle ma.
373. SinA.sin2a.sin3a.....Sin(na) =sin3a/{2^
(n-1)}
Am i correct??
Ans- Ok...tried to invent this formula but its not correct...otherwise would had/have won Noble
prize in/for Maths
*Find error in sentence `/`*
374. Doubt: Q.116, 1st july Morning shift, Test Form No. 012 NL 5
Two circles of radii 4 cm and 9 cm respectively touch each other externally at a point and a
common tangent touches them at the point p and q respectively. Then the area of a square with
one side pq is:
A) 72 sq. cm.
B) 144 sq. cm.
C) 97 sq. cm.
D) 194 sq. cm.
I've marked D, 194 sq. cm.
Ans- (9+4) ^2 = (pq)^2 + ( 9-4)^2
matlab maine hi galat solve kiya hai, SSC ne B hi diya hai

375. find the area of triangle whose vertices are (2,-2)


(-8,12)and(-6,-10)
Ans- 96
use the formula - -- 1/2{x1(y2-y3) + x2(y3-y1) + x3(y1-y2)}
96 by martix method.

376. minimum value of sinx^2+cosecx^2=?


actually the minimum value of x square + inverse of x square is always 2.so,if we replace
cosecx as t then t square + inverse of t square,then we can say that the answer is 2.
Am i right friends,if not,then plzz correct me!!!
Ya right and it comes from the fact dat AM>GM
As am>gm, so sin^2x+1/sin^x > 2rt(sin^x.1/sin^x)
which equals to 2
Wrong post. It woud be
(sinx - cosecx)^2 + 2
So the min value is 2
right bro ...ye exam nhi click kiya mind me..easy 1..!!
is it 2 since x found to be 90 degree
i hv used differentiation
cosec^2 x ka differentiation kya hota hian bhai
2cosecx * (-cosecxcotx)
377. One interior angle of a regular polygon measure 177*. how many side does it have?
Ans- 3N =360 SO N=120 :)
120 correct? As its 'one' , not 'each' interior angle
360/180-177.... 120
378. if in triangle ABC angle ABC=90 degree and BD is perpendicular to AC ,BD=4 cm,AD=2cm
CD=?
Ans- 8
answer wud be the same;as corollary also says.......bd^2 = ad * cd.:)

379. A cone has angle of 60deg at vertex,if its curve surface area is 100.find volume
Ans- see since the angle formed at the vertex is 60..we will have a 30-60-90 triangle..
so height h = rt 3/ 2 * l
now pi*r *l = 100 so pi* r^2 /3 = 50/3 .......eqn 1
also pi * l^2 = 200 so l = rt (200/pi)
so h = rt3 / 2 * rt (200/pi).......eqn 2
multiply eqn 1 and 2 to get

380. If in triangle, angle CAB =90degree and AD perpendicular to BC. Also AC=75cm, AB=1mt
and BD=1.25mt find AD?
Ans- Angle bac = angle bda =90degree and Angle B=B So trian BAC~ TRIAN BDA therefore
BA/BD = AC/AD putting values AD= 93.75cm
bhai i got the solution ; but iF you will see in RIGHT ANGLE tri(ABD),AB<BD.THIS cant be
possible simply because in any ri8 triangle hypotenuse is always greater than two the other
sides.correct me if am wrong
yes you are right, but the logic they used to prove is also correct na?
is bade wale triangle main angle A 90* hai ........aur tumne diya hai AB=100cm ,AC=75cm ...so
BC would be 125cm so how BD can be 125cm ...untill point C and D ek na ho to
bhai there must be some error;it's just that we r unable to spot that error.
yes may be some error.... RD sharma ka ques hai
382. sin10*sin20*sin40*sin60= ??
Ans- sinq*sin2q*sin4q=1/4*sin3q--shrt cut
rt3/16
Yaar 3/16 aana chahiye tha
1/4*root3/2*cos60=3/16

383.sum of the four consecutive two-digit odd numbers, when divided by 10, becomes a perfect
square. Which of the following could pe possibly be one of these four numbers ?
1-21
2-25
3-41
4-67
5-73
Ans- take four conecutive no as 2k-3, 2k-1, 2k+1, 2k+3 now sum it ........ie. 8k ....when divided
by 10 it gives a perfect square so k=5 ,20 so on........bt so 41 is right ans
8K is the sum of the four digit.......now find out the value of K so that it would be divisible by 10
and the qoutient is a perfect square. When k=5 quotient comes 4 which is a perfect square ....bt
it gives no as 5,7,9,11.....bt all are nt two digit no. .....now try if k=20 then quotient comes 16 , a
perfect square.......so no is 37,39,41,43
384. 4X =SIN@ & 4/X =TAN @ THEN 8(X^2 - 1/X^2) = ?
1/2 , 1/4 , 1/16 , 1/8
Ans- cos@ = x^2...put the value in exp and solve
1/16
Take @=45
Sorry friends yeh mere dimaag ke upar gaya....bhagwaan bachaye muzhe maths
385. x + y = 45, then (cot x -1)(cot y - 1)..?
cot(x+y) = 1....cot(x+y) = cotxcoty-1/cotx+coty....expand it...answer will be 2
See after expansion ans comesCotx+ coty-cotxcoty+1=0
Phir yaaro ans 2 kaise aa raha hai
Expand this (cot x -1)(cot y - 1) too
Any desi method??
386. What is the relation between the angle formed at orthocentre and the vertical angle?
Ans- 180- angle..
387. Sides of a triangle are 10,12,15 and the altitudes are 14,11 and 13.find d area of the
triangle. No option
Ans- not possible
sum of altitudes shud be less than sum of sides of triangle

388. if the lenght of the side PQ of the rhombus PQRS is 6cm and <PQR = 120. then length of
QS is?
a. 3
b. 5
c. 4
d. 6
Ans- bhai figure draw karo triangle rsq me QS/sin 60 = 6/sin 60, therefore QS = 6
all sides of rhombus are equal.
Waise b vo RSQ tr equilateral h
ff bro.....kya diagonal 120* ke angle ko bisect karta hai? is it a property?
yes bhai correct...
Somebody make PDF of DESI METHODS in maths also
389. 4b^2+1/b^2=2 than 8b^3+1/b^3 =
0
1
2
5
Ans- by ading 4 get 2b plus 1/b
then by cubing
get 0
(2b + 1/b) = rt6. now cubing. 8b^3 + 1/ b^3 + 3*(2b)*(1/b)*(2b+1/b) = 6rt6. see the last term. b &
b cancel out. and substituting 2b + 1/b = rt6 last term reduces to 6rt6. 6rt6 on both sides
canceled to get 0 as answer....even i was stuck for long time....dont know...agar aisa aya mains
me to lag gayi...
390. ABCD is a cyclic quadrilateral. Sides AB and DC, when produced meet at the point P and
sides AD and BC, when produced meet at the point Q. If Ang ADC=85 and Ang BPC=40, then
Ang CQD =?
Ans- angle ADC + angle ABC = 180 as it is cyclic quadrileteral .....so angle ABC = 95 ...so angle
CQD = 180-55-95 = 30*
391. If one side of a triangle is 12cm and opposite angle is 30degree,then diameter of
circumscribed circle is? Ye ssc type quesn h
Ans- 24
2r=a/SINA

392. If x+y=z find the value of cos^2x+cos^2y+cos^2z


Ans- put 30 60 and 90... simple
ans is in book is 1+2cosx.cosy.cosz
393.the length of the side of a square is 14 cm find out the ratio of the radii of the inscribed and
circumscribed circle of the square
Ans- inscribed circle's diameter would be the side of square and circumscribed on will have
diagonal as its diameter. Thatswhy
i dia=a and c=rt2a thn 1:rt2
394. x^4+1/x^4=194 than x^3+1/x^3 = ??
Ans- 52 (x^2+ 1/x^2 + 2 ) = 194 + 2 .. x^2+ 1/x^2 = 14..
x^2+ 1/x^2 + 2 = 14 + 2 .. ( x+1/x) = 4..
395. If sin@ +sin^2@ +sin^3@ =1
then Cos^6@-4cos^4@+8cos^2@=????????
1,,,,2,,,,,,,,3,,,,,,,,4
Ans- 4
yar bada typical sa sum hai.... sin@ +sin^2@ +sin^3@ =1 ===> sin@ +sin^3@ =1-sin^2@
==>sin@ +sin^3@ =cos^2@
sin@+sin@(1-cos^2@) = cos^2@
sin@(2-cos^2@) = cos^2@
sq both sides
Bhai yeh SSC ke exam me aaya hai either in FCI or in CGL 2012
(1-cos^2@)(2-cos^2@)^2 = cos^4@
cgl 2012

are yar isko expand kar lo ajayega


Yr ye ssc kaise kaise sums dene lag gayi h

396. TIER2-Area of the right angled triangle ABC is 6 units. Two of it's vertices are (-2, 2) and (1, -2). Which one of the following cannot be the third vertex?
(a) (-2, -6) (b) (1, -6) (c) (-2, 4) (d) (1, 2)
Ans- C
Triplet 3 4 5 and use distance formula .
Haa bro Waise hi kiya tha Ho gya tha..m ne graph draw kiya tha toh aur easy ho gya tha
397. x+1/x=1 than x^12+x^9+x^6+x^3+1
-1
-2
1
2
0
Ans- 1
ek funda woh hai ki.. if x + 1/x = 2.... then x^n + 1/x^n = 2
398. (1) if x^a.x^b.x^c=1 then the value of a^3+b^3+c^3 is Ans-9,abc,a+b+c,3abc
(2) x^2+3x+1=0 then the value of x^3+1/x^3 is
plz with some explanations....
Ans- 1-3abc ..............x^a*x^b*x^c=x^0 (ant thing has power 0 is 1)..........so X^a+b+c=
x^0.........a+b+c = 0 ..........now you can go ahead
means a+b+c=0 then its 3abc yes got it
2. 18 -----from first equation (x+1/x)= -3............now apply formula a^3+b^3
2nd) take x common. x(x+3+(1/x)) = 0. so x+1/x = -3. now cube both sides.

399.(rt.(1-sin 4x) + 1) / (rt.( 1 + sin 4x ) - 1)


Ans- rt(sin^2x+cos^2x-2sin2xcos2x)+1/
rt (sin^2x+cos^2x+2sin2xcos2x)-1
=sin2x-cos2x+1 / sin2x+cos2x-1
=2sinxcosx-1+2sin^x+1 / 2sinxcox+1-2sin^x-1
=2sinx(cosx+sinx)/ 2sinx(cosx-sinx)
=cosx+sinx / cos x-sinx
=1+tanx / 1-tanx
=tan(45+x)
rather i took (sin2x- cos2x) and (sin2x+ cos2x)
i got (sin 2x - cos 2x +1)/(sin2x+cos2x-1)
(2sinxcosx +2sin^2x)/(2sinxcosx-2sin^2x)
Ok fir se kr k dekhti hu
take common you vl get
(cos x+ sin x)/(cos x - sin x)
divide by cos x
(1+tan x)/(1-tanx)
=tan(45+x)
Thanks for solutions guys...actually i am writing all sums discussed here in note book ...so that i
wont forget their solutions.. :)
400. sec^2@+ tan^2@=7/12 then sec^4@-tan^4@ = ?
Ans- 7/13
DESI jugad ya detail???????
yeah triangle vl b 5,12,13
haan.........base is same in cos@ and sin@ =13 so 5, 12, 13
triplets yaad kar lo yahha bahut kam aane hai..baba ranchod das ne sahi kaha hai

401. x + 1/x=5 than 2x /(3x^2-5x+3) = ??


5
1/5
3
1/3
Ans- 1/5
bhai x^2 +1 = 5x
multiply by 3
put in the above eq
2x/10x = 1/5
(x^2 +1)/x = 5; x^2 +1=5x; multuply by 3; 3x^2 +3-15x = 0
so 3x^2 +3-5x = 10x....put in eqn and get the value...
402.Find min and max value of sin2x-x
Ans- minimum is 0 and maximum is root 3/2
Bouncer h
403.-Desi --X+1/X= -2 THAN X^32+1/X^47
Ans- 0
put x = -1
make quardtic equation x^2+2x+1=0
404. AB and CD are two parallel cords of a circle drawn on opposite sides of thier parallel
diameter if AB= 6 cm and CD = 8cm and radius is 5cm what is the distance between the cords
in cms
Ans- 7 hi hai bhai... (Y)
also got 7 but answer option given is 2,8,5,3 don't know answer
3+4=7 ??
bhai yaaad mat dilaao is ques ki..:( mein bhejungaa sab se pehle representation :)
jo ques hum nikal dete hai wohi galat kyu hote hain :)

405. Find DE : DB, if BC = 3 units, EB = 1 unit and DE is an angle bisector of CDB and DB is an
angle bisector of ADE.
Ans- We are asked to find DE/DB,DE is internal angle bisector of Triangle(DCB) whereas DB is
the
External angle bisector of Triangle(DCE).
So,
From an external angle bisector in Triangle(DCE) we have,
DC/DE = BC/BE..............i
From an internal angle bisector in Triangle(DCB) we have,
DC/DB = EC/EB .............ii
From i & ii
we get, DE/DB = 2/3.
:)

406. sin^2 5 +sin^2 6-------sin^2 85 = ??


Ans- 5 to 44....40 terms....and (sin45)^2.....adding 40.5
407. A regular polygon with n side has interior angle measuring 178*, What is the value of
180/n ?
Ans- waise logic is tht if interior angle is 178 then exterior angle would be 2 .........so no of side is
360/exterior angle = 180............. so 180/n = 1
n will be 90 in that case so 180/n=2
408. If x= 1 + rt 2 + rt 3. Find
2x^4 - 8x^3 - 5^x2 + 26x - 28
6 rt 6
0
3 rt 6
2 rt 6
Ans- 6rt6
(x-1)=rt2+rt3 sq both sides
U wil get x^2-2x=4+2rt6, sq again,
And add

409. Find the smallest number which when divided by 7 gives remainder 5 , when divided by 11
leaves
remainder 3 and when divided by 13 leaves remainder 9?And whats the next no. after the
smallest one?
Ans- first take any two condition 11b + 3 = 13c + 9
divisor of 13 + 9 = 9,22,35,48,61,74,87,100,113,126,139
divisor of 11 + 3 - 3,14,25,36,47,58,69,80,91,102,113
so smallest number is 113
whats the next number then ?
its of form LCM(11,13) + 113 = 143k + 113
so we have combined two conditions
so now our job is to compare this with third one
143k + 113 = 7a + 5
143k + 108 = 7a
140+ 3k + 105 + 3 = 7a
so 3k + 3 should give 0 remainder when divided by 7
so k = 6
final remainder is hence 143(6) + 113 = 971
410. Ok guys easy one
find the sqrt of 3+rt5
Ans- Its a sqrt so y a square? Its simply (rt5+1)/rt2
Multiply by 2 in num. and denom both.num wil a 6+2rt5, it can b written as (rt5)^2+1^2+2rt5
=(rt5+1)^2 nw take d sqrt
ans shud be( root10 + root2 )/2
411. If cos(A-B) =3 cos (A+B) then,
cotAcotB is..?
Ans- Open d bracket ,take the cos terms to one side n use componendo,dividendo
Ha its a/b =a+b/a-b
bhai seedha formulae khol do ho jayega . kuch karne ko hia hi nahi isme

412. Area of the trapezium formed by x axis; y axis and the lines 3x + 4y=12 and 6x + 8y=60 is?
I know ans is 31.5 , but hav no idea abt how to solve it.
Ans- simple...first write the equation in the form x/a + y/ b =11 form..
so.. x/4 + y/ 3 = 1 and x/10 + y/ 7.5 =1...
so for 1 line x in tercept = 4 and y = 3, for 2nd , x = 10 and y = 7.5...
now you got 2 right triangles ...1 with base 4 and height 3....other with base 10 and height
7.5...find the area of individual triangles and substract...u will get ur answer..
can simply do it in mind....if u clear the concept....have to write the equation in x/a + y/b = 1
form...then a is the point where the line touches x axis and b the point where it touches y axis..

413. if 3x+5y=15. find maximum value of x^2*y^3


Ans- 3x/2 + 3x/2 + 5y/3 + 5y/3 + 5y/3 = 15.--------------1;when sum of any quantities is constant,
there product is maximum when they are equal.
here sum is constant.
so when 3x/2 = 5y/3. we get maximum value of x^2*y^3.
taking 3x/2 = 5y/3 putting it in 1,
=> 5(3x/2) = 15.
=>x=2. and y = 9/5.
answer is 2^2*(9/5)^3.
414. (1+sin 2A-cos 2A)/(1+sin 2A+cos 2A) is..?
Options
tan A
cot A
1
sec A.
Ans- put a 45 n check option
1+tan A / 1- tan A
By putting A=45 we will get ans=1, now CotA & TanA are both =1.... So how o decide betwn
them??
You are right..
change both 1 in to sin^2 +cos^2
and expand sin2A and cos2A

415. simple one


x=ycos(2pi/3)=zcos(4pi/3) then xy+yz+zx is..?
Ans- 0
-2x^2+4x^2-2x^2 =0

416. x+1/x=2cosq thn x^3+1/x^3


Ans- yes 2cos3q is right
417.1) tan 20+tan 40+root3. tan 20. tan 40 equal to
a)rt3/2 b)rt2/4 c)rt3 d)1
2)cosec A+cot A=11/2, then tan A equal to
a)21/2 b)15/16 c)44/117 d)117/43
3)if tan A=1/2 and tan B=1/3, then value of A+B equal to
a)pi/6 b) pi c)0 d)pi/4
4)if sinA=3/5, 02 and cos B=-1213, pi<3pi/2, then sin(A-B) equals
a)-16/65 b)16/65 c)65/16 d)-65/16
Ans- 1. rt3
2. 44/117
3. pi/4
4. what is , 02?
1.d
2.
3.d
4.a
what is 02 in the ques?
bro i just copy pasted the questions from our pgforum dont know wat it is
last one is -16/65 since sin B will be positive
418. 7sin^2q+3cos^2q=4 whre q is acute angle than tanq will be
Ans- 1/rt3
i think these are the sample of the question trigo that ssc can ask ..........so please post these
kind of question as many as possible
1/rt..aata chaki pe time lag gya tha.

419. (1+Sec20+Cot70)(1-Cosec20+tan70)=????
options are ,,,,,,,,,,, -1,,,,,,,, -2,,,,,,,,2,,,,,,,0
Ans- 2
yes solution please?

420. m+ 1/m-2 =4 than (m-2)^2+1/(m-2)^2


Ans- 2
Substract 2 frm both sides and apply a^2+b^2
421. If sec A - tan A = a, then sin A is..?
Ans- (a^2 -1)/(a^2+1)
secA-tanA = a. Rationalize. u will get secA + tanA = 1/a. solve these 2 equations to get secA.
secA = (a^2+1)/2a. so we have hypotenuse and one side. calculate other side to get a^2-1. so
we have three sides. so get sin A

422. (1+cota-coseca)(1+tana+seca)=???
Ans- 2
Take a=45

423. The perimetre of an iosceles right angled triangle is 2p.Find out the area of the same
triangle.
a)(2+rt2)p^2 b)(2-rt2)p^2 c)(3-rt2)p^2 d)(4-2rt2)p^2 e)(3-2rt2)p^2
Ans- x + x + rt 2 x = 2P...
x ( 2 + rt 2 ) = 2P
x = 2 P / ( 2 + rt 2 )
area = 1/2 *x *x = 1/2 * 4 P^2 / ( 2 + rt 2 ) ^2
= 2 p ^2 / ( 6 + 4 rt 2 ) = p ^2 / ( 3+ 2 rt 2 ) = ( 3 - 2 rt 2 ) P^2
424. (243)^n/5 x 3^(2n + 1)/(9^n x 3^(n - 1) =?
Ans- 9

425. if tan@-cot@=a and cos@-sin@=b


then find d value of (a^2+4)*(b^2-1)^2
1,2,3,4
Ans- Simply square both a and b and put in d eqn
by putting @= 0 or pie we get the ans 4+8-8=4,,,,,,, but is there any logic
At d end u wil get 4(sin^2@+cos^@)^2
=4, bt desi method also very gud,time saving
426. sin (45 + ) cos (45 ) is equal to
this ques was in my set copied from NCERT buk...coudnt attempt such easy ques bcoz of time
... NCERT Books r best
Ans- Cos (45-)= Sin(90-(45-)) = sin(45+) we get 0
427. sinx = 1/2 then tan2x=?
a.2/rt3 b.1/rt3 c.rt3/2 d.rt3
Ans- By using pythagoras th,,, we get the triplet as (rt3,1,2) , tan x=1/rt3 Tan 2x= tan(x+x) =
(tanx+tanx)/1-tanx*tanx ,,, put the value of tanx we have rt3,,,,,,,,,
In simple way we have sinx=1/2 so x=30 , tan2x= tan60=rt3
428. sin^2 1+sin^2 5+sin^2 9--------sin^2 89 = ??
Ans- 11
429. CGL2012 problem
If length of a chord of a circle, which makes an angle 45* with the tangent drawn at one end of
the chord, is 6cm, then the radius of the circle
1-5
2-3rt2
3-6
4-6rt2
Ans- 3rt2 use trigno metry
430. sin105*sin75=??
sin(A+B) ka formula laga do
(2+rt3)/4
sin105 = cos15, sin75 = cos15 then given expression = cos^2(15) i.e. [cos(45-30)]^2 m i right?

431. Value of sin(x+30) +cos(x+30) is max when x=?


Ans- 15
Ok got it...by converting cos(x+30) to -sin(x-60) & applying sinA-sinB formula...bu any short cut
for it??

kya yaha...geo n trig k q hi post krne h kya...eng b h...n algebra b h....


432. vallue of 1/rt2(cosA-sinA)
Ans- ans is cos(pi/4+A)
Take 1/rt2 as sin45 and cos45 put in d eqn and aply sin(a+b) formula
433. tanq-cotq=a and cos+sinq=b than (a^2+4)(b^2-1)^2 = ??
1
2
3
4
Ans- 4 easy one ...let q=45
434. 3sin20-4sin^3 20 = ?
Ans- Sin3A ka formula h....sin3X20.....sin60...rt3/2
Ye formula hai sin3x=3sinx-4sin^x
yaha x=20

435. sinq+cosecq=2 than sin^43q+coseqc^43q= ?


Ans- 2
Q =90*
always 2 for any power
(sinq - 1)^2=0
shrt cuts
sinq*sin2q*sin4q=1/4*sin3q
cosq*cos2q*cos4q=1/4*cos3q
tanq*tan2q*tan4q=tan3q

436. If tan @ = (xsin$)/(1-xcos$)


tan $ = (ysin$)/(1-ycos@)), then x/y is..?
Ans- sin@/sin $
Solution=??
Change the equation into the forms
x=
y=
change tan into sin/cos.
will get the ans.
find x/y
437. Cos(x+y)=12/13 , sin(x-y)=3/5 find sin2x
Ans- 56/65
sin{(x+y)+(x-y)} =sin(a+b
438. ratio of side of 2 regular polygon is 5:6 and their internal angles is 24:25 than no. of sides
of these two polygons is _____
Ans- 10 and 12
let the sides be 5x and 6x hence 5x-2/5x:6x-2/6x=24/25 solve u will get x=2
I 1 /I 2 = (n1 -2 )* n2 / n2 -2 * n1 here n1=5x n2 = 6x and I1/I2 = 24/25 given

439. 2^a+3^b=17 and 2^(a+2) -3^(b+1) =5 then find a and b


Ans- a =3 , b= 2
Yes, seperate 2^a amd 3^b term in second eqn,assume 2^a as x and 3^b as y,solve 4 x and y

440. 1/{1 + x(b - a) + x(c - a)} + 1/{1 + x(a - b) + x(c - b)} +1/{1 + x(b - c) + x(a - c)}= ?
Ans -1
1/(x^b/x^a + x^c/x^a)+ 1/(x^a/x^b+ x^c/x^b)+1/(x^b/x^c + x^a/x^c) solve this
"1/(x^b/x^a + x^c/x^a)+ 1/(x^a/x^b+ x^c/x^b)+1/(x^b/x^c + x^a/x^c) solve this"
bro there is little mistake in Q ,,,,, as u not mentioned there x^(b-a) in all but u written x(b-a)
correct bro typo mis..:(
most of all these types of question ans will be 1

441. ABC is a right angled triangle. AD is perpendicular to the hypotenuse BC. If AC=2AB, then
the value of BD is?
A. BC/2
B. BC/3
C. BC/4
D. BC/5
Ans- D
jst draw the diagram and find the length of AD
now take the similar sides in trngle ABC and ABD
442. if MN is 10 meters in length and is tangent to one of the concentric circles at point A. If it is
given that the raddi of the both the circles are integers, then what is the radius of the inner
circle.
ans is 12 ,,,,,, plz explain Kindly read P=A

Ans- AN=5, ON=13 So radius of inner circle is 12, no othr integer value possible
MN always be greater than inner radius in this case
Y shud MN b greatr than inner radius?
ohh sryyyyy actuaallly mt fault i made the diagram wrng and it appears like that
443.1. if A = 75 degree, B = 45 degree, then b + c rt 2 = ?
1. a, a+b+c, 2a, none
Ans- Draw it..
Use cosine rule.
b^2 = a^2+c^2-rt2 ac
c^2 = a^2+b^2-ab
adding these equations will give
b+rtc = 2a.

444. Two ships are sailing in the sea on the two sides of a lighthouse. The angle of elevation of
the top of the lighthouse is observed from the ships are 30 and 45 respectively. If the
lighthouse is 100 m high, the distance between the two ships is:
Ans- tan45= 100/x
X=100
Tan30 = 100/y
1/root3=100/y
Y= 100root3
Distance=x+y
=100(1+root3 ) or 273

445. Three utensils contain equal mixtures of milk and water in the ratio 6 : 1, 5 :
2, and 3 : 1 respectively. If all the solutions are mixed together, find the ratio
of milk and water in the final mixture.
(a)65:19 (b)25:13
(c)35:18 (d) None of these
ANs- 65:19
446. Mr A drives to work at an average speed of 48 km per hour . The time taken to cover the
first 60 % of the distance is 10 minutes more than the time taken to cover the remaning distance
. How far is his office ?
A. 30 km
B. 40 km
C. 45 km
D. 48 km
Ans- when speed is constant then time is proportainal to the covered distance so 60% : 40% =
6:4 is the ratio of the time taken .....now you know the difference of the time ie. 10min so total
time =50min......distance=40
.6x/48=.4x/48+1/6=x=40

x/5 =1/6 *48 i.e x =40


time : 60:40 i.e, 3:2
3x-2x = 10
X= 10
time = 5x = 50
d= s*t =48*50/60
= 8*5 = 40.....

447. A lamp lighter has to light 100 gas lamps. To go from one lamp post to the
next he takes 60 seconds. Each lamp burns 12 cubic feet of gas per hour. If
he lights the first lamp at 7 p.m. then the gas burnt when he lights the last
lamp is
(a) 1110 ft3 (b) 1000 ft3 (c) 999 ft3 (d) 990 ft3
Ans- 990
U r always right.

448. A circle is inscribed in a triangle wid sides 8,15,17 find the radius of circle
Ans- r = area of triangle/s
Area of triangle = 1/2*8*15 = 60
S = 8+15+17/2 = 20
R = 60/20 = 3.
449. The value of 2( sin ^6A + cos ^6A)- 3(sin^4A+ cos^4A) +1..?
Ans- 0 put A=0
i dont think we can put A=0 in this question, can we ?......pls correct me
@shashi u can also take A=90, it wil b applicable to any value of A
2((sin^2a+cos^2a)^3-3sin^2acos^2a)) - 3((sin^2a+cos^2a)^2-2sin^2acos^2a)) +1
=2(1-3sin^2acos^a)-3(1-2sin^2acos^2a)+1
=0
450. One litre of water is evaporated from 6 litres of solution containing 5% salt.
The percentage of salt in the remaining solutions is
(a) 5% (b) 6% (c) 1% (d) 25%
Ans- 6%
bro.. you deserveto be ssccgl topper ..:)
bhai kyu majak kr rhe ho?
dekh rha hu yaar pg pe.. yaha pe apko kab se..tehelka macha rakha h..:)
hehe tehelka :)

451. sin ^2( 15+ A) - sin^2 ( 15- A) is equal to..?


a) (cos 2A) /2
b) (sin 2A) /2
c) (tan 2A) /2
d) cot 2 A
Ans- 1/2-cos(30+2a) /2 - 1/2+cos(30-2a)/2
cos(30-2a)-cos(30+2a) /2
2sin(30+2a+30-2a/2) sin (30+2a-30+2a/2) / 2
2sin30sin2a /2 = sin2a/2
put A = 15.. can also do
also do by
aSQR - bSQR equal to (a -b)(a plus b) formula
sin2a/2

452. Tan20*Tan35*Tan45*Tan55*Tan70 = ?
options: 1 , 2 ,0 , none
Ans-1
453 if a man cycles at 10km/h, then he arrives at a certain place at 1 p.m. If he cycles at
15km/h, he will arrive at the same place at 11a.m. At what speed he must cycles to get there at
noon?
Ans- 2xy/x+y = 2*15*10/25=12
Oral method Lcm of 15 , 10 is 30 ..so distance should be 30 or its multiple ..check the values 60
is satisfying so speed is 12..
454. if x^2+y^2+z^2=xy+yz+zx,(x is not equal to zero)then the value of 4x+2y-3z/2x is
Ans- 3/2 X=y=z=1
@rssj x=y=z=1 kaise aaya??
455. if x^2+y^2+z^2+2=2(y-x),then value of x^3+y^3+z^3 is equal to
Ans- 0.. hit and trial
Sare term ko ek side le jao, it will b (x+1)^2 +(y-1)^2 +z^2 =0, thus x=-1 y=1 n Z=0
456. 2tan70 - tan 80 + tan 10 is..?
Ans- 0
Yes..
tan (80) = (tan 70 + tan 10)/( 1- tan70tan10)

457. Find the minimum value of Root x^2 +y^2 if 5x + 12y= 60


Ans- root ( x^2+y^2 ) .... differenciate... 1/2 (x^2+y^2).(2x+2y)=0
x=-y.... 5x+12y=60... or y=60/7,,x=-60/7... if put in rt(x^2+y^2)
ans shld be rt2.60/7.
d(5x+12y)/dx = 0, dy/dx = -5/12, let root (x^2+y^2) = z ,d(root (x^2+y^2))/dx = 0.......(2x +
2dy/dx)/2(root (x^2+y^2)) = 0................get x = 300/169 put in eqn get y = 720/169.....put in root
(x^2+y^2) get z = 60/13
Dosto derivatives bhul gaya hun..koi normal way se hi batao

One more request guys....


When you post a quesion...if possible plz give options also...kabhi kabhi sirf options dekhke bhi
S.C. Nikala ja sakta hai..
Jst wanna thank all those guys who put trigno n geom quesn here.these r most covered topic in
ssc,n what we lack in is good books.so its good dat we discus these here,so dat we hv collectn
n practice of good quesn

458. The value of


(sin 38 - cos 68)/(cos 68 + sin 38) is..?
Ans- make it as sin38-sin22/sin38+sin22..then using formula u will get
2cos30sin8/2sin30cos8
=rt3tan8
459. Easy h 1-tana/1+tana=?
Ans- cot(a+45) or tan(45-a)

460. Tan(-405degree) =?
Ans- tan(-theat)= -tantheta..so ans will be -1
461. 2! + 4! + 6! +8! +.............100! when divided by 3, would leave reminder?
1-0
2-1
3-2
4-3
Ans- 4! =4 * 3 * 2.. so divisbile by 3.. 6 ! = 6 * 5* 4 * 3 *2... so every no. from whose factorial is
considered will be divisible by 3.
except for 2! whic h 2 * 1.. so no multiple of 3 in it

462. A dealer sold three-fourth of his articles at a gain of 20% and the remaining at C.P. he gain
earned by him in the whole transaction is :
a. 25%
b. 20%
c. 15%
d. 10%
Ans- 15%
6/5*75= 90+25 = 115
115-100 = 15%
463. Square root of
(x^5-1/x-1)+(x^3+2x^2+x)
x^2+x+1
x^2-x+1
x^2+x-1
x^2-x-1
Ans- (x^5-1)/(x-1)+x^3+2x^2+x
=(x^5-1+x^4+2x^3+x^2-x^3-2x^2-x)/(x-1)
=(x^5+x^4+x^3-x^2-x-1)/x-1
={x^3(x^2+x+1)-1(x^2+x+1)}/(x-1)
=(x^2+x+1)(x^3-1)/(x-1)
=(x^2+x+1)^2
square root=x^2+x+1
DESI put x=2....a
464. Find d min value of sin^3x+cos^3x
diff will give sin^2x=cos^x
tan^2x=1
tanx=+/- 1
x=45 or 135
it's better to do these type of ques with options...and vaise bhi min value of sin@ or cos@ is -1
and so other value will be 0...to -1 to aayega hi...kam chance hoga ki ques. mein -2, -3 etc bhi
diya hua ho...

465. If the length of the smallest altitude of a triangle is 6 cm, then the inradius is
greater than 3 cm
less than 3 cm
3 cm
non of these
Ans- less than 3 cm

466. ABCD is a rectangle and ABE is a triangle whose vertex E lies on CD. If AB = 5 cm and the
area of the triangle is 10 sq. cm, then the perimeter of the rectangle is:
a. 20 cm
b. 18 cm
c. 15 cm
d. 14 cm
Ans- 9 plus9 18
467. O is orthocentre of ABC.IF Angle BOC=120,THEN angle BAC=?
Ans- 60

468. PQRS is a rectangle inscribed in a quadrant of a circle of radius 13 cm. A is any point on
PQ. If PS = 5 cm, then area of triangle RAS ?
Ans- 30
469. x^2/(by+cz)= y^2/(cz+ax)= z^2/(ax+by)=1
then...a/(a+x) + b/(b+y) + c/(c+z) is......plz explain it..too
Ans- 1
a=b=c=1
x=y=z=2

470. If A = sin 45 + cos 45 and B = sin 44 + cos 44, then


a. A> B, b. A<B c. A=B d. None
Ans- hmm.... A>B he hoga.. agar dono k square karein pata chalta hai
yes a bt its cat level ques,a is answer

471. A light was seen at intervals of 13 seconds. It was seen for the first time at 1 hr 54 min 50
secs. a. m. and the last time at 3 hrs 17 min 49 secs. a.m. How many times was light seen ?
a. 382
b. 384
c. 383
d. none of these
Ans- 3*3600+17*60+49 - 1*3600+54*60+50
11869-6890=4979/13+1=384

472. If sin X + cosec X = 2, then sin ^n x + cosec ^ n x = ?


a. 2^n, b. 2 ^ ( n-1) c. 2^ ( n-2) d. none
Ans- d.
Take al d terms in sin form epn wil b sin^2x-2sinx+1
=> (sinx-1)^2=0
=sinx=1
put the val of sinx in gvn eqn n u wil get 2
x = 90
473.A cow is tied with a 50m rope to a corner of a 20m by 30m rectangular field.the field is
completely fenced and the cow can graze on the outside only.what area of land can the cow
graze?
Ans- 2200pi?

474. y+1/z=1
x+1/y=1
Find the value of xyz?
Ans- y=1-1/z
X+1/y=1, or xy+1=y
Substituting value of y, xy+1=1-1/z, solving, xyz+z=z-1 or xyz= -1
take x =1/2, y=2 , z=-1.............

475. Three circles are drawn touching each other, their centers lying on a straight line. The line
PT is 16 units long and is tangent to the two smaller circles, with points P and T lying on the
larger circle. The area inside the largest circle but outside the smaller two circles is equal to
(a) 4
(b) 8
(c) 16
(d) 32
Ans- bhai CAT ke questions mat rakho..hamari maths waise hi itni weak h in questions se
demoralize aur ho jayenge..:(
476. 3 circle each of radius 1cm are touching each other.AB is the line passing through the
centre of three circle,with A lying on one of the outermost circle and B being the centre of the
other outermost circle.AC is tangent to the circle with centre B and cut chord DE on the middle
circle.find the length of DE.
Ans- sir mai bhi fig banane ki kosis me kab se laga hun..ban hi nahi reha :( i got this question
from other community
when three circle will touch each other then it will be like a equileteral triangle........ab isme
outermost or inner most circle kaha se aayenge .........so something is missing ........leave it
477. Find the minimum value of sin ^2 x - 3 sin x + 2 ?
Ans- by putting sinx = 1 ..........we get 0 as min value

478. Minimum value Sin^n x + Cos^n x


When n=2. 4. 6. And. 3??
P.S. I don't know the ans :P
Ans- If n=1 min is- rt2
if n=2,min/max is 1
if n=6 min is
For n=4, min is
bhai ques gadbad lag raha hai... agar n = 2 to value 1 hi hogi hamesha

thanks for undestnding the point I wantd to raise thru this Q and replying as I wntd :D Ricky bhai
I creatd this Q myself so there is a possibility that I did a or few mistakes :) thanks anyway.
479. values of a and b which makes 8X^3 - aX^2 + 54x + b perfect cube ?
Ans- 8X^3 - aX^2+ 54x + b
in question it is -a
so a is 36
(A-B)^3 is formula me hai...which is a cube n dis is satisfied if a is 36 n b is -27
480. The greatest value of sin ^4 x + cos ^4 x is ?
1,2,3,4
Ans- Sin^nx +Cos^nx where n is odd or even the ans is always maaximum value is 1 only
exception is power 1 where rt2 is max value
It is 1-2sin^2A cos^2A
1-2sin^2A+2sin^4A
when we solve it we will get 1-1/2(sin^2(2x))... minimm will when sin term will b max.. which will
be 1 hence min will be
yaar min ke liye shayad............ sin^nx+cos^nx whre n is even then put,,x= 45 it is always min.
value and for n=odd then -1 is always minmimum value
481. Min value of (1/2*sinx)+4*cosx+3 is......
Ans- M getng (6-rt65)/2

482. equa. of lines is


x cos 60+y sin 60=6
then find the lenght of perpen. frm point (6,2) to the line
Ans- Mod(rt3-3)
3-root3

483. a^m* a^n * = a^ mn, then m ( n-2) + n ( m-2) = ?


a. 0 b. 1. c.1/2 d. -1
Ans- 0
explain desi please any of the above who answer the question.
put m and n =2
a=m=n=2
Or even u can put a=b=0
484.If sin x + sin y = 3 ( cos y - cos x), then sin 3x + sin 3y
Ans- 0.put x=0 n y=0
Sin ki value 0 ho gayi at x and y =0 and cosx=1 and cos y=1 so yeh bhi 0
Desi rocks ,ssc shocks!
:p
485. 5^6 - 1 is divisible by
a) 13 b) 31 c) 5 d) none of these
Ans- a^2-b^2 se ho jayega.....125^2-1^2...124 is divisible by 31...so 31 is the ans...
bhai i think sabse badda option 31 hai to hume a ki wo value leni hai jo 31 se baddi ho takki
remainder 31 possible ho sakke...
5^6 = (5^3)^2 /31 = (-1)^2 = 1 so 1-1 =0

486. Two lines, 3x 4y + 8 = 0 and 8y 6x 1 = 0, are tangent to a circle. What is the area of
the circle?
options :
25pi/16 square unit
16pi/9 square unit
9pi/4 square unit
9pi/16 square unit
not a unique value
Ans- this que is beyond ssc level but doable in one min..both lines are parallel and tangent to a
circle..so now take any point in one line which satisfies it and then find the distance b/w 2
lines..and then r=d/2..
The formula to find the distance from a point (x0,y0) to a line Ax+By+C=0 is
d = | A(x0) + B(y0)+C| / sqrt(A^2 + B^2)
step 1) Make sure these are 2 paralle lines.
3x - 4Y + 4 = 0 <-- (1)
3x - 4y - 7/2 = 0 <-- (2)
step 2) pick a point on one of the lines.
From line (1), we pick the point (0,1). This point satisfies line (1) equation.
step 3) Use the formula to find the distance from (0,1) to line (2),
d = | 3(0) - 4(1) - 7/2| / sqrt(3^2 + 4^2)
d = 3/2
D..hi..h..r=3\4..h...so..area..is..9\16pie

487. If sin(pie.cos@)=cos(pie.sin@),then sin2@=?


Ans- sin(pi.cosA) = sin(90-pi.cosA)
pi(sinA+cosA)=90
sinA+cosA=1/2
By squaring
sin2A =-3/4
488. If A+B=225 find d value of cotA/(1+CotA)*CotB/(1+cotB)
Ans- cot(A+B) ka formulae lagao...ho jayega...ans is
we can use cot(a+b)=cot225=1=(cota* cotb-1)/(cota+cotb) it provide the value for denom...of the
question.

489. If cos@+sin@=rt2cos@ find the value of @


Frm eqn tan@=rt2-1 calculate tan2@=2tan@/(1-tan^2@) it wil gv tan2@=1
it means 2@=45
=> @= 22.5
- 22.5 aa raha hai bhai.....apka positive kaise aa raha hai
490. sin@+cos@=rt3.cos@
cos@-sin@=?
Ans- simpl h......... sin@+cos@=rt3cos@ sin@=rt3cos@-cos@ now sin@-cos@=rt3cos@cos@-cos@ so ans. sin@-cos@=rt3cos@-2cos@ so ans. =(rt3-2)cos@
bhai simple sa hai...divide the first equation by cos@....tan@=rt3-1...now let cos@sin@=k...again divide the equation by cos@...k=(2-rt3)cos@....30 sec bhi nhi lagenge isme....:-)
491. if 2-cos^2x=3sinx.cosx and sinx is not equal to cosx, then tanx=?
(a)1/2 (b)0 (c)2/3 (d)1/3
any shrtcut we hv ??
Ans- tanx=1\2..or..1..so..here.is..1\2
No short cut. Its lengthy..
Not too lengthy .........simply cos^2x se both side divide karo .......then tanx ka quadratic equ
solve karo.....
492.Speed of a boat in still water is 5km/hr.it crosses a river 1km wide, via min. possible path in
15 min.find the speed of stream
Ans- Let speed of stream is x.if OA is perpendclr on BA thn velocity in directn of OA =Rt(5^2x^2)=rt(25-x^2)
OA=1km so eqn is
15/60=1/rt(25-x^2)
it give x=3km/hr
i took 5-3=2
493. If the radii of the ends of a bucket 45cm high are 28cm and 7cm, determine its capacity.
a. 48510 cc
b. 48710 cc
c. 49510 cc
d. 49710 cc
Ans- a using formula (pi *h /3 [R^2+r^2+R*r
Use the formula 4 vol. Of frustum, Calculatn is lengthy here

494.If tan^2A=2Tan^2B+1 find cos2A+sin^2B


Ans- 0
put A = 45 and B = 0
all r gud questions....its actually nt easy to apply desi....difficult to view bt once executed ans
comes within secs
495.Sin@ + cos@ = a
Sin^3@ + cos^3@=b
Then (3a-2b)= ?
Ans- Sin^3@ + cos^3@=(sin@+cos@)(sin^2@+cos^2@-2sin@cos@)==>b=a*(1sin2@/2)..and from first eq by taking sq we get 1+sin2@=a^2...nw put d values u will get ans
a^3
find the value of Sin2@= (a^2-1) the put up
3a=3sin@+3cos@....2b=2sin@(1-cos^2@)+2cos@(1-sin^2@)....now do 3a-2b....ans is a^3
496. if A=sin^2+cos^4
then value of A?
Ans- sin@-cos@..
cos^2A+sin^4A
1- (sin 2@^2/4)
497. LCM of 4v2, 3v2, 5v3 is. V stands for root.
A. 60v2
B. 60v3
C. V6
D. None
Ans- option D.
factorise each term nd take heighest of factor in each..
eg..for 42..factor are 2^2*2...for 32..its 3*2...for 53..its 5*3..multiply all the
factor...eliminate multiple factors by comparing similar factors in all three terms nd take heighest
powered factor...
so LCM wud b..2^2*3*5*2*3=606

498. In d xy-coordinate systm if (a,b) n (a+3,b+k) are 2 points on the same line defined by d
eqn x=3y-7, k=?
Xplain, how to solve
Ans- a = 3b - 7 n a+3 = 3 (b+k) -7..solve kar answer laa..!
agar koi bhi point line par lie karte h..then they will satisty the line Equation..aankh band karke
value put kardo..!
k=1 make two equation bu putting the two coordinates in given equation in the form of a and b

499. hey..how to solve it..


tan2@.tan4@= 1 ...then tan3@= ?
Ans- 1
jugadu system take @=15 degree
any genuine method to clean the concept
change tan in sin cos then get it
500. if x= [rt(3) +1]/2
find 4x^3+2x^2-8x+7
Ans- DESI BOYZZZZZ
Put =1.37 ~1.3...
Ans 9+3.4+7-10.4= ~ 10....
If options r very close then only it ll create some problem
2x = rt3+1...4x^2= 4+2rt3....4x^3+2x^2-8x+7 isme x common lo, 7 ko alag rakhna and baaki
values put kardo....10
501. What is the number of total triangles with integer valued sides and perimeter 14 c.m. ?
Ans- sum of the two sides must be greater than third sides is this the concept here applied.
(6,6,2) (6,5,3),(6,4,4)............iske alawa kya possible hoga
(5,5,4) baith rha h kya
haan bahi
502. (sin70+cos40)/(cos70+sin40)
Ans- root 3
simply apply sinc(C+D)/ Cos (C+D)

503. Consider a sequence of seven consecutive integers. The average of the first five integers
is n. the average of all the seven integers is
a) n b) n+1 c) k *n whee k is function of n d) n + (2/7)
Ans- n+1 i.e 7n+7/7
504. what is the unit digit in 22^51
a) 2 b) 8 c) 1 d) 4
Ans- 8, sol using power cycle of numbers
2,4,8,6,2....and so on...8 aayega...
505. Let N= 1421 * 1423 * 1425 . waht is the remainder when N is divided by 12 ?
a) 0 b) 9 c) 3 d) 6
Ans- 5*7*9/12 then reminder 3
1421 / 12 gives a remainder 5, similarly 1423 , 7 and 1425 , 9...
when the whole number is divided by 12, divide the individual remainder product by 12....
5*7*9 / 12 /......gives a remainder 3
506. 6 men and 8 women can do a piece of work in 10 days. In how many days can 5 men and
9 women do the job?
a. 240/47 b. 11.5 c. 12 2/3 d . none
Ans-6 men and 8 women ..............Only word and made difference..........IF it is OR then soln
possibl
A..answer le aya m jaise Taise shuru me toh can't determine hi lag rha tha..:D
507. A loan was repaid in two annual instalments of Rs. 121 each. If the rate of interest be 10%
per annum, compounded annually, the sum borrowed was:
a. Rs. 200
b. Rs. 210
c. Rs. 217.80
d. Rs. 216
Ans- P=(121*10/11)+(121*10^2/11^2)
=210
if you cant remember above formula than............think this problem as banker is earning interest
on rs 121 for one year ie. 12.10 ....so total amount banker recieved 121 + 121 + 12.10 = 254.10
so this is the amount A nw find P = 254.10 * 100/121 = 210 instalment formula ye bhe use kr
skte h amount = m(1+r/100)^t-1 + m(1+r/100)^t-2.....+m where m is emi

508. Let N=55^3 + 17^3 - 72^3. N is divisible by a) both 7 and 13 b) both 3 and 13 c) both 17
and 7 d) both 3 and 17
Ans- 55^3 + 17^3 - 72^3 = (54+1)^3 + ( 18 - 1 ) ^3 - 72 ^3 = ( 51+4)^3 + 17^3 - (68+4)^3 . these
2 forms are divisible by 3 and 1 both.
3*55*17*72
So ans is d
509. On a Rs 10000 payment order, a person has a choice between 3 successive discounts of
10%, 10% and 30% and 3 successive discounts of 40%, 5% and 5%. By choosing the better
offer, he can save :
a. Rs 200
b. Rs 255
c. Rs 400
d. Rs 433
Ans- 255
fir apna desi batao
naa koi desi nahi ..X+Y-XY/100 lagao do dbaar..yaa calc. strong hai toh 100 assume karke karo
90---81---56.7
510. the no of positive integers not greater than 100 which are not divisible by 2,3 or 5
a) 26 b) 18 c) 31 d) none of these
Ans-26
Set theory based
nos. divisible by 2 + nos. divisible by 3 + nos. divisible by 5 - nos. divisible by 2 & 3 - nos.
divisible by 2 & 5 - nos. divisible by 3 Nd 5 + nos. divisible by 2 , 3 & 5.
511. If three positive real numbers x,y,z satisfy y-x=z-y and xyz=4, then what is the minimum
possible value of y ?
Ans-(2)^2/3
correct take x=y=z
x = y = z only in this case as x, y ,z are in AP
512. The difference between the interest received by A and B is 18 on Rs. 1500 for 3 year.
What is the difference in rate of interest?
a. 1 b. 2.5 c. 0.5 d. 0.4
Ans- d

513.Let ABCDEF is a regular hexagon. what is the ratio of the area of triangle ACE to that of the
hexagon ABCDEF?
Ans-
diagram draw karne se easily aa jata hai..bt whats accrdng to formula?
na^2/4cotpi/n : ???
514. Dale Steyn whose bowling average is 12.4, takes 5 wickets for 26 runs and thereby
decreases his average by 0.4. The number of wickets, taken by him before his last match is :
a. 85
b. 80
c. 72
d. 75
Ans- 85s
12.4Y+26/Y+5=12...where Y is no of wkts before last match
515. What is the mode of the following data of wages per day ?
5, 4, 7, 5, 8, 8, 8, 5, 8, 9, 5, 7, 9, 10, 8
a. 4
b. 5
c. 7
d. 8
e. 10
Ans-8
516. A group of students decided to collect as many paise from each member of the group as is
the number of members. If the total collection amounts to Rs. 5929, the number of members in
the group is :
a. 67
b. 77
c. 87
d. 57
Ans-77
naa nahi ayegaa..... trick dekho sq root nikalan bhi nahi padegaa...go by options naa galat
hogaa na phassoge..5-6 sec
517. What is the sum of all two-digit numbers that give a remainder of 3 when they are divided
by 7?
Ans- 676

518. The total monthly sales of two companies A and B are in the ratio 2:3 and their total
monthly expenditure are in the ratio 3:4. Find the ratio of the profits of the two companies given
that company A's profit is equal to one fifth of its sales.
A) 6:13
B) 5:12
C) 12:5
D) 7:14
E) 14:7
Ans- 6:13
mera lengthy hai....2x, 3x sales. 3y,4y expenditure. 2x-3y = 1/5 * 2x. so 8x=15y. ratio asked:
(2x-3y)/(3x-4y)....substitute x = 15/8*y. ans a ayega. took 2-3 min. koi short method hai to batao.
sale: 2x and 3x
expendi: 3y and 4y
profit: 2x-3y and 3x- 4y
now A's profit is 2x-3y = 2x/5...
find ration of x and y and put for profit ratio...
i think allegation se bhi hosakta hai... kissi ko aato ho alligation se to pls explain
519. selling at rs21 man suffers a loss %=cost price,....what is cost price
Ans- 70,30
simple SP ka formula use karo ......... x*(100-x) / 100 = 21 .........quad eq .........x^2- 100x + 2100
= 0 , or x^2 -70x-30x+2100=0 ....solve
520. Two sides of a trngl r 4cm n 10cm. If d lnth of the 3rd side is 'a'cm,thn
a) a <6 b)a>5 c)6<=a<=12 d)6<a<14
process?
Ans-d
sum of 2 sides always greater than 3rd
laga aur answer..!
And diff of two sides shud be smaller than the third side
521. The base diameter of a right circular cylinder is 3 cm. there is a section making angle of
30* with the cross section . what is its area
9pi/4 9pi/8 3rt3pi/2 9rt3pi/8 sq cm
Ans- i did, cosec30 degree = h/1.5, then area by usual formula

522.Two sides of triangle are of length 15 &7 cm.if the length of 3rd side is an integer what is
the sum of all the possible length of third side
Ans- 195
15+7=22 so max 21 15-7=8 so min 9 ab 9-21 jod do
523. Two equilateral triangles of side 2rt3 cm are joined to form a quadrilateral. The altitude of
the quadrilateral thus formed is equal to:
a. 8 cm
b. 6 cm
c. 4 cm
d. 3 cm
Ans- 3cm
Altitude of traingle=3,,,,,par ye altitute of Quadrilateral kaise nikala....i mean 2 diagonals to mil
gaye but whats Ht of Quadrilatral
diag. banaa bhai 2 trg se quad ans mil jayegaa
Ht of d quad is ht of d trng here
524. There is a regular octagon A B C D E F G , a Frog is at the vertex A. it can jump on to any
of the vertices except the exactly opposite vertex the frog visits all the vertices exactly once and
then reaches vertex E then how many times did it jump before reaching E ?
a. 7
b. 2n + 1
c. 6
d. can't be determined
Ans- total number of vertex 8.Starts from A,so 7 vertex left,hence total number of jumps are 7 to
reach each vertex.Question is how many jumps before E,so 7-1=6.
Correct me if wrong.
even the pattern will be A-1D-2G-3B-4F-5H-6C-7E
confusion hai....before E matlab E tak pahunche se pehle....
bilkul sahi sir, main b isi confusion me 7 ans de bhata tha.Paper me asi galti na ho bas

525. A table is offered for Rs 300 with 20% and 10% off. If in addition, a discount of 5% is
offered on cash payment, then the cash price of the table is :
a. Rs 240
b. Rs 216
c. Rs 210
d. Rs 205.20
Ans- d
ans de rakha hai b..:)
if last 5% discount is not included then ans will b 216
526.The length of a rectangle is increased by 60%. By what percent would the width have to be
decreased to maintain the same area ?
a. 37(1 / 2)%
b. 60%
c. 75%
d. None
Ans- A
526. There are two taps, first tap fills tank P and second tap fills tank Q. The rate of flow through
second tap is twice of that through the first tap. Tank P gets filled up in 4 hrs and tank Q gets
filled up in 6 hrs. Find the ratio of the capacities of the two tanks.
a. 2: 3
b. 1 :3
c. 3 : 2
d. 1: 2
Ans- b 1:3
527. The median of
0, 2, 2, 2, -3, 5, -1, 5, 5, -3, 6, 6, 5, 6 is :
a. 0
b. -3
c. 3.5
d. 5
Ans- 5
MODE --- jo sabse jyadda reapeat ho ..MEDIAN Middle value MEAN pata hi hai...;) DI mein
aate hain
sirf lyk nahi, ans bhi confirm kar de
ek aisa hi ques post kiya hai...iska sahi hai woh aur try karo

528. If one of the interior angles of a regular polygon is found to be equal to3/4 times of one of
the interior angles ofa regular hexagon, then the number of sides of the polygon is :
a. 8
b. 7
c. 5
d. 4
Ans- 4
528. Two circles touch each other internally. The sum of their area is 116 pi . And the distance
between their centre is 6 cm. Find the radii of the circles...
Ans- 10 and 4. I already posted this question.
529. In a triangle having sides 4 cm, 4 cm and 6 cm, what is the area of a quadrilateral formed
by the centroid, the orthocentre, the circumcentre and the incentre of the triangle?
16 sq.cm
8 sq.cm
3 sq.cm
non of these
Ans- d. Bhai Such quad is nt possible acc. to me as they all lie on same line i.e Median
Euler line kehte hain us line ko
530. if (x-2a)(x-5a)(x-8a)(x-11a)+ka^4 is a perfect square.then value of k???
Ans- 81 hai bhai log.... when u put x = 0 u get 880.. 880 + 81 = 961.. which is also a perfect
sqaure... but put x = any other no.. say 15a, or 21a or 29a..4a.. or any other value.. only 81 ..on
added to the result will get u a perfect square
the intention of the question was.. ki tum log jab jugad lagaoge main ... tab be watchful.. try for a
couple more nos... ur answer shud be a universal answer.. thik hai.. and the other imp thing "0"
may cause u to lose ur wicket.. :-P
880 mein options add karke dekhte jo bhi perf. sq deta usse tak marte..20 ke baad 81 hi
possible hai
531. A report consists of 20 sheets each of 55 lines and each such line consists of 65
characters. This report is reduced onto sheets each of 65 lines such that each line consists of
70 characters. The percentage reduction in number of sheets is closet to
a) 30% b) 15% c) 20% d) 25%
Ans- ans. c) 20% total no of characters= 20* 55 * 65 = 71500 , Number of pages required if
report is retyped=71500/( 65 * 70 )=15.70 hence 16 pages are required. Hence % reduction =
(20-16)/20 * 100 = 20 %

532. what are the last two digits of 7^2008


a) 21 b) 61 c) 01 d) 41 e) 81
Ans- (7^4)^502 =2401^502 and whatever power of 2401 is taken , last 2 digits will come as 01
in resultant.
533. A man invested 1/3 of his capital at 7% ; 1/4 at 8% and the remainder at 10%. If his
annual income is Rs. 561, the capital is
a. Rs. 5400
b. Rs. 6000
c. Rs. 6600
d. Rs. 7200
Ans- c
534. 3^x = 5^y = 45^z, then
a. x + y +z =0
b. 2/x = 1/z - 1/y
c. 2/z = 1/x - 1/y
d. 2/z = 1/y - 1/x
Ans- b
3^x = 5^y = 45^z = k (say)
3 = k^1/x; 5 = k^1/y
45^z = (3^2 * 5)^z = k = (k^2/x . k^1/y)z
=> k = k^(2/x + 1/y) = k^1/z--->2/x + 1/y = 1/z
xlog3=ylog5=z(log5+2log3)..isse b ans ayega..par mayank bhai ki baat bhi sahi hai..yahaan
desi kyun fail ho rha hai..
Saale angrez chale gaye,,,,,par videsi chod gaye
535. sides of the triangle are in a.p and largest angle is double of smallest angle then find the
ratio of side.
Ans- not correct, to bhai options to dedo
4:5:6
536. Two circles of radii 4 cm and 9 cm respectively touch each other externally at point and a
common tangent touches them at the point P and Q respectively. Then the area of a square
with one side PQ is
option- 97,194,72,144 sq cm
Ans- 144
(9+4)^2 = (9-4)^2 + ( common tangent lenght)^2..

537. ABC is a right angled triangle at A. BL and CM are its two medians ,
1 4(BL^2 + BC^2) = 5CM^2
2 4(BL^2 + CM^2) = 5BC^2
3 4(CM^2 +BC^2) = 5 BL^2
Ans- 2.....its a property
Ye example de rkha h NCRTE ki book ma .
538. x^2 + 3x +1 =0, then the value of,
x^3 + 1/x^3 is
A . -18
B. 36
C . 18
D. -36
Ans- -18
x^2+1=-3x and put in a^3+b^3 a=x b=1/x
539. In a circle of radius 5 cm, AB and AC are two chord with length 6 cm each. Find the Area of
Triangle ABC.
Ans-no, its an isosceles triangle with sides 6,6 and 9.6
hehe... ha hero log.. draw perpendicualrs from centre O to the chords AB and AC.. so we have
2 triangles with sides 3,4,5 units.. we need to find angle A.. use sin 2x = 2 sinx . cosx// formula
to find sin A... and from there find the area of isoscles trangle using ( l^2* sin of the angle) /
2........... sin A shud come 24/25... u cant miss it simple hai.. nahi aaya to bol do.. aur achche se
exaplian karunga
540. if perimeter of rectangle and square are same.then which has greater area.
Ans- sqr
bhai tringle ki sides lelo 3,4,5 to a=6 and sqr ki side=3 to area=9 so sqr>tringle
sqaure with side 4.. rectangle with sides 5 and 3
541. if x^2+y^2=1 than max. value of xy
Ans- put x=sin@ and y=cos@ xy=sin@. cos@ =1/2*2sin@ cos@=1/2 sin2@=1/2*1 =1/2
ok bhai.. without desi kafi dimag lagaya nahi hua
(x-y)^2 > 0...so x^2 + y^2 > 2xy....

542. y +1/z =1 and x+1/y =1 then find the vlaue of x*y*z


Ans- y=1/2, z=2, x=-1
-1,1/2,2....desi....-1
without assuming values: yz+1=z, xy+1=y. multiplying second eqn with z....xyz+z=yz.
xyz+yz+1=yz. so xyz=-1. :D
543. What z the max value of 1/(sin^2x+3sinx cosx+5cos^2x)
Ans- sin^2x+3sinx cosx+5cos^2x = 1 + 4 cos ^2 x + 3 sin x cos x ( sin ^2 x = 1 - cos ^2 x )
= 1 + 4 ( 1 + cos 2 X ) / 2 + 3/2 * sin 2 x ....( cos ^2 x = ( 1 + cos 2 x ) /2 and 2 sin x cos x = sin 2
x)
= 1 + 2 + 2 cos 2 x + 3/2 sin 2 x .....
= 3+ 2 cos 2 x + 3/2 sin 2x...the maximum value of 2 cos 2x + 3/2 sin 2 x is 5/2 and minimum is
- 5/2 ( rt a^2 + b^2 )
to have maximum value for 1/(sin^2x+3sinx cosx+5cos^2x), (sin^2x+3sinx cosx+5cos^2x)
should be minimum....
so minimum value of (sin^2x+3sinx cosx+5cos^2x)= 3 - 5/2 = 1/2 ....
so maximum value f 1/(sin^2x+3sinx cosx+5cos^2x) = 1/ 1/2 = 2
544. If x = 2 + (2)^2/3 + (2)^1/3 then the value of x^3 - 6x^2 +6x ?
4
2
-2
0
Ans- (x-2)^3 = [2^(2/3)+2^(1/3)]^3.......RHS mein 3x^2y + 3xy^2 karoge to 3xy common
lele...x^3 - 8 - 6x^2 + 12x =2^2 + 2^1 + 3*2(x-2)....now do it
545. if @ is positive acute angle and cos^2@+cos^4@=1 then find value of tan^2@ +tan^4@ .
option 3/2 ,1 ,1/2 ,0
Ans- 1
546. If the difference between areas of the circumcircle and the incircle of an equilateral triangle
is 44cm^2, then the area of the triangle is???
options: a) 28 b)7rt3 c) 14rt3 d)21
Ans- c 14rt3, find circumradius and inradius i.e a/root3 and a/2root3

547. if 4x=sec@ n 4/x=tan@, then 8(x"2-1/x"2) ?


" denotes square.
Koi desi hai kya? Process?
Ans-u dont need to apply desi.. aise hi karo best hai... hardly 15 secs
1/2*(16x^2 - 16/x^2 ) = 1/2* (sec^2 - tan^2) = 1/2
548. CosX+SecX=root3 then cos^3x+sec^3x=?
Ans- 0 cubing
549. The bisector of triangle ABC cuts BC in D and its circumcircle in E . Then
a. AB : AD :: AC:AE
b. AB: AC :: BD:DC
c. AB:AD :: AE:AC
Ans- b
Ye circum circle or inscribing /circumscribing circle mera dimaag kharab karte hai....inke liye koi
book batao...i hope rd sharma ne issse cover kia ho
550. ABC is in which <B=2<C....D is pt on BC such as AD bisects <BAC & AB=CD,,,,whats
<BAC=?
Ans- Its 72 given in RD SHARMA...Little bit complex...
551. Remainder when 17^37 + 19^37 is divided by 23=?
Ans- 13 if odd then a'n plus b'n
is a plus b divide by that so 17 plus 19 divide by 23
a^n + b^n is divided by a+b if n is odd
0 aayega answer
19 ki jagah 29 hai

552. the value of 1-1/20+ 1/20^2- 1/20^3+.........correct to 5 places of decimals is...


Ye kaise solve hoga???
options..
1.05
0.95238
0.95239
10.9
Ans- yeah... see the formual for infininte gp sum is 1/(1-r)..where r is the common difference..
here r = -1/20... ab karo
x=1-1/20(1-1/20+ 1/20^2+....)
x=1-x/20
x+x/20=1
21x/20=1
x=20/21

553. A polygon with n sides has n diagonals. What is the value of n?


Ans- 5
554. sec x + tan x = root 3, then the positive value of sin x is
Ans-
x=30.
555. A wheel rotates 3.5 times in 1 sec. What time it take to rotate thru 55 radians
Ans- (1\7pie)*55=5\2
556. The maximum possible reminder when (104)^n is divided by 51 is ? (n is a natural no)
Ans- 2*2*2*2*2.......agar 1 aur baar kiya to remainder 32 se kam ho jayega to max. is 32
tumhara logic to samzha par idhar 104 ka kya role hai?kya har 51 divisor ke liye ye rule hai
104/51...remainder =2, (104*104)/51....remainder 2*2 and so on....aise hoga bhai
557. if sin@+cos@ is rt2cos(90-@), thn cot@ is?
process? desi?
Ans- no need for desi.. simply divide by sin..
rt2-1
Simple hai. Aise Qs mein bhai desi lamba ho jayega.

558. In ABC,
Angle A=90,AD _|_ BC. If AD=4,BC=12 then cotB+CotC=?
Ans- triange banao....cot B = BD/4, cot C = DC/4...cotB+CotC = (BD+DC)/4 = 12/4 =3
559. 2sinAcosBsinC=sinBsin(A+C), then tanA, tanB, tanC are in
1. A.P.
2. G.P.
3. H.P.
4. A.P. and GP
Ans- a..b..c=45
Now if a,b,c=45,,,,,,tana,tanb,tanc=1 each then how its HP
1,1,1 are in ap,gp and hp all..as 1/1-1/1=1/1-1/1..0=0..so hp also..so u cant apply desi with
a,b,c=45
because 0 = 0...
2sina.cosb.sinc = sinb.sin(a+c)
2sinacosbsinc=sinb(sinacosc+sinccosa)
2sinacosbsinc-sinasinbcosc=cosasinbsinc
now divide by cosacosbcosc
2tanatanc-tanatanb=tanbtanc ho jayega..
So 1/tanb-1/tana=1/tanc-1/tanb..means tanA, tanB, tanC in HP
560. Which of the following can be a product of two 3-digit numbers **3 and ** 8 ?
a. 1010024
b. 991014
c. 9124
d. None
Last ques..dekhte hai India kab Mujhe wapas lautne ka mauka deti hai..kabhi bhi 270+target
chase nahi huya hai RPS colombo mein
Ans- b,,as 999*999 = max 6 digit possible

561.cosx/cosy=n and sinx/siny=m then (m^2-n^2)sin^2y=?


Ans- 2/3
put x = 60 y =30.
But sir if we put X=Y=45 then ans comes 0 :(
arre hero... this is the same thing i told other day.. u shud put such values so tht neither of term
gets redundant.. if a term is there it must be having some effect on the eqn... so its not wise to
put values as such tht remove the effect of one of the terms.. it MAY work.. but not always.. (Y)

562. A discnt of 30% on MP of a toy reduces its SP by rs 30. whats its new SP?
Xplain
Ans- 70
hero.. let the MP be 100 Rs..if u r giving a discount of 30% and sp is reduced by 30 Rs.. then
the new Sp has to be 70
0.7x=(x-30) where x- old sp
563. If Usain Bolt Covers 100 mtr in 9.60 sec than his speed in Km/hr
Ans- 37.5km/h
Data insufficient...you havnt mentiond wind factor assisting or resisting him!
564. A hemispherical bowl B1 and a hollow right circular cylinder B2 (having length equal to its
radius) have the same diameter equal to the length of a side of a hollow cubical box B3. Water
is filled in all these vessels up to the same level and such that hen-dspherical bowl is full of
water and the volumes of filled water are V1, V2 and V3, respectively, in B1, B2 and B3, then:
a. V1< V2< V3
b. V2 < V3 < V1
c. V3 < V2 < V1
d. V3 < V1 < V2
Ans- a
565. A circus tent is cylindrical to a height of 3 m and conical above it. If its diameter is 105 rn
and slanting height of the cone is 53 m, then the area of the, canvas used is equal to:
a. 9435 sq. cm
b. 9535 sq. cm
c. 9635 sq. cm
d. 9735 sq. cm
Ans- d., pir(l+2h)
ques metre mein hai yaa cm mei?
mtr mein option mein galat de rakha hai ssc aisi hi cut copy paste karta hai
566. A and B together can do a piece of work in 12 days which B and c can do in 16 days .After
A has been working at it for 5 days ,and B for 7 days ,C takes up and finishes it alone 13 days
.In how many days could each do the work by himself??
EXPLAIN
Ans- A+B=5 ,B+C=2 C=11 leke karo
a,16 b..48 c,24

567. two men A and B working separately ca mow a field in 8 and 12hrs respectively .If they
work in stretches of one hr alternately ,A beginning at 9 am when will the mowing be finished??
Ans- 6.30 pm
568. A round balloon of unit radius subtends an angle of 90* at the eye of an observer standing
at a point say A. What is the distance of the center of the balloon from point A
1/root2
root2
1/2
2
Plz explain
Ans- Root 2
569. A person who has a certain amount with him goes to market. He can buy 50 oranges or 40
mangoes with this amount. He retains 10% of the amount for taxi fare and buys 20 mangoes
and oranges. Number of oranges he bought is ?
a)36 b)40 c)15 d)20
Ans- d
570. On two of his tests, Harry scored 30 out of a maximum of 50 and on the third he scored 40
out of a maximum of 100? What is his average percentage score on the three tests?
(a) 33.33%
(b) 40%
(c) 50%
(d) 53.3%
Ans- 60+60+40 = 160 ........so 160/3 = 53.3
571. A, O , B are three points on a line segment and C is a point not lying on AOB. If angle
AOC=40 and OX and OY are internal and external bisectors of angle AOC respectively. then
angle BOY is??
Ans- ek straight line draw karo AOB, C upar ek point lo now draw triangle ABC ............ab
triangle AOC = 40* iska external bisector 140/2 = 70* hoga
572. What is the unit digit of the sum 1! + 2! + 3! +... + 20!
a. 0 b. 2 c. 3 d. 5
Ans- 3 just see till 4! Uske aage zero aenge sabme unit place par....
factorial syllabus me hai??????

573. what is the sum of all two digit numbers that give a remainder of 3 when they are divided
by 7 ?
a)666 b) 676 c) 683 d) 777
Ans- AP 10 ,17.....94 ...........sum of the AP = 676
n=13 then find the sum a=10 d=7 ans is 676
574. Consider a sequence where nth term tn = n/(n+2) , n=1, 2 . The value of t3 * t4 * t5 *
. t53 equals
a) 2/495 b)2/477 c) 12/55 d)1/1485
Ans- ans. a) 2/495
575. What is the remainder when 7^74 -5^74 is divided by 4?
a)1 b)2 c)0 d)1
Ans- 0
it will be divisible by 2 and 12 both. so must be divisible by 4 also.
576. a^3-b^3=56, a-b=2, thn a^2+b^2 is?
I know its desi method, want to knw traditional way.
Ans- a^3-b^3 = 56. (a-b)(a^2+ab+b^2) = 56. a^2+ab+b^2 = 28. Now a^2+b^2 = (a-b)^2+2ab.....
a^2+b^2+ab = (a-b)^2+3ab....28=4+3ab.....ab=8..... a^2+b^2 = (a-b)^2 + 2ab. substitute a-b, ab
to get a^2 + b^2 as 8.....
a^3 -b^3 = (a-b)*(a^2+ab +b^2)............so divide it by a-b = 2...so a^2+ab +b^2 = 28 .........nw (ab)^2 se a^2+b^2 ki value put kar do ..........nw solve ........ab = 8
thanx...but its better searchng for desi i think
yaa....desi is much more better....a=4, b=2....khel khalaas
577. When you revers the digits of the number 13, the number increases by 18. How many
other two digit numbers increase by 18 when their digits are reversed?
a)5 b)6 c) 7 d)8
Ans- 6
24,42..35,53......46,64.....57,75......68,86,,,,,,,,79,97,,,,,,aur koi ho to add karo
X- Y = 2 ..........so there are 6 value of x and y satisfying it
(10x+y)-(10y+x)=18==>x-y=2..ab 13 k unit digit aur ten's digit mein 2 add karte raho..35,57,79
ayega,,isi tarah even digits 24,46,68 ayega
did u took no as yx so (10x+y)- (10y+x)=18 else in case of digit as xy it would be (10y+x) - (
10x+y)=18

yes i took no as 10y+x..or u can say yx


chalo ye bhi jaan lo.. aisa sum rahega intercahnging the digits ka... then the difference between
the orginal no and interchanged no is alwasy multiple of 9.. and if xy is the no.. and yx is the
interchanged no.. then the difference tht is 18 ias in this case.. so 18/9 = 2.. so the nos can be
such nos where... x- y = 2
578. if a^2-ab+b^2
______________ = 1/3 then find a/b
a^2+ab+b^2
a) 1 b) 2 c)3 d)4
Ans- put a=b=1
Option a
if you want to solve it in traditional way then multiply it ........sare term ko ek side le jao .........phir
ye (a*rt2- b*rt2)^ =0 ............nw a*rt2 = b*rt2 ........so a=b
579. 2-cos^2theeta=3 sin (theeta)cos (theeta), sin theeta not equal to cos theeta then tan theeta
equal to
1.1/2
2.0
3.2/3
4.1/3
Ans- hoga
bhai cos^2@ se divide karlo..2sec^2@-2 = 3 tan@..sec^2 @ ko 1 + tan^2@ likh lo..equation
banao ho jayega..!
580. Two vessels contain mixtures of spirit and water. In the first vessel the ratio of spirit to
water is 8 : 3 and in the second the ratio is 5 : 1. A 35-litre cask is filled from these vessels so as
to contain a mixture of spirit and water in the ratio of 4 : 1. How many litres are taken from the
first vessel?
1) 5 lt
2) 6 lt
3) 8 lt
4) 11 lt
5) 24 lt
Ans- 11lt

581. a person walks a distance in 114 days when he rest 9 hours a day.how long will he take to
walk twice the distance if he walks twice as fast and rest twice as long each day as before...??
a.57
b228
c..285
d.324
Ans- 570/2 = 285
582. On what sum of money will the simple interest for 3 years at 8% per annum be half of the
compound interest on Rs. 400 for 2 years at 10% per annum?
a. Rs. 125
b. Rs. 150
c. Rs. 175
d. Rs. 200
Ans- 175
583. Arun, Barun and Kiranmala start from the same place and travel in the same direction at
speeds of 30, 40 and 60 km per hour respectively. Barun starts two hours after Arun. If Barun
and Kiranmala overtake Arun at the same instant, how many hours after Arun did Kiranmala
start?
1) 3
2.) 3.5
3) 4
4.) 4.5
5) 5
Ans- 3)4
584. The average of 5 consecutive numbers is n. If the next two numbers are also included, the
average will :
a. increases by 1
b. remain the same
c. increase by 1.4
d. increase by 2
Ans- a

585. the ratio of x to y is 25 times the rstio of y to x rhen what is the ratio of x to y?
Ans- x^2/y^2=1/25
1:5
x/y= 25* y/x
(X/y)^2 = 25:1
So x:y = 5:1
confirm 5:1 hi aega na..i dnt hav ans..ye cds k ques paper ka hai..
586. Which one of the following statements is NOT correct?
a. If the sum of angles on an n sided polygon is n right angles, then n = 4
b. No regular polygon can have interior angle equal to 110
c. A regular polygon with 180 sides has each interior angle equal to 178
d. No regular polygon can have interior angle equal to 140
Ans- No regular polygon can have interior
angle equal to 140
Thos is not correct.
587. If an equileteral triangle of side 6 cm is rotated about an axis perpendicular to the plane
and passing through the centroid, then what is the area of the figure that is formed.
36pi
12pi
3pi
9pi
Ans- 12pi
588. A ladder leans against a vertical wall.The top of the ladder is 8 m above the ground.When
the bottom of the ladder is moved 2 m farther away from the wall, the top of the ladder rests
against the foot of the wall. What is the length of the ladder?
a. 10 m
b. 15 m
c. 20 m
d. 17 m
Ans- bhai 17 ans hai..... 8 perpendicular.....15 base....so option d

589. The ratio between the volume of a sphere and the volume of the circumscribing right
cylinder is :
a. 1 : 2
b. 2 : 1
c. 1 : 1
d. 3 : 2
e. 2 : 3
Ans- e 2:3
4/3 pi r^3 / pi * r^2 * 2r
4/3 * 1/2
2:3
ha hero.. volume of sphere = 4/3 * pi * r^3.... volume of the circumsrcibing cylingder = pi * r^2 *
(2r)..
590. 0.204 x 42 / 0.07 x 3.4 is equal to :
a. 6
b. 0.6
c. 0.06
d. 1 / 6
Ans- a.6
591. If a * y * z = (a + 2) (y + 3) / (z + 1) , the value of (6 * 15 * 3) is :
a. 2
b. 3
c. 4
d. None
Ans- d. none 6.
592. A college has raised 75% of the amount it needs for a new building by receiving an
average donation of Rs 600 from the people already solicited. The people already solicited
represent 60% of the total people the college will ask for donations. If the college is to raise
exactly the amount needed for the new building, what should be the average donation from the
remaining people to be solicited?
a. Rs 300
b. Rs 250
c. Rs 400
d. Rs 500
Ans- a. Rs.300
take..total..as..100..75%=60..solicited..amout=36000..total.chaiye..48000..so.remai=1200\40..pp
l=300rs

593. A sum of money trebles itself in 15 years 6 months. In how many years would it double
itself?
a. 6 years 3 months
b. 7 years 9 months
c. 8 years 3 months
d. 9 years 6 months
Ans- b 31\4..years.
594. two men undertook to do a job for rs. 1400 ,one of them alone can do it in 7 days and the
other in 8 days .With the assistance of a boy they together completed thr work in 3 days .How
much money will the boy get??
Rs 300
rs 325
rs 275
rs 250
Ans- ratio 24:21:11
1/3-1/7-1/8=1/x
x=168/11 days
boy's part = 1/3*168/11=56/11
boy will get= 11/56*1400=275 rs
595. A can do a piece of work in 7 days of 9 hours each and B can do it in 6 days of 7 hours
each. How long will they take to do it, working together 8(2 / 5) hours a day ?
a. 3 days
b. 4 days
c. 4(1 / 2) days
d. None of these
Ans- 3 days
596. If x + y + z = 1 and x, y, z are positive numbers then (1 - x)(1 - y)(1 - z)>= A*xyz where A is
a positive integer. Find the most suitable value, of A.
a. 6
b. 8
c. 9
d. 10
Ans- (a+1)xyz=xy+yz+zx..put.x..y..z=1\3..so..A=8

597. ABCD is a rhombus with diagonals AC & BD intersecting in at the origin in the xy plane. If
equations of AD is x + y = 1 then the equation of line BC is
a. x + y = 1
b. x + y = - 1
c. x - y = 1
d. none of these
b....parallel lines
aise questions aane chahiye.....pen uthane ki zaroorat hi naa ho jisme
598. The population of a city increases at the rate of 5% annualy. Its present population is
1,85,220. Its population 3 years ago was :
a. 121500
b. 143433
c. 160000
d. 127783
Ans- 160000
599. tea worth rs 126 per kg and rs 135 per kg are Mixed with a third variety in the ratio 1:1:2 .if
the mixture is worth rs 153 /kg then the price of the third variety per kg will be
Ans- 126+135+y*2=153*4
126+135+2y=612
y=175.5
600. Two circles touch each other externally. The distance between the centres of the circles is
35 cm and the sum of their areas is 2002 sq.cm. How many times the smaller radius is larger
radius. ?
Ans- 1.5
r1+r2=35.....r^2+r2^2=1225-2r1r2
22/7(r1^2+r2^2)=2002
22/7(1225-2r1r2)=2002
1225-2r1r2=637
r1r2=294
two factors of 294 ....21 & 14 whose last digit addition is 5.
or r1-r2=1225-4*294=49
r1=21
r2=14
x^2 - 35x + 294 = 0
So radius 14, 21
21/14 = 1.5

chalo meri taraf se ek trick... 637 ..main "7"hai units digits.. which can be either 1,6
...2,5...3,4...9,8.....leaving the first none of the other pairs are possible.. these are the units digits
of r1 ^2 and r2 ^2..
601. A and B completed a work in 5 days. Had A worked at twice the speed and B at half the
speed, it would have taken them four days to complete the job. How much time would it take for
A alone ot do that ?
10
20
25
15
Ans- a+b=1/5
2/x+1/2x=1/4
5/2x=1/4
x=10 days
1/A + 1/B = 1/5
1/2A + 2/B = 1/4
Solve and equalize ans is A=B so double of 5 Days :)
602. a book seller bought 200 textbooks for rs 12000 .He wanted to sell them at a profit so that
he got 20 percent books free. At what profit percent should he sell them.
Ans- 1 book ka price = 60..20 % books free which means 40 books free..then 160 books sell
karega 12000 ki..S.p = 12000/160 = 75profit = (75-60/60)*100
160 kyu hoga bhai 220 hoga na..
ye books me kitna misprint hota hai 20 percent se ans nahi arha 20 books se arha hai..ap check
kro agar 20 percent se arha hoto..ans is 10 percent
603. If 12 man and 16 boys can do a piece of work in 5 days and 13 men and 24 boys can do it
in 4 days. Compare the daily work done by a man with that done by a boy ?
1:2
1:3
2:1
3:1
Ans- 60x+80y=52x+96y..thrfr..x\y=2\1

604. The HCF and LCM of a pair of numbers are 12 and 926 respectively. How many such
distinct pairs are possible?
(a) 3
(b) 7
(c) 1
(d) 0
Ans- no pair exist as 926 is nt divisible by 12..!
605. In a triangle ABC, a^2 + b^2 + c^2 = ac + ab rt 3, then the triangle is...
1. equi 2. iso 3. right 4. None
Ans- I did put a=b=C=1 & solved .,,,no equilatrl,righ...may b none
haar case lagaa ke dekh lo ....d ans hai
606. If 8tanx = 15, then find the value of (sinx-cosx) ??
Ans- (15-8)/17 = 7/17
607. If (x + 2)^2 = 9 and (y + 3)^2 = 25, then the maximum value of x / y is ???
Ans- 5/8
x is -5
y is -8
x + 2 = -3 so x = -5
Y+3 = -5 so y = -8
X/y = 5/8....correct dekhke kar raha tha sirf..:p
608. Two circles touch each other internally.The sum of their area is 116 pi . And the
distance between their centre is 6 cm.Find the radii of the circles
Ans- 10, 4
10^2+4^2=116
609. a barrel contains a mixture of wine and water in the ratio of 3:1 How much fraction of the
mixture must be drawn off and substituted by water so that the ratio of wine and water in the
resultant mixture in the barrel becomes 1:1???
A.1/4 b.1/3 c.3/4 d.2/3
Ans- first barrel wine is 75% ....in final mixture it is 50% .......let fraction be X , so 75*(1-x) = 50
..........so x = 1/3
1/3....take total mxture as 120....it will take less than 1 min to solve
2-----1 i.e 2/3 and 1/3 so 1/3 is the ans

610. ax+by=6, bx-ay=2 n x^2+y^2=4 thn (a^2+b^2)=?


process?
Ans- 10
put..x=rt2..y=rt2..phir..easy..ho..jyga
(ax+by)^2 = 6^2......................(1)
bx-ay)^2 = 2^2 ......................(2)
add both eq. and rest is the result
a=rt2 b=2rt2
611. if tan A= 1-cosb/sinb then what is 2tana/1-tan^2a equal to ......
1.tanb/2
2.2tanb
3.tanb
4.4tanb
please explain
Ans- 1-cosb = 2sin^2(B/2).sinB = 2sinB/2cosB/2. so tanA = tanB/2. so tan2A = tanB
(1-cosA)/sinA =tan(A/2)
(1+cosA)/sinA= cot(B/2)
612. A painter takes 5 days to paint a room. How much time would it take him to paint a room
without the roof twice as long , twice as broad and twice as high ?
5
40
10
20
Ans- let me explain ...........original surface are to be painted = 2( l+ b) *h ...........new surface
area to be painted = 2(2l + 2b) * 2h = 4* original area so it will take him 4*5 = 20 ............correct
me
16 hai
iss question mein kuch gadbad hai...without roof karenge to time kam lagna chahiye na???
jyada kaise lag raha hai???

613. the rightmost non-zero digit of the number 30^2720 is a) 3 b) 7 c) 9 d)1


Ans- 30^2720 = (3*10)^2720 = 3^2720 * 10^2720....3^2720's unit digit 1.....so rightmost nonzero digit 1 :D
2720/4 = 680. so by the time 3's power is raised to 2720...680 cycles of 4 steps will be
over....and unit digit will correspond to 4th step of 680th cycle. 3's unit digit's 4th step = 1. (3, 9,
7, 1)
If it is 2722...then 2722/4=680 and remainder 2. so 680 cycles of 4 steps over and unit digit
corresponds to 2nd step. so it will 9. if remainder comes as 1. then 680 cycles of 4 steps
complete and unit digit corrsponds to 1st step....i.e. 3.
it will go like this: 3,9,7,1...20th term=4th term i.e 1
614. The mean of 100 abservations was calculated was 40. It was found later on that one of the
observations was misread as 83 instead of 53. The correct mean is :
a. 39
b. 39.7
c. 40.3
d. 42.7
Ans- 39.7
615. The price of sugar is increased by 20%. As a result, a family decreases its consumption by
25%. The expenditure of the family on sugar will be decreased by :
a. 10%
b. 5%
c. 14%
d. 15%
Ans- let the rate be 1rs/kg. let the consumption be 100kg. 20% price increase = 1.2rs/kg. and
consumption = 75. so expenditure = 75*1.2=90. so reduction of 10% in expenditure.....any other
method puys?
x+y-xy/100
100x100=120x75
Diff is 900...
1000-900=10%

616. if the selling price of an article is 8/3 times its cost price then the profit percent on it is..
Ans- (5/3)*100=166.6
exact arha hai...option r 120 percent,160,40,60
are you sure that the ans given is correct???
book me misprint hai..8/5 hona chahiye...
then it's 60

617.In a certain office, 72% of the workers prefer tea and 44% prefer coffee. If each of them
prefers tea or coffee and 40 like both, the total number of workres in the office is :
a. 200
b. 240
c. 250
d. 320
Ans- 72 + 44 = 116. so 16 like both. 16% = 40 so 100% = 250 :D
618.A cylinder 105 cm high has a radius of 10/pi cm. A string makes exactly 7
complete turns round the cylinder while its two ends touch the cylinder's top and
bottom. How long is the string in cm?
Ans- It is divided into 7 parts.
we can take 7 rectangles and sum of its diagonals will be the required length.
SA of cylinder is 2100
so one section will be
2100/7 = 300
total height is 105. so for one section its 15.
base will be 300/15=20
its diagonal is rt(20^2+15^2)
25.
so total 25*7 = 175.
619. if x= - 0.5 then which of the following has smallest value?
a) 2^1/x b)1/x c) 1/x^2 d) 2^x
Ans- b's value will be negative....rest all positive....so b :D

620.Sum of the square of any three consecutive odd number increased by 1 is divisible by ?
9
12
16
18
Ans- 12 ...................12n^2+11+12n then add 1 divisible by1 2 add as said
bhai just try with no's 3,5,7...12 will only satisfy the question
621. of the three numbers,the ratio of the first and the second is 8:9 and that of the second and
third is 3:4 .if the product of the first and third no. Is 2400 ,then the second no is
Ans- 45.....no's are 40,45,60
622. A and B together can do a piece of work in 12 days, which B and C together can do in 16
days. After A has been working at it for 5 days and B for 7 days, C finishes in 13 days. In how
many days C alone will do the work ?
a. 16
b. 24
c. 36
d. 48
Ans- 5/12 + 2/16 = 13/24 ........so 11days work of c is 11/24 .....so C can complete it in 24 days
Waah.....kya faadu solve kiya hai....maan gaye bhai maan gaye....ek se ek expert hai yahan
623. A tank, of capacity 1000 litres, contains a solution with equal amounts of alcohol and water.
Each time 100 litres of solution is extracted, it is replenished with an equal amount of water.
After 4 times, the solution contains x% alcohol and rest water. The value of x is:
a. 35%
b. 30%
c. 36.45%
d. 37.55%
Ans- 32.80...but it is not in the options :-( kya gadbad hai??
waise yaha 10% depreciation apply karke we can approximate kar sakte hai ki 30 -35 ke beech
main ans aayega ..........agar option close na ho to 100% accurate
(x(1-100/1000)^4,,,,,,,, 500*(9/10)^4= 729/20=36.45
your process is right but i think you are doing some calculation mistake ........please check
we spend lot of time in wrong option in exam ...........yahan bhi maine is sum ke peeche 5 min
laga diya .......I think SSC is doing this trick with us ............So be prepared

624. If [x^4 + (1/x^4)] = 727, the value of [x^3 - (1/x^3)] is ??


Ans- 140
625. how to find the min. Value of
cosX plus secsquareX
Ans- It's 0, for x=180. {-1+(-1)^2} = 0
bhai jugad laga hi diya..:P
Aur iski value negative ho nhi sakti bcoz cosx {-1 to 1} and sec^2 x {>=1}
626. A walks at a uniform rate of 4 kmph nd 4 hr after his start B biycles after him at the uniform
rate of 10 kmph ,How far from he starting point will B catch A.
Ans- when B will start A will cover 16km .......nw B will cover it with 10-4 = 6km/h .....16/6 hr ..nw
10*16/6
4 ( x+ 4 ) = 10*x..x = 8/3..distance = 10*x = 10*8/3
bhai orally karlo...16--0, 20--10,24--20,26--25,26.67--26.67
627. (49^15)-1 is divisible by..8 ..28..50..51
Ans- 8
48 se hogaa toh 8 se bhi hogaa
Unit digit of Q=8...so 8 se divide hone ki prob. Jyada thi wrt other options...
a^n-b^n formula kaise lagega idhar?
Ok...n odd hai to 49-1 se div hoga
628. each member of a picnic party contributed twice as many rupees as thr total collection was
rs 3042 .the number of members present in the party was
Ans- 39
2x^2=3042
629. value of sin^6 @ + cos^6 @ + 3 sin^2 @ cos^2 @
Ans- 1
1 cube of sin square plus cos square
Put @=0 or 90

630. If the rate of the inflation is 1000% then what was the cost 3 years ago of an item that costs
6655 now ?
Ans- 5
6655/( 1+ 1000/100)^3..compound interest
6655\1331
631. A piece of paper is in the shape of a square of side 1 m long. It is cut at the 4 corners to
make a regular polygon of 8 sides. The area of the polygon is?
a. 2 ( rt 2 - 1 ) b. ( rt2 - 1 ) c. 1/ rt 2 d. none
Ans- a
direct formula lo >> a^2 * ( 2 + (2* rt. 2))
let the side of octagon be "a" ..then a+a/rt2 + a/rt2 = 1 .......nw go ahead
pythagoras 45-45-90 laga ke cut kiya hue side ki length find ki
Put ques quickly...have to watch olympics from 1pm
632. A takes 2 days to finish a task, B takes twice that amount of time, C takes twice of B and D
takes twice of C. If they do it in pairs, one pair takes twice the amount of the time as the other .
Which is the pair that takes longer?
AB
BC
BD
CD
Ans- BD 20 days
yar A KO 2 DIN LAGE B KO 4 , C KO 8 N D KO 16..now If they do it in pairs, one pair takes
twice the amount of the time as the other..2 pair banao COZ TWICE THE AMOUNT OF TIME
h..then pair will be A+C N B+D as B+D takes double time of A+C..gt it?
633. A student goes to school at the rate of 2.5 km/hr and reaches 6 minutes late. If he travels
at the speed of 3km/hr, he is 10 minutes early. The distance between the school and home is ?
Ans- 4km
x/2.5 - x/3 = 16/60..!

634. A walks at a uniform speed rate of 4 hr after his start ,B biycles after him at the uniform rate
of 10 km an hr.how far from the starting point will b catch A
a.16.7 km b.18.6 km c. 21.5 km d.26.7km
Ans- Data insufficient....
635. If G is d centroid of trg ABC, AG=BC, then angle BGC?
Ans- G ko centre consider kar ek circle draw karo jo C, B to touch karta ho ...........BC diameter
hoga us circle ka ..........nw solve
636. least value of 12sinA - 9sin^2A
Ans- 12 sin A - 9 sin ^2 A = - ( 9 sin ^2 A - 12 sin A )
= - ( 9 sin ^2 A - 12 sin A + 4 ) + 4
= - ( 3 sin A - 2 ) ^2 + 4
= 4 - ( 3 sin A - 2 ) ^2 ...
for maximum value square term shoud be zero as any higher values will fetch a result less than
4.....so

max value = 4
for minimum value square term should be maximum , for maximum value of square sin A = -1,
sothat 3 sin A - 2 = -5
so least value = 4 - ( -5 ) ^2 = 4 - 25 = - 21
637.The least number which when divided by 20, 25, 35 and 40 leaves the remainder 14, 19, 29
and 34 respectively, is :
a. 1664
b. 1406
c. 1404
d. 1394
Last Ques---LONDON DREAMSSSSSSSSSSSSS
Ans-d 1394
Here (20-14) = 6, (25 -19) =6, (35 - 29) =6 and (40-34) =6. So, required number = (l.c.m. of 20,
25, 35, 40) - 6 = 1394---concept ke liye

638. After replacing an old member by a new member, it was found that the average age of five
members of a club is the same as it was 3 years ago. What is the difference between the ages
of the replaced and the new member ?
a. 15 yrs
b. 8 yrs
c. 4 yrs
d. 12 yrs
Ans- Age decreased = (5 x 3) years = 15 years. So, the required difference = 15 years
639. a wheel of radius 2.1 m of a vehicle makes 75 revolutions in one minute. What is the speed
of the vehicle?
78 kmph
59.4 ''
37.4 ''
35.4 ''
Ans- 2.1*2*22/7*75/60=16.5 m/s or 16.5*18/5=59.4 km/hr
640.The perimeter of an isosceles right angl trngl is 2p unit. The area of d trngl is
a)(2+rt2)p"2 b)(2-rt2)p"2 c)(3-rt2)p"2 d)(3-2rt2)p"2
process?
Ans- take A as equal sides so hypotenuse will be rt2*a
now perimeter = 2P (given)........2a+rt2*a = 2p
Let equal sides x then hyp.=rt2x ana a/c qstn x+x+rt2x=2p nw go ahed shayd ans. C h.
a = rt2/1+rt2 *p.....rest can be easily done...........ans----D
Ans- d
641. The value of sin^25*+sin^210*+sin^215*......sin^285*+sin^290
Ans- itz 9.5 sure
its square for every term
5 to 40 8 terms
sin^2+cos^2
so 8 + sin 90+sin^245
642. the cost of 2 articles r in the ratio of 3:5 .If there is 30 percent loss on the first article and 20
percent gain on the second article, what is overall percentage of loss or gain?
Ans- take 3= Rs 30 and 5= Rs 50 and clculations

643. Thoda hat ke


the maximum number of identical coin from which a faulty one (weighing less then all other) can
be found in three weighing on a simple balance ?
25
27
29
23
Ans- 27
This was just for extra recharge of mind .........no use of SSC

644. remainder when 15^23+23^23 is divided by 19 a) 4 b) 5 c) 2 d) none of these


yar 15^23+23^23 will be divisible by 38 n obviously by 19..thus remainder will be 0..!
a^n + b^n is always divisible by a+b if n is odd .........this is the rule
(x^n+a^n+b^n) is divisible by
(x+a+b) when n is odd and x, a and b are in AP
answer is 0 and a^n + b^n is always divisible by a+b if n is odd and positive
645. What is the least no that shoud be multiplied to 100! to make it divisible by 7^18 ?
Ans- on diving 100! by 7, then by 7^2 we get 14+2 ( 14 times divided by 7 and 2 times by 7^2)
so 100! is divisible b 7^16 only. to make it divisible by 7^18 we need to multiply by 7^2=49
646. a/b= 1/3, b/c= 2, c/d = 1/2, d/e =3 and e/f= 4 then what will be value of abc/def a) 3/8 b)
27/8 c)3/4 d)27/4 e)1/4
a/b* b/c* c/d=1/3 ie. a/d=1/3 & c/d* d/e=3/2 ie. c/e=3/2
b/c *c/d *d/e *e/f=3/4 ie. b/f=3/4
now a/d * c/e * b/f= 3/8
bhai b/f ki value is 12
It also give 6.
a/d*c/e= 1/3*3/2= 1/2
opts ke according hi ans lana tha bhai logo.

647. A tennis tournament is held in a school where every student plays one game against every
other student. There are 210 boys vs boy game held and 300 girs vs girls games. How many
boys vs girl games were held ?
1-40
2-225
3-810
4-525
Ans- 4-525
648. which among 2^1/2, 3^1/3, 4^1/4, 6^1/6 and 12^1/12 is the largest?
Ans- 3^1/3 take lcm of 2,4,6,3, 12
649. what is one of the square root of 16x^6-24x^5+25x^4-20x^3+10x^2-4x+1 question
1. 4x^3-3x^2+2x+1
2. 4x^3-3x^2-2x-1
3. 4x^3-3x^2+2x-1
4. 4x^3-3x^2-2x+1
Ans- Ha bhai x=1 3 anna chaiye option
hehe,,, method to shai lag rahi.. but answer c aayega
c is answer calculation le doobegi
650. The rate of simple interest on a sum of money is 6% p.a. for the first 3 years, 8% p.a. for
the next 5 years and 10% p.a. for the period beyond 9 years. If the simple interest accrued by
the sum for a total period of 10 years is Rs. 1560, what is the sum?
a. Rs. 1500
b. Rs. 3000
c. Date inadequate
d. None of these
Ans- d
for the period beyond 9 years ki jagah 8 years ayega..!
15600/78 = 2000
651. 3/x + y + 2/x-y =2 and 9/x+y - 4/ x-y =1 then what is x/y equal to??
Ans- 5
x+y =3
x-y =2

652. what is one of the square root of 16x^6-24x^5+25x^4-20x^3+10x^2-4x+1


653. a number consists of two digits .the sum of the digits is 10, on reversing the digits of the
no. The no. Decreases by 36. What is the product of the two digits
Ans- 3*7=21
654. if both the radius and height of a right circular cone are increased by 20 percent ,its volume
will b increased by.
Ans- %increase =3a+3a^2/100 + a^3/100^2,,,,hr a=20 for d ques
3x+ 3x^2/100 +x^3/100^2....
But me aise nahi karti...
20+20+20*20/100 =44
44+20+44*20/100=72.8 .
655. A boy multiplied 423 by a number and obtained 65589 as his answer. If both the fives in
the answer are wrong and all other figures are correct, the correct answer is :
a. 62389
b. 61189
c. 62189
d. 60489
Ans-65589/423=155 with 24 remainder,,tht should nt come,,,
unit digit should be 9 as given,,,so the multiple digit's unit digit should be 3 only so,,143 is just
below 155 having 3 as unit digit,,so let multiply them
423*143=60489........yippy we get an option,,,(d)
656. A train travels at an average speed of 200km/h without any stoppages. However its
average speed decreases to 120km/h on account of stoppages. On an average how many
minutes per hour does the train stop?
Ans- short method- in 60 minutes train covers 200 kms ie. in 6 minues it covers 20 kms. so if its
speed is 120 km due to stoppages means stoppage of 80 km. so in 6 minutes 20 km then 80 km
in 24 minutes.
der is formula...
Time of rest per hr= difference of speeds/speed without stoppage *60
200-120/200 *60=24
657. the sum of a number and its reciprocal is 10/3 then the numbers Are
Ans- 3 and 1/3

658. if (x+1)^2 =16 and (y+1)^2 = 25 then the maximum value of x/y is ?
Ans- 5/6
x+1= plus/minus 4 if we take minus siign value then x+1=-4, x=-5 and same for y=-6
659two kinds of Vodka are mixed in the ratio 1:2 and 2:1 and they are sold fetching the profit
10%and 20% respectively. If the vodkas are mixed in equal ratio and the individual profit
percent onthem are increased by 4/3 and 5/3 times respectively, then the mixture will fetch the
profit of
(a) 18% (b) 20% (c) 21 % (d) 23% (e) Cannot be determined
Ans- Let p% be profit on type 1 and q% on type 2.
Let C.P. for both = 100
1st : Mix them in ratio as 1:2 and get profit = 10%.
p+2q = 3*10
p+2q = 30
2nd : Mix them in ratio 2:1 and get profit = 20%
2p+q= 3*20
2p + q = 60
Solving both the equations
p = 30 and q = 0
3rd : Increase p by 4/3 and q by 5/3 and mix them in ratio 1:1
p = 40% and q = 0
Profit = (40/200)*100 = 20%
660. range of sin^2A + cos^4A
Ans- 3/4 to 1
put 0 n 45 degree
SSC ke itne group h Fb par ki sabko join karlo toh khane peene ka tym nhi milega..PG was
better bt the new version is pathetic..!
661. find the range of 1/(2sin^2A + 3cos^2A)
Ans- no need to go for derivative..1 / (2sin^2A + 3cos^2A) = 1/ ( 1 + cos^A)..cos A can range
from 0 to 1..ho gya
converting it to sin i got....1/(3-sin^2A) ....now when sin is max....denominator will be min and
expression will be max....sin^2A max = 1. so 1/2 is max. sin^2A min = 0, so 1/3 min.

662. if A+B+C=PIE,
&cosA = cosB.cosC
than tanB + tanC= ?
Ans- tanB +tanC = sin(B+C)/cosBCosC
sin(180-A)/cosA
=tanA
bhai cosa=cos{180-(b+c)} laga lo
663. tanA = m/m+1
tanB = 1/2m+1
find value of tan(2/3(A+B))
Ans- desi..m=0
m=0 ..A=0 B=45 TAN30=1/RT3
664. a+b+c=6 a^2+b^2+c^2=26 then the value of ab+bc+ca is equals to.
0245
Ans- 5
5..a=4,b=3,c=-1
36=26+2a a=5
665. sin10.sin30.sin50.sin70=
1/8. 1/12. 1/16. 1/20
Ans- 1/2. cos 20.cos40.cos80= 1/2. 1/4.cos 60=1/16
yaha pe cos a x cos(60-a) x cos(60+a)= 1/4 cos3a
666. find the range
sin^4A + cos^4A
Ans- Eqn is 1-2sin^2A*cos^2A
=1-1/2(4sin^2Acos^2A)
=1-1/2*sin2A
4 min val sin2A Shud b max =1
Thus min is 1-1/2=1/2
667. if sin A = sin B and cos A = cos B, then sin ( A- B)/2 = ?
Ans- 0, A=B=45
laga do desi..:)

668. The length of a rectangle is increased by 20% and the width is decreased by 20%. The
area decreases by :
a. 0.8%
b. 1.2%
c. 4%
d. 8%
Ans- 4
669 In a triangle ABC, the lengths of the sides AB and AC equal 17.5 cm and 9 cm respectively.
Let D be a point on the line segment BC such that AD is perpendicular to BC. If AD = 3 cm, then
what is the radius (in cm) of the circle circumscribing the triangle ABC?
(1) 17.05
(2) 27.85
(3) 22.45
(4) 32.25
(5) 26.25
Ans- Option 5
cat 2008
use area of traingle = abc/ 4R ... where R is the raidus of the circumcirlcle
670. x+y=25 & x^2y^3+y^2x^3=25 then xy
Ans- +-1
bhai itna kyu kar rahe ho....2nd eq. mein x^2*y^2 common lelo.... (x^2*y^2)(x+y) = 25....ab
karo...
X^2y^2=1
So xy=+-1 ???
671. Ram starts working on a job and works on it for 12 days and completes 40% of the work.
To help him complete the work, he employs Ravi and together they work for another 12 days
and the work gets completed. How much more efficient is Ram than Ravi?
1. 50%
2. 200%
3. 60%
4. 125%
5. 100%
Ans- 2 times efficient so 100% MORE efficient....:D....chalo ek to sahi aya aaj
han bhai 100% hi hai....likha bhi tha ki jo Ravi 36 days mein karta hai vo Ram 18 days mein
karega..

672. The sides of a triangle are in the ratio 1 / 2 : 1 / 3 : 1 / 4 and its semiperimeter is 52 cm.
The length of longest side is :
a. 52 cm
b. 48 cm
c. 32 cm
d. 26 cm
Ans- 6/13*104 = 48
673. The average weight of 50 balls is 2 lbs. If the weight of the container be included, the
average weight will increase by 0.05 lbs. The weight of the container is :
a. 4.55 lbs.
b. 4.50 lbs.
c. 4.45.lbs.
d. 2.55 lbs.
Ans- a
674. A property dealer leaves 12900 sq yards area to be divided among- his 5 brothers, 4
daughters and 2 Sisters'. If each daughter recieves 4 times as much as each sister and each
brother gets 5 times as much as each sister, then how much did each daughter get ?
a. 200 sq. yards
b. 500 sq. yards
c. 320 sq. yards
d. Data inadequate
e. None of these
Ans- 1200 none of these.
675. If sin 2A + sin 2B = 1/2 and cos 2A + cos 2B = 3/2, then cos^2 (A - B) = ?
Explain please.
3/8
5/8
3/4
7/4
Ans- sin 2A + sin 2B is
2sin(A+B)cos(A-B)
and cos 2A + cos 2B is
2cos(A+B)cos(A-B)
dividing will get
tan(A+B) =1/3
a right triangle
cos(A+B) = 3/rt10
sibstitute in other equation. will get the ans

tan(A+B) =1/3
Using pythagoras theorem
adj side 3
opp side 1
Hypns rt10
so cos (A+B) =3/rt10
given 2cos(A+B)cos(A-B) = 3/2
so cos^2 (A-B) = 5/8
676.If x and y are the two digits of the number 653 xy such that this number is divisible by 80,
then x + y is equal to :
a. 2
b. 3
c. 4
d. 6
Ans- 6
65360 is the no
3x0 is not....its 53x, which is divisible by 8....right? we are asked to find the number divisible by
80....not by 8....so the second last three digits should be divisible by 8 as the last number is 0
677. If sin A sin B - cos A cos B + 1 = 0, then find 1 + cot A tan B ?
Ans-0
cosa cosb- sina sinb=cos(a+b) ; so cos(a+b)=1=cos zero;a+b=0 ;a=-b so cota=cot(-b)=-cotb=1/tanb ;so cota tanb=-1/tanb*tanb=-1 ............
hero.. take A = 135 and B = 225.. for the simple reason tht their sum shud be equal to -1.
678. The largest number amongst the following that will perfectly divide 101^100 - 1 is
1. 100
2. 10,000
3. 100^100
4. 100,000
Ans- 10000
x^n - a^n is divisible by x- a for any value of n wala hi :-)... bas thoda ek step aagey socho..

Questions..daalo..SPARTANS..its lunch time...sadupyog ho jaye..

679. If 3x-2y=11 & xy=12 find 27x^3-8y^3


Any desi 4 this?
Ans- simple sum hai re hero... jugad ki koi zarurat nahi.. sirf cube karo... terms adjust karo.. u
get the answer..
cubing the given eqn 27X^3 - 3.9x^2.2y + 3.3x.4y^2 - 8y^3 = 1331.. 27x^3 - 18xy(3x-2y) - 8y^3
= 1331...which is 27x^3 - 8y^3 = 1331 + (18 *11 *12)... khush isse aayega answer.... last time
maine orally kiya tha.... to sirf 18 *11 kiya * 12 karne bhul gaya... isse aayega tumhara answer...
bas kya hero B
27x^3-8y^3 = (3x)^3-(2y)^3....a^3-b^3 ka formulae lagao...values dalo...ans 3707 aayega
680. least value of 4cosec^2A + 9 sec^2A
Ans- 4cosec^2A + 9 sec^2A = 4 ( 1 + cot ^2 A ) + 9 ( 1 + tan ^2 A )
= 4 + 4 cot ^2 A + 9 + 9 tan ^2 A
= 13+ 4 cot ^2 A + 9 tan ^2 A = 13 + ( 2 cot A - 3 tan A) ^2 + 12
= 25 + ( 2 cot A - 3 tan A ) ^2....so for least value square should be zero, so least value = 25
iss sum me humne (2CotA+3tanA)^2 aise terms q nahi adjust kiye?...- ka hi q sahara lia?
@ no...if we write it as what u said, for the square to be zero...
2 cot A = - 3 tan A...=> tan ^2 A = - 2/3, which is not possible.......i think u asked about what i
have explained...
sorry cant understand hindi...only thoda thoda maloom..
681. Two vessels A n B containing 25 Litres each of Milk n Water resp. 5 Litr from A is taken n
poured into B, then 6 Litre from B is poured into A. Find Ratio of Water in A n B?
a. 4:5. b1:4 c.5:4 d. 2:3
Ans- A :Milk = 25 Water = 0
B : Milk = 00 Water = 25
5 litres of milk taken from A and poured in B now
A : Milk = 20 Water = 0
B :Milk = 5 Water = 25
6 litres(1 lit Milk & 5 lit Water) of mixture from B and poured in A.
So, Finally
A : Milk = 21 Water = 5
B : Milk = 4 Water = 20
Ratio of water in A & B is 5:20 = 1:4

682. If 3 sin A + 5 cos A = 5, find 5sin A - 3 cos A ?


a. 5, b. +/- 5, c. 3, d. =/- 3
Ans- put A = 0 ans -3
since minus and plus 3 are do different options and we hv to mark only one toh minus 3....
+- 3 is the correct answer
Squaring both and add
683. If sin ( 120 - A ) = sin ( 120 - B ), find the relation b/w A and B ?
1. A=B, 2. A=B & A - B = pi, 3. A=B and A- B = pi/2, 4. A=B and A+B = pi /3
Ans- simple...sin ( 120 - A ) = sin ( 120 - B ),means...
either 120- A = 120 - B....from that A =B
or 120- A = 180- ( 120 - B).....sin A = sin ( 180 - A )
so....120- A = 60 + B
60 = A +B......so option d follows
root3/2 cosa+1/2sina=root3/2cosb+1/2sinb , root3(cosa-cosb)=(sinb-sina) , (sinb-sina)/(cosacosb)=root3 , a+b/2=pi/6 , a+=pi/3 so option 4
684.a+1/a=1, then a^3=?
Ans- a+1/a = 1..a^2 - a + 1 = 0..a^3+1 = (a+1)(a^2 - a + 1)..thus a^3 + 1 =0..a^3 = -1
lo bhai ek aur easy method a+1/a=x hota hai toh a^3+1/a^3=x^3-3x
x=1 hence x^3-3x = 1-3 =-2 tht means a^3+1/a^3=-2 so a^3=-1
685. in an army camp ration is available for 100 soldiers for 10 days ,after 2 days 60 soldiers
joined .then how many more days will the remaining ration last?
Ans- 100*10 - 100*2 = 800........800 = 160 *X........So X = 5

686. if sin^2A = cos^3A,


value of (cot^6A- cot^2A)
0 1 -1 2
Ans- 1 sin^2A = cos^3A
ie tan^2A = cosA
Now substitute tan^2A in the other equation. will get the ans.
cot ^ 6 A = cos ^6 A/ sin ^6 A.... now sin^2A = cos^3A... sin ^4 A= cos ^ 6 A... this shud help... u
cant miss from here
It is (1-tan^4A)/tan^6A
= sin^2A/cos^3A
=1
687. The simple interest on a sum of money at 8% per annum for 6 years is half the sum. The
sum is
a. Rs. 4800
b. Rs. 2400
c. Rs. 9600
d. None of these
Ans- bas bhai iskaa ans nahi aa sakta..:)
688. A man invests a certain sum of money at 6% p.a. simple interest and' another sum at 7%
p.a. simple interest. His income from interest after 2 years was Rs. 354. One fourth of the first
sum is equal to one fifth of the second sum. The total sum invested was:
a. Rs. 2600
b. Rs. 2700
c. Rs. 2880
d. Rs. 2900
Ans- yups 2700, 1200+1500

689. if (a^2-b^2)sin@+2abcos@=a^2+b^2,
find value of tan@
Ans- desi batau... :-P
put a = 4 b = 3.. aur traingle banayo..
this is a generalised form... if a sinx + b cos x = rt.( a^2 + b^2 )... then tan x = a/b
divide all by a^2 + b^2...then it will be in the form sin^2 A + cos ^2 A = 1....one sin term and one
cos term is there....so
sin A = ( a^2 - b^2 )/ ( a^2 + b^2 ) and cos A = 2 ab / ( a^2 + b^2 )....
so tan A = ( a^2 - b^2 ) / 2ab.....
another example....3 cos A + 4 sin A = 5....
then divvide by 5, wll be 3/5 cos A + 4/5 sin A = 1...so cos A = 3/5 and sin A = 4/5 to get sin ^2
A = cos ^2 A = 1
690. When x5 - 5x4 + 9x3 - 6x2 - 16x + 13 is divided by x2 - 3x + a, the quotient and the
remainder are x3 - 2x2 + x + 1 and -15x + 11, respectively. The value of a is equal to:
a. 1
b. 2
c. 3
d. 4
Ans- x=1
without putting the value you can ans this question............you need to look at the constant term
......... a + 11 = 13 so a=2
691. The value of sin 12 sin 48 sin 54 ?
Ans- 1/8
uppar niche 2 se multiply karo 2sinasinb lagao
692. sin 47 + sin 61 - sin 11 - sin 25 = ?
Ans- sin a + sin b lagao..2-2 k group baanao
Ans is cos 7

693. sin x + sin y = root 3 (cos y - cos x) , then sin 3x + sin 3y = ?


Ans- 0
Take x=0, y=0
2sin(x+y)/2cos(x-y)/2
---------------------------- =root3
2sin(x+y)/2sin(x-y)/2
cot(x-y)/2=root3
x-y/2=pi/6
x-y=pi/3
x=y+pi/3
sin(3y+pi)+sin3y
-sin(3y)+sin(3y)
0
694. if a+b+c=11 and ab+bc+ca=20 then the value of expression a^3+b^3+c^3-3abc will be
Ans- a+b+c=11 a"2+b"2+c"2=121-3*20=61 so 11*61=671
695. If (cos A + cos B)^2 + (sin A + sin B)^2 = k cos^2(A - B /2), then the value of k is?
Explaination required........
Ans- bhai LHS ko expand karo.....ayega....2+2cosAcosB+2sinAsinB = 2+2(cosAcosB+sinAsinB)
= 2+2cos(A-B) = 2[1+cos(A-B)] = 2[2cos^2(a-b/2)] = 4*cos^2(A-B/2)
A = B = 0....ho gaya...k = 4
bhai koi values le lo koi bhi...best rahega 90, 0 kyuki calc asaan ho jayegi
696. The sum and the difference of two expressions is 5x^2 - x - 4 and x^2 + 9x - 10,
respectively, then their L.C.M. would be equal to:
a. (x - 1)
b. (2x - 3) (3x + 7)
c. (2x + 3) (3x + 7)
d. (x - 1) (2x - 3) (3x + 7)
Ans- put a+b= 5x^2 - x - 4 a-b = x^2 + 9x - 10, now let x= 2
you vl get a+b = 14 n a-b = 12
solve to get a and b... find out heir LCM .... check similarly in the options
put x=2,3,4.... any value

697. if x^45+1 is divided by x^5+1 then the remainder will be..


a.1 b.0 c.-1 d 2
Ans- 0
2/2
are simple hai ,, x^odd+y^odd open krte hai to (x+y)(x^odd-1-....)me aayega,,, means x+y se
devide hoga hi
698. If a^2 = (b + c), b^2 = (c + a), c^2 = (a + b); then the value of 1/(a+1) + 1/(b+1) + 1/(c+1) is
equal to:
a. 1
b. -1
c. 0
d. -[ 1/a + 1/b + 1/c ]
Ans- 1/3+1/3+1/3=1
A=b=c=2 desi :p
699. The traffic lights at three different road crossings change after every 48 sec., 72 sec. and
108 sec. respectively. If they all change simultaneously at 8 : 20 : 00 hours, then they will again
change simultanrously at :
a. 8 : 27 : 12 hrs.
b. 8 : 27 : 24 hrs.
c. 8 : 27 : 36 hrs.
d. 8 : 27 : 48 hrs.
Ans- lcm of 48,72,108=432 sec
432/60 = 7 min 12 sec
next time = 8:27:12 hrs .... option a
700. If sin x + sin y = 3(cos y - cos x) then sin 3x/sin 3y equals
1
-1
0
none of these
Ans- put 150 and 210 ..shud work :-)
Aapko strike kaise hua 150 & 210....kya trial & error method se
jaguad try karne ke liye.. aise values apne paas hai nahi jo woh eqn satisfy kare( like 30, 60,
45..)...isliye aise values liye ki mujhe zero aa jaye....
kaise bhai... wapas .. sin 450 = 1 sin 630 = -1... ???

701.

Ans- Let the radius of the circle with center x, y, z be X, Y, Z so nw ........X-Y = xy=6 , X-Z =
xz=7 , Y+Z = 9 ............sum this you will get ...2X =22 ......so X = 11 ..........Y = 5 , Z =4

xb=xc 6+yb=7+zc and yb+zc=9 yb=9-zc, so 6+9-zc=7+zc , zc=4 and yb=5, xb=6+5=11
702. x^2+2=2x
x^4-x^3+x^2+2=?
Ans- x(x^3-x^2+2)
x(x^3-2x+4)
x(x(2x-4)+4))
2x(x^2-2x+2)
2x(-2+2)
2x*0=0
703. if A, B, C, D are consecutive integers then the rt(ABCD + 1)= ?
(B+1)(C+1)
(A+1)(C+1)
BC -1
(A+B)(A+C)
Ans- BC-1

704. If y = sec^ theta + cos^ theta, theta is not equal to 0


then
y=0
y<=2
y>= -2
y not equal to 2
Ans- D
Put random angles....all will come more than 2....
705. if x+ y+z =0, then x^2 / yz + y^2/zx + z^2/xy =?
1-(xyz)^2 / x
2-x^2+y^2+z^2
3-9
4-3
Ans- 3..x+y+z = 0 then x^3+y^3+z^3 = 3xyz..!
3xyz/xyz = 3
706. x=4ab/a+b
a is nt equal to b
x+2a/x-2a + x+2b/x-2b =?
Ans- 2.....
By componendo and dividend
x/2a=2b/a+b ... apply c& d.... work out similarly for other term... u vl get the answer
707. if 0 degrees > alpha > 180 degrees
then sin alpha * cos cube alpha > sin cube alpha * cos alpha is valid for??
alpha between 0 and 45
alpha between 0 and 90
alpha between 45 and 90
none of these
Ans-a
use two values i.e 30 and 60 for alpha in above eq..30 secs
sin theta * cos theta ( cos ^ theta - sin ^ theta ) >0 hence three individual terms need to be
greater than 0 and cos theta shud be greater than sin theta whch is true when theta is between
0 and 45

i did it this way.....sin@.cos^3@>sin^3@.cos@ i.e. cos^2@ > sin^2@. i.e. cos@>sin@....now


this is valid for value of 0 to 45 only......i hope my concept is right....
708. when the circumference of a toy balloon is increased from 20 cm to 25 cm ,its radius is
increased by...
A.5 b.5/pie c. 5/2pie d.pie/5
Ans- c
709. x-y=(x+y)/7=xy/4
Value of xy?
Ans- 4/3
Now I think trigo ka hauaa khatam ho jaega coz ppl kisi na kisi tarah desi laga hi denge...and
answer bhi aa jaega....min effort se max result....thts wt I like abt the engineers they dont ask wt
the syllabus is....they ask when is the exam???
710. In a race of 600m. A can beat B by 60m and in a race of 500m, B can beat C by 25m. By
how many metres will A beat C in a 400m race ?
Ans- 58
711. In a circle, triangle ABC with an abtuse angle B is inscribed. The altitude AD of the triangle
is tengent to the circle. The side BC has a length 12cm and the segment BD has length 4cm .
What is the area of the triangle ABC?
64
60
48
96
Ans- ad^2=db*dc it gives ad=8 ...area of abc =(8*16/2)-(8*4/2)=48
figure me ky BC ko extend kiya hain
ye obtuse engle hai to perpendicular BC pe dalne ke liye extend karna padega na
712. If sin A + sin B = 1/4 and cos A + cos B = 1/2 , then tan (A + B /2) = ??
How to solve?
Ans-
use half angle formula
lllrly cos A + cos B = 2 cos (A+B/2)cos (A-B/2)
sinA + sin B = 2 sin(A+B/2) cos (A-B/2)

713. If sin 2theta = cos 3theta and theta is an acute angle, then sin theta equals to
(rt5 -1)/4
-(rt5 - 1)/4
(rt5 + 1)/4
(-rt5 - 1)/4
Ans- a
sin 18 =rt5-1/4
how u find d value of sin18
learn by heart
714. D n E r d midpoints of AB n AC of trngl ABC, BC is produced to any point P, DE,DP,EP r
joined, Then
a)trng PED=trn BEC
b)trg ADE=BEC
C)BDE=BEC
D)PED=1/4 ABC
Ans- d
PED ka base DE hoga jo 1/2*BC hai .........iska hight bhi half hoga ABC se
DE is 1/2 BC is ok bt what abt height
point p ko khiska ke BC ke mid point pe lao (area of triangle PED will not change). Then it is
easy.
arBEC=2 arDEP=2arADE=2arBDE=1/2arABC
Kindly post any ques from trigonometry which came in Tier 1 exam that can only be solved
using concept of class xi and xii...let me check if SSC really giving ques based on concepts of
class XI and XII or we r just wasting our tym..:)
715. A train overtakes 2 persons who r walkng in d same directn in which d train is runng, at
2kmph n 4 kmph n passes them cmpltly in 9 n 10 secs respctvly.. length of d train ?
Ans- length = (22-2)*5/18*9 = 50 m

716. If A = tan 6 * tan 42* and B = cot 66 * cot 78 (the angles are in degrees)
then
A =2B
A=B/3
A=B
3A=2B
Ans- A=B
Divide A and B.. you vl get tan 6 * tan 42 * tan 66 * tan 78
now use tan A * tan(60-A) * tan(60+A) = tan 3a
A = B..sin cos me chng karke 2sina sinb n 2 cosa cosb lagao..answer aa jayega
717. The side of a triangle ABC are in A.P. The perimeter of the triangle ABC is 72cm. If the
smallest side is 18cm. Then find the circumradius of triangle ABC ?
10
15
9
12
Ans- sides are 24, 18 and 30....right angled triangle....15 is the ans
area of triangle = abc / 4R
718. If the interior angles of a polygon are in A.P with common difference of 5degrees and the
smallest angle is 120 degrees, then the number of sides of the polygon is
9 or 16
9
13
16
Ans- 9
largest exterior angle 60* now proceed sum of AP = 360 with common difference = -5
No of side 16 not possible
719. If tan ^ theta = 2 tan^ A +1
then cos 2theta + sin^ A equals
-1
0
1
none of these
Ans- Q=45,A=0, gud one....desi rox

720. ABC is a right triangle, rights angled at A . A circle is inscribed in it. The lengths of the two
sides containing the right angle are 6cm and 8cm. Find the radius of the incircle.
Ans- 2
radius of incircle = ar of triangle / semi-perimeter..ho jayega..!
721. if (x-a-b)/c + (x-b-c)/a + (x-c-a)/b = 3 , the possible value of x will be ?
1- c-b+a
2- a+b+c
3-1
4-(a+b+c)^2 / 2bc
Ans- 2
in this ques 3 individual terms r equal to 3 so i just assume each of them equal to 1 x-a-b/c=1
when a=b=c and x=a+b+c
722. a^4 + a^2 b^2 + b^4=8
a^2 + ab + b^2=4
Find ab?
options:1,2,3,4..:)
Ans- lo aab to kam au asaan 4 ,3 ho nahi sakta 1 condt satisfy kar raha hai
ab=a^2b^2=1
(a^2+b^2)^2-a^2b^2=8
16+a^2b^2-8ab-a^2b^2=8
8ab=8
ab=1
723. If rt{(x-a)/(x-b)}+a/x = rt{(x-b)/(x-a)}+b/x, b not equal to a, then the value of x?
b/(a+b) ab/(a+b) 1 a/(a+b)
Process?
Ans- one side would be (a-b)/x = {(x-b)-(x-a)} / {rt(x-a)* rt(x-b)}
then x = {rt(x-a)* rt(x-b)} .........nw square it and solve
724.DESI.....
x^2 + y^2 + 1/x^2 + 1/y^2 = 4
x^2 + y^2=?
Ans- 2
desi(x=y=1)

725. sin^2A+cos^2B=2 find


Tan^3A+sin^5B=?
2,1,0,-1
Ans- A = 90* and B = 0*
exactly..question galat h phir..!
1+1/4rt2
kaise nikal diya bhai..?
maine sin^2a+cos^2b=1 leke kr dala ,,, ye 2 kaise hoega yr,,simple hai typo error hai
By logic A=B=45
726. a/1-a + b/1-b + c/1-c = 1
1/1-a + 1/1-b + 1/1-c = ?
Ans- a=0,b=0,c=1/2..so 4
a/1-a =0 lo..b/1-b =0 lo and c/1-c =1 lo..isse a=b0 and c=1/2 aa jayega
Ye strike kaise hua 0,0,1/2
a/(1-a)+1=1/(1-a).. strike..ki..jarurat..hi..nhi..h
727. a+b=2c
a/a-c + c/b-c = ?
A=2 B=0 C=1
desi..ki..jarurat..nhi..thi..isme..waise..hi..ho.rha..h..a-c=c-b, put the value of b-c=c-a and get the
answer 1
baat aisi hai aise ques dekhte hi desi yaad aata hai jyadda tym nahi lagaa ...RHS mein 2c dekar
hint de rakhi hai ki c ko idhar udhar karna hai
M khud pehle desi hi lga rha tha par sahi values dimag m nhi Ayi toh socha solve hi kr lete h..20
sec me ho gya..desi k layak b nhi tha ye ques..:p
728. An arc AB of a circle subtends an angle x radians at the centre O of a circle. Given that the
area of sector AOB is equal to square of length of arc AB, find x.
Ans- 1/2

729. The value of sin78 - sin66 - sin42 + sin60 is?


1/2
-1/2
-1
1
Ans- -1/2
sin78 - sin42 + sin60 - sin66
2cos60*sin18 + sin60 - sin66
Options are wrong
bhai galti se sin 60 ki jagah sin 6 le liya toh -1/2 aaya h..!
730. Raw paper pulp contain 75% of water. Drying process takes of 70% of the moisture. What
is the percentage of water in dried pulp ?
1-47.4
2-52.6
3-55.3
4-42.8
Ans- 7suppose weight 100g ........so water 75g .......70% water dried out so .........nw water
22.5g .........so dried pulb weight is 22.5+25= 47.5g ...........% of water now (22.5 * 100)/ 47.5 =
47.4%
731. sin (B - C) cos (A -D) + sin (C -A) cos (B - D) + sin (A - B) cos (C - D) = ?
Ans- 0
formula lagao 2 sina cosb = sin(a+b) + sin ( a-b)

732. Two vessels A and B contain mixtures of spirit and water. A mixture of 3 parts from A and 2
parts from B is found to contain 29% of spirit and a mixture of 1 part from A and 9 parts from B
is found to contain 34% of spirit.find the percentage of spirit in A and B
option (a) 35,25,(b) 40,20, (c)25,35, (d)50,50
Ans- c
for short way in this question if you go through the option you will have to work with only one
option 25 ,35 ..........as other option is out of range ........

733. cos 20 cos 100 + cos 100 cos 140 - cos 140 cos 200 = ?
How to solve?
= 1/2 ( -3/2 ) = -3/4 answer..!
COS A + COS B lagao..2 cos (a+b)/2 cos(a-b)/2..aa jayega..!
bhai thnx to u for such valuable questions..put some more if u have..!
-3/4 {(2 cos60 cos20) - cos20}
-3/4 (cos20 - cos20)
734.

Ansssss

Ans- 1st ka 1:4


2- 49-x

735. A lump of two matels weighing 18g is worth Rs.87. If their weights be interchanged, it
would be worth Rs. 78.60. If the price of one metal is Rs. 6.70 per gram, find the weight of the
other metal in the mixture ?
1-10
2-8
3-9
4-11
ax+6.7(18-x) = 87 n 6.7x+a(18-x) = 78.6..solve
Waise yaar yahan trick ye hai ki 36 kg of mixture will equal ratio of weight and its price is 87+
78.60 = 165.60 ...so 4.60 per gram is the cost of lump ........now so price of second will be
2.50.......now easy
Price of 2nd 2.5 kaise likha
lump ka cost 4.60 per gram hai ..this is the mixture of equal ratio of 6.70 and second mixture
..............now second mixture ka price nikal lo
x*y+(18-x)6.7=87
(18-x)*y+x*6.7=78.6.... add them
18y+120.6=165.6
18y=45
y=5/2=2.5
2.5x+120.6-6.7x=87
4.2x=33.6
x=8 gm
736. if A works alone,he would take 4 days more to complete the job than if both A and B
worked together.If B worked alone,he would take 16 days more to complete the job than if A
and B work together.How many days would they take to complete the work if both of them
worked together??
Ans- 1/(x+4) + 1/(x+16) = 1/x..........x^2 = 64..........so x=8

737.The price of commodity decreases by 30%. By what percent should the consumption be
increased in order to maintain the expenditure just 2% less than the original expenditure?
a. 28%
b. 32%
c. 42 6/7%
d. 40%
Ans- d

738. A and B start simultaneously from the same point and run in the same direction on a
circular track. If their speeds are 4.5kmph and 6kmph respectively and length of the track is
1.5km find the time taken by them to meet anywhere on the track for the first time.
a. 1hr
b. 2hr
c. 3hr
d. 1/2hr
Ans- a
739. a^2 - bc = 3
b^2 - ca = 4
c^2 - ab = 5
then value of a+b+c = ?
Ans- a^3 + b^3 +c^3 -3abc= 3a + 4b +5c iske baad kya karna
Ans is 1/2 samjh me aane par plz EXPLAIn kar do bhai
740. cos@.cos2@.cos3@.cos4@.cos5@=? find the value
1/11, 1/12, 1/13, 1/14
Ans- 1/13
741. What is the least no. Which is perfect square and contains 3675 as its factor?
Ans- 3675*3
tell the logic bro!
Factor kr lo
742. The greatest no. less than 900, which is divisible by 8, 12 and 28 ??
Ans- 840
743. ABCD is a parallelogram in which AB = 6root3 cm and BC = 6cm and angle ABC = 120
degree. The bisector of angles A,B,C,D form a quadrilateral PQRS . The area of PQRS in sq cm
is
a 18root(2-root3)
b 18root3
c 36/root3
d 18(2-root3)

744. A ladder 25 mtr long is placed against a wall with its foot 7 meters away frm the foot of
wall.how far shud the foot be drawn out so that the top of ladder may come down by half the
distance of total distance if the foot is drawn out..
6
8
8.75
None
Ans- b
25^2=(24-x/2)^2+(x+7)^2...solve for x using options
745. 3(sinx-cosx)^4+4(sin^6x+cos^6x)+6(sinx+cosx)^2
11
12
13
14
Ans- 13 by putting x =90
mereko ek baat samazh nahi ati....tum logon ko pata kaise chalta hai ki kaunsa value dalna
hai.......
Bachpan se usi cheez me man laga hai jiska answer jaldi aa jaye bina mehnat kiye
Chahe IT industry me code copy paste karna ho yaa ques solve
Iss ques ka videshi sol de rkha h 15 lines ka aur Yaha hamare desi logo be 40 sec approx me kr
diya...isliye toh MERA BHARAT MAHAN...;)
Tabhi toh kehte hai jo baat desi me hai woh vilayati me kahan....
45 se13
746. If cosec -sin = a^3 and sec -cos = b^3, then what is the value of a^2*b^2(a^2+b^2)?
Ans- 1..by putting thetha as 45
It is 1
cos^2A/sinA = a^3---- 1
sin^2A/cosA = b^3
cos A =sin ^2A/b^3
sub. this in eqn 1
will get
sinA = ab^2
cosA = ba^2
sin^2A +cos ^2A =1
will get
a^2*b^2(a^2+b^2) as 1

747. Three jars contain alcohol to water solution in the ratio 3:5 , 1:3 , and 1:1. If all the three
are mixed, what will be the ratio of alcohol to water in the final solution ?
1- 3:5
2-3:4
3-4:5
4- can not determined
Ans- d
748. x+y+z=1 , xyz=-1 , xy+yz+zx=-1
than x^4+y^8+z^12=???
Ans- 3..desi lagao..
1,1,-1 put karo to gt th answer
749. if x(3- 2/x)=3/x , x is not equal to 0
then the value of x^2 + 1/ x^2
Ans- x-1/x=2/3
x^2+1/x^2=4/9+2=22/9
750. which among the following is greatest
(5)^1/2 , (11)^1/3 , (123)^1/6
Ans- (5)^1/2
shortest way apart frm taking lcm of roots?
isse shortest kya bacche ki jaan loge..:P
751. The average of 2, 7, 6 and X is 5 and average of 18, 1, 6, X and Y is 10 . then Y is
1-10
2-15
3-20
4-30
Ans- 20

752. Cos 2theta + 2cos theta is always


> -3/2
<= -3/2
>= -3/2
none of these
Ans- Eqn can b wrtn as 2(cos^2@+cns@+1/4)-1-1/2
=2(cos@-1/2)^2-3/2
so min wud b -3/2
60 rakho bas
i put theta as 120..
Hw do u knw ki 60 rakhna hai??
theres no guarntee of that
Yeh desi thodi hi hai yeh toh zabardasti method hai....
90 se start kiya phir 60 aur kaunsaa range nikalne ka ques ssc pochegii
Normal tarike se karo....Isme desi kismat se laga hai....Not convincingly....
Bracket value will always be positive toh it wud drive the second negative term towards positive
side
Hence min value is -3/2
753. asin@+/-bcos@ ki min value = - (a^2+b^2) n max value = + (a^2+b^2)..!
Ans- ratta mar lo 1 question jarror aana h..:)
a sin @ - b cos @ - min value is not - rt ( a^2 + b^2 ), i think
as far as i knw this is minimum..!
754. lo bhai ssc type qestion
rt3sin@-cos@ find the min n max value
Ans- 2,-2
asin@+/-bcos@ ki min value = - (a^2+b^2) n max value = + (a^2+b^2)..!
bahi plzz tell me is tarah ke question konsi book me he n aap konsi book se dalte he ye
question
bhai aab sabko rat gyaa hai aab nahi denge :)
bhai SSC me kaafi Q's traditional hote h..aa sakta h!
bhai mere pas 1986 ki ek trigo ki sadi hui kitab hai, bro uska naaam kya he

755. how to solve this typr of questionsif, x + 1/x = 3 then wht is the value of
x^5 + 1/x^5 ?
Ans- + 1/x = 3. squaring both sides. x^2 + 1/x^2 + 2 = 9. x^2+1/x^2 = 7.....(1) cubing both sides.
x^3 + 1/x^3 + 3(3) = 27. x^3 + 1/x^3 = 18.....(2).....MULTIPLY (1) & (2) (x^2+1/x^2)*(x^3+1/x^3)
= 126. x^2 * x^3 + x^2 * 1/x^3 + 1/x^2 * x^3 + 1/x^2 * 1/x^3 = 126 x^5 + 1/x + x + 1/x^5 = 126. so
x^5 + 3 + 1/x^5 = 126. so x^5 + 1/x^5 = 123
756. FCI main
The average value of the numbers 15, 21, 32, 35, 46, X, 59, 65, 72 should be greater than or
equal to 43 but less than or equal to 44. then the calue of x hould be
1- 42 <= x <= 51
2- 43 <= x <= 50
3-42 < x < 49
4-43 < x < 50
Ans- a
345+x/9=43 to 44 option a i think
757. sec 20 - 3cosec 20
Ans- -4
758. suggest the method to get
sin5
sin10
sin20
sin25
sin35
sin40
sin50
Ans- sabse easy method..GOOGLE IT..:P
759. Which among the following is the greatest
rt7 + rt3 , rt5 + rt5, rt6 + 2 ??????
Ans- rt5+rt5 is largest...squarin is the shortest way..unless u know rt3,45,6,7 values
shortest way- when sum of all dis term(without taking root) is equal, lyk 7+3=10, 5+5=10.....d
term whose multiplied value is maximum is biggest n minimum is lowest

760. if X = 17^4 and Y= 14*16*18*20 then


1- x > y
2- x<y
3-x=y
4-none
Ans- 1
761. x^4+1/x^4=194
calc (x^3+1/x^3)(x^2+1/x^2)(x+1/x)
Ans- 52*14*4 = 2912?
yes 2912 is crrct...self made ques
762. find the value of sin 22 1/2 degree
Ans- (rt2-1)/2rt2
sin (@/2) = {(1 - cos @)/2}..cos@ = 45
Ans is 1/2rt un2-rt2
763. a shopkeeper sells 1 choclate@ rs 1 each, you can exchange 3 wrappers for 1 choclate.If
you have rs 15.How many choclates can u totally get.?
Ans- 22
pehle din 1 wala bar denge....fir next day jab vo aayega to 1 le lenge usse wapas...aur 2 wala
denge..........fir on 3rd day usse... 1 de denge jo wapas liye.... 4th day usse 2 aur 1 le lenge aur
4 wala de denge.... 5th day... fir 1 de denge....aur 6th day....1 leke 2 ..and last day..fir se 1 de
denge :P
15+5+1+1=22
Make two cuts on the gold bar such that you have the following sizes of bars.
1/7, 2/7 and 4/7. For convenience sake, I would just refer to the bars as 1, 2 and 4.
At the end of Day 1: Give Bar 1 (You- 2 and 4, Worker- 1)
At the end of Day 2: Give Bar 2, Take back Bar 1 (You- 1 and 4, Worker- 2)
At the end of Day 3: Give Bar 1 (You- 4, Worker- 1 and 2)
At the end of Day 4: Give Bar 4, Take back Bar 1 and Bar 2 (You- 1 and 2, Worker- 4)
At the end of Day 5: Give Bar 1 (You- 2, Worker- 1 and 4)

At the end of Day 6: Give Bar 2, Take back Bar 1 (You- 1, Worker- 2 and 4)
At the end of Day 7: Give Bar 1 (You- Empty, Worker- 1, 2 and 4)
764. SSC CGL 2011 main
The square root of
[{(0.75)^3 / (1- 0.75) } + ( 0.75 + (0.75)^2 +1 ) ] is
1-1
2-2
3-3
4-4
Ans- 3/4*3/4*3/4*4+3/4+9/16+1
27/16+12/16+9/16+16/16
64/16=4
root of it =2
(a^3-b^3) ka formula lagega
765. SSC CGL 2011 MAIN
If the length of a chord of a circle at a distance of 12cm from the centre of is 10cm, then the
diameter of the circle is ?
1-13
2-15
3-26
4-30
Ans- 26
767. Four men can do work in 15 days. If a man left the work after 5 days and again joined after
5more days, The remaining three work continuously till the end of work. How many more days
than the estimated, it takes to complete the work?
...i thought this is worth sharing
Ans- 16 days 6 hours or 1.25days or 1(1/4)
one man = x units per day
60x = total work
units got completed in 1st 5 days = 5*4x = 20x
in next 5 days = 5*3x = 15x
in next 5 days = 5*4x = 20x
in next 1 day 6 hours work will get comp.
total time taken = 16 days 6 hours

768. Equal quantities of a 1:5 and 3:5 of milk and water solution are mixed together. What is the
final ratio of water and milk in the resultant solution?
1-13:35
2-4:10
3-5:8
4-35:13
Ans- 1/6+3/8 : 5/6+5/8
13:35
@ above all ...................this was a easy question Quite SSC type ..................bt everyone ans is
wrong...............ans is 35: 13 ..........here question ask ratio of water to milk ............SSC does
this trick by asking diameter instead of radius .........I have done many mistake like this in past
.....so thought to share it with you

769. In a cage there are sparrows, parrots and doves in the ratio 3:7:5. if the number of parrots
was more than the number of sparrows by a multiple of both 9 and 7. What is the minium birds
in the cage ??/
1-945
2-630
3-238
4- none
Ans- 3 + 5 - 7= 1 >>> which corresponds to (9*7) so 63 * ( 3+5+7) = 945
770. There are two bars of gold-silver alloy; one piece has 2 parts of gold to 3 parts of silver,
and the other has 3 parts of gold to 7 parts of silver. If both bars are melted into a 8-kg bar with
the final ratio of 5:11 (gold to silver), what was the weight of the first bar?
(A) 1 kg
(B) 3 kg
(C) 5 kg
(D) 6 kg
(E) 7 kg
Ans- 2/5x + 3/10(8-x) : 3/5x+7/10(8-x) = 5/11
x+24 : 56-x=5/11
11x+264=280-5x
16x=16
x=1 kg

771. x+1/x=3, x^18+x^12+x^6+1=?


Ans- if x+1/x=3
now (x+1/x)^3=(3)^3
x^3+1/x^3=0
now, x^18+x^12+x^6+1
=x^12(x^6+1)+1(x^6+1)
=(x^12+1)(x^6+1)
=(x^12+1)x^3(x^3+1/x^3)
=0 ans.
but if X+1/X = 3 then kaise hoga
772. Find the maximum value of cos^2 A - 6 sin A cos A + 3 sin ^2 A + 2 ?
Ans- 4-cos2a-3sin2a....
diff
sin2a=3cos2a
4-10cos2a ..... a=90
gives 14 max value
we are getting...4 - ( cos 2a + 3 sin 2a)
min vvalue of cos 2a + 3 sin 2a = rt 10..
so max value of function = 4 + rt 10..
773. a figure showing a set of concentric squares. If the diagonal of the innermost square is
2 units, and if the distance between the corresponding corners of any two successive squares is
1 unit, find the difference between the areas of the eighth and the seventh squares, counting
from
the innermost square.
Ans- The diagonal of the innermost square is 2 units. The
diagonal of every successive square would increase
by 2 units (since corners are one unit apart). So the
diagonal of the 7th square = 14 and that of the 8th
square = 16. Areas of the 7th square =
1/2 *14^2
and that
of 8th square =
1/2 *16^2
, i.e. 98 and 128 respectively.

Hence, the difference in their areas


= (128 98) = 30 sq. units
774. Fresh grapes contain 90% water while dry grapes contain 20% water. What is the weight of
dry
grapes obtained from 20 kg fresh grapes?
a. 2 kg b. 2.5 kg c. 2.4 kg d. None of these
Ans- 20 kg fresh grapes contains 2kg of pulp.
In dry grapes 20% water
2 kg is the 80% pulp in dry grapes
so 2*100/80
2.5 kg
775. The points of intersection of three lines 2X + 3Y 5 = 0, 5X 7Y + 2 = 0 and 9X 5Y 4=
0
a. form a triangle b. are on lines perpendicular to each other
c. are on lines parallel to each other d. are coincident
Ans- correct ans is -------------- d. are coincident
776. There are two concentric circles with centre O. PQRS is a square inscribed in the
outer circle. It also circumscribes the inner circle, touching it at points B, C, D and A. What is the
ratio of the perimeter of the outer circle to that of polygon ABCD?
Let the radius of the outer circle be x = OQ
Hence, perimeter of the circle = 2*pie*x
But OQ = BC = x (diagonals of the square BQCO)
Perimeter of ABCD = 4x
Hence, ratio = 2*pie*x/4*x = pie/2
777. Navjivan Express from Ahmedabad to Chennai leaves Ahmedabad at 6.30 a.m. and
travels at
50 kmph towards Baroda situated 100 km away. At 7.00 a.m. Howrah-Ahmedabad Express
leaves
Baroda towards Ahmedabad and travels at 40 kmph. At 7.30 a.m. Mr Shah, the traffic controller
at
Baroda realizes that both the trains are running on the same track. How much time does he
have to
avert a head-on collision between the two trains?
a. 15 min b. 20 min c. 25 min d. 30 min
Ans- 20 min
At 7.30 a.m., Navjivan Express is at 50 km from A at
the same time, Howrah-Ahmedabad Express is at 20
km from B.

Hence, distance between the trains at 7.30 a.m. is


30 km.
Relative speed = 50 + 40 = 90 kmph
Hence, time left = 30/90 or 1/3 hr
or 20 min.
778. if 3 tan A tan B = 1, then find 2 cos ( A + B) ?
a. cos ( A-B), b. cos ( A +B ), c. 1, d. none
3 sin a sin b =cos a cos b ......so 2 cos (a+b)=4 sin a sin b ...which is equal to option a
779. rt 3 cosec 20 - sec 20 = ?
a. 1, b. 2, c. 4, d. none
Ans- 4
Eqn ko sin cos k form me kr lo. 2 se divide kro .take rt3/2 as sin30 and cos60
Use sin (a+b) formula then
780. sin 65 + cos 65 = ?
a. rt 2 sin 20, b. rt 2 cos 20, c. rt 2 tan 20, d. none
Ans- x^2=1+sin130=1+cos40=2cos^2 20
or x= rt2 cos20
781.The remainder when 7^84 is divided by 342 is
a. 0 b. 1 c. 49 d. 341
Ans- 1
1 7^3 343
782. Find max value of 3cosx+5sin(x-30)
1.5
2.rt19
3.rt17
4.3
Ans- 2.rt19
x=90 5rt3/2

783.The greatest number, that divides 43, 91 and183 so as to leave the same remainder in
each case, is
(a) 9 (b) 8
(c) 4 (d) 3
Ans- find hcf of positive difference of numbers
ie find hcf of (91-43) , (183-43) , (183-91) ie 48,140,92 ie 4
784. The maximum value of 12sin@ - 9sin^2@ ?
@=theeta
Explain please......
Ans- 12sin@-9(1-cos2@)/2
12cos@-9sin2@=0
12cos@=18sin@cos@
2=3sin@
sin@=2/3
12*2/3-9*4/9
8-4= 4
785. when (67^67+67) is divided by 68 the remainder is:
a) 1 b)63 c)66 d) 67
Ans- u can write 67^67 as (68-1)^67 wchigh gives -1 n remaining terms divisible by 68 ....now -1
67 & 66
binomial theoram
(-1)^67+(-1) / 68
-2/68
68-2=66
786. The perimeter of a triangle is 8cm and one of the side is 3cm. Area of the triangle is
maximum when the other two side are ?
1- 5/2 , 5/2
2- 3 , 2
3 -3/2, 7/2
4 none
Ans- bhai 5/2 and -5/2 one side is x and another side be 5-x then area^2= 4x1x(8-x0x (3+x)
then differentiate it you will get x=5/2 and -5/2 max and min don't get afected by squaring.
s=8/2=4
area=root s(s-a)(s-b)(s-c) ,here s=4 let a=3 then b+c=5 now put these value in area formula
then diff to find max value

then area will be 3


but when sides are 2,2,3
then root 4*(4-2)*(4-2)*(4-3)
=root 4*2*2*1
=4
which is max area
787. If the angles of a triangle are in ratio 1:2:3 then corrosponding sides are in ratio ?
Ans- 1 : rt3 :2
Use sine rule and take angles as 30,60,90 one liner hai....
1Lrii3:2
a/sinC=b/sinB= c/SinC _______________........a:b:c =SinA:SinB:SinC

788.IN a triangle ABC coaA/a=coaB/b=cosC/c and a=2 then area of triangle is


a-1 b-2 c-root3/2 d-root3
Ans- d-root3
it can be found by cos side method a=b=c=2 well put the value of cos A= b^2+c^2-a^2/2bc

789. the side of a triangle are in a ratio 6 : 8: 9 thus the triangle will be ?
1- acute
2- obtuse
3-right angle
4 -none of
Ans- (9^2<8^2 +6^2) its acute
Desii drew rough diagram & got ans :P
use cosine rule..
c^2 = a^2 + b^2 then triangle will be right angle ...........if c^2 < a^2 + b^2 triangle will be acute
.............if c^2 > a^2 + b^2 triangle will be obtuse ........

790. The angle of elevation of a jet plane from a point A on the ground is 60 degree.After a flight
of 15s, the angle of elevation changes to 30 degree.If the jet plane plane is flying at a constant
height of 1500rt3m, find the speed of the jet plane in km/hr??
Ans 200m/s
200 m/s..dasvi kaksha ki yaad aa gyi..tab kiya tha ye ques..wo bachpan ki yaadein..:)
3000 m/s
3000m/s sec bhai plane hai kya hai
791. If x^1/3 + y^1/3 = z^1/3, then (x + y z)^3 + 27 xyz
is equal to
(a) 0 (b) 1
(c) 1 (d) 27
Ans- 0..go desi...:)
don't need desi here.
0 ... x=z=1,,y=0
792. The interior angle of a regular polygon exceeds the exterior angle by 132* . The no of the
side ?
Ans- 15
explanation plz in equation term is it (2n-4)x90/n - n(n+1)/2= 132
793. If X= rt( rt5 + 1)/rt(rt5 - 1) then find the value of x^2 + X - 1 ?
a.rt5 b.0 c.2 d. -rt5
Ans- multiply with rt ( rt 5 + 1 ).. x = rt 5 + 1 / rt 4 = rt 5 + 1 /2 ...then solve..
bro aise hi to kia tha to rt5 + 1 aaya hai but ans rt5 dia hai plzzzz explain that bro
794. The ratio of the area of a sector of a circle to the area of the circle is 1 : 4. If the area of
thecircle is 154 cm2, the perimeter of the sector is
(a) 20 cm (b) 25 cm
(c) 36 cm (d) 40 cm
Ans- pir^2=154, r= 7 & l=rq=7*pi/2=11
perimeter of sector 2r+l = 14+11=25
bhai ek quarant banega central angle 90 as ratio of area is 1/4...L=length of arc=radii*central
angle

795. find Tan(45+a) + Tan(45-a) = ?


a.2tan2a b.tan2a c.sec2a d.2sec2a
Ans- take a=15....d aayega
a=0 badi easy calc aur agar option d se check karna shru kiya toh 5 sec
a=0 se to 2-3 sec mein hogya....:-) 15 liya tha to 10-15 sec lag gaye the
796. Desi
tanx-cotx=a and cosx+sinx=b find (a^2+4)*(b^2-1)^2
Ans- 4
Take x=45, soln 1 line ka hoga
by desi put x is 45 degree so a is 0 n b is rt2 put in eqn n got 4
(b^2-1)^2=4sin^2xcos^2x
tan^2x+cot^2x+2 =a^2+4
(tanx+cotx)^2=a^2+4
(sin^2x+cos^2x / sinxcosx)^2=a^2+4
a^2+4=1/sin^2xcos^2x
by multiplying,,,, ans got 4
desi ki aadat dalo :p
i did it in max 1 min,,,deshi vidheshi sb krna pdta hai
Jaha 5sec me ho jaye 1min waste krne ki kya jarurat h
797. In a trianglen Find the value of tana/2*tanb/2 + tanb/2*tanc/2 + tanc/2*tana/2 = ?
Ans- 1 take a=b=c=60
its also a property
798. find the value of 4SinA*Sin(60-A)*Sin(60 + A) ?
a.cos3A b. CosA c. Sin3A 4.SinA
Ans- sin3A by desi...A=30
Put 0 and 60 only c will satisfy..:)
bro u.p. Board 10th mathematics ka sum hai.
sin3A a property

799. the side of one equilateral triangle equals the altitude of a second.what is the ratio of their
areas.
Ans- 3:4
800. The simple interest on a certain sum ofmoney for 4 years at 4 percnet per annum exceeds
the compound interest on the same sum for 3 years at 5 percent per annum by Rs. 57. Find the
sum
Ans- Rs. 24,000/1.5 min lag gaye woh toh 21^3 ki value pata thi
jsut remember 21^3 = 9261.. :-P
[3r^2/100+r^3/(100)^2]
801. Whats d diffrnce bw 'possibility' and 'probability' ?
Ans- "probability" is the degree of measurement of the "possibility"
802. if sin @= -12/13 and pie <@<3pie /2 then the value of sec @
a.13/5 b.-13/5 c.-12/13
Ans- pyth triplets aur pandit bhadri....
-h/b
- 13 / 5
yeah 5,12,13 triplet
803. 1/6 + 1/12 + 1/20 + .........+ 1/90 is equal to
1-0.8
2-0.7
3-0.9
4-0.75
Ans- koi formual nahi hai bhai... its simply... (1/2 -1/3) + (1/3 -1/4)... (1/9-1/10)
1/2*3+1/3*4+1/4*5+.....+1/9*10
(1/2-1/10)=4/10=0.4
ans will be 0.4 ............YE SSC CGL ka question ho gaya jaha sare option wrong hai

804.The measure of an angle which is 28 more than its complement?


Ans- x plus x -28 equal to 90
so x is 59
805. a/a-b +b/b-c +c/c-a =6 find b/a-b +c/c-b +a/c-a
Ans- 3 hga agar c/b-c h toh..Q thik h..i think it shud be c/b-c nt c/c-b?
a/(a-b) main 1 minus karo second ka first term aayega .........and so on
806. If x+y+z=5 and xy+yz=zx=3 wat is d largest value dat x can have.
If X will be greatest then Y and Z will be equal so let Y = Z = K then X+2K = 5........also XK +
K^2 + XK = 3 then solve it it will give K = 3 and 1/3 .........3 is not possible so through 1/3
............X = 13/3.........
807. If x= 2+rt3, then the value of (x^6+x^4+x^2+1)/x^3 = ??
Ans- 56
divide the eqn by x^3, nw x=2+rt3 so 1/x=2-rt3, nw put these values in the eqn usin
x^3+1/x^3=(x+1/x)^3-3(x+1/x)
808. If sin@ + sin^2@ + sin^3@ = 1, then what is the value of
cos^6@ - 4cos^4@ + 8cos^2@??
Ans- 4
Lengthy h.sin@+sin^3@= cos^2@,square both side n put it in eqn.then change al d terms in
cos
809.In a triangle Tan(A+B) Equal to ?
a. tanC b.cotC c. - CotC d. tanC
Ans- - tan C
810. If x2 + y2 2x + 6y + 10 = 0, then the value of
(x2 + y2) is
(a) 4 (b) 6
(c) 8 (d) 10
Ans- x=1,y=-3..ans is 10
811. If (a^2-b^2)sinx+2abcosx=a^2+b^2, then
tanx=?
Ans- a^2-b^2/2ab

812. A and B entered into a partnership with the ratio of their capitals 3 : 5. A is paid salary for
managing the business and rest of the profit is divided in proportion of their capitals. If the ration
of their total incomes is 7 : 9 , express A's salary as a percentage of the total profit.
Ans- 3/8,5/8.....now7/16,9/16.........let salari is x....so...(1-x)*5/8=9/16...=>x=1/10...
813. sin A sin (B - C) + sin B sin (C - A) + sin C sin (A - B) = ?
Which formulae to apply?
Ans- 0
desi
814. if Sin17 = x/y , then the value of sec17 - sin73 ?
Ans- x^2/(y rt y^2 - x^2)
815. a man takes a loan of rs. 240000 to build a house at the rate of 12% p.a. After 1 year he
completes the house and rents it for rs 5200 per month. After how many years of taking the loan
he can repay his debt by the rent he gets?
Ans- 240000+(240000*0.12*(x+1))= 5200*x*12...=>x=8 yrs
816. 3sinx + 5cosx =5,then value of (3cosx - 5sinx)^2 is
a)2 b)3 c)4 d)5 e)9
Ans- correct ans is 9.........................let 3cosx - 5sinx = a and we have 3sinx+5cosx.........add the
square s of both the equation.
817.Four students tried to find the sum of the first 21 primes, and only one student was correct.
Patsy got 709, Leena got 711, Sam got 712, and Diana got 713. Which student was correct?
Ans- i did an experiment,,,,
2+3 = 5
2+3+5=10
2+3+5+7=17
2+3+5+7+11=28
means odd no of prime no's total must be even....
hehe... bahi options dekte to bhi chal jata.. baki sab 0dd hai except for one..

818. If 3sin@+5cos@=5 find 5sin@-3cos@


Ans- desi ..............3sin@+5cos@=5 is possible when @ =0* so ............5sin@-3cos@ = -3
asinq+bcosq=rt(a^2+b^2-c^2) and asinq-bcosq=-c
c=3 &-c=-3
correct ,both 3 and -3 r possible
819. If x+1/x= 2cos@ find x^3+1/x^3
Ans- 2cos3@
cubing both side in left side u will get the formula of cos3a
cos3x = 4cos^3x - 3cosx
820. The system of equations
a1x + b1y + c1 = 0
a2x + b2y + c2 = 0
(i) is consistent with unique solution, if a1/a2 is not equal to b1/b2
i.e the lines represented by equations are not parallel.
(ii) consistent with infinitely many solutions, if a1/a2 = b1/b2 = c1/c2
i.e. the lines represented by equations are coincident.
(iii) inconsistent, if a1/a2 = b1/b2 is not equal to c1/c2
i.e. the lines represented by equations are parallel and non-coincident.A + B + C = 180
A + B + C = 180
sin 2A + sin 2B + sin 2C= 4sinAsinBsinC
cos 2A + cos 2B + cos 2C = - 1 - 4cosAcosBcosC
tanA + tanB + tanC =tanA tanB tanC
sinA + sinB + sinC = 4 cosA/2 cosB/2 cosC/2
cosA + cosB + cosC = 1 + 4sinA/2 sinB/2 sinC/2
sin^2 A + sin^2 C = 2 + 2 cosAcosBcosC
CotBCotC+CotCCotA+CotACotB=1
cotA/2+cotB/2+cotC/2=cotA/2cotB/2cotC/2
TanB/2TanC/2+TanC/2TanA/2+TanA/2TanB/2=1

821. Two watermelons of the same quality is being sold. The first is 60cm in cirucmference and
the other is 50cm. The first is one and half times as costly as the second. Which of the two is
profitable to buy ?
Ans- BTW .......ans is -1 ..........
agreed bt have u ever heard watermelon ka circumference hota hai...it shud have been surface
area
ryt sir u r 100% ryt,....
822. If sin (pie cos@) = cos (pie sin@), then sin 2@ is?
@=theeta.
Explain please.......
Ans- sin (pie cos@) = cos (pie sin@),
cos ( 90 - pi cos @ ) = cos ( pi sin @ )
90 - pi cos @ = pi sin @
pi ( sin @ + cos @ ) = 90 = pi /2
sin @ + cos @ = 1/2..
squaring, 1 + 2 sin @ cos @ = 1/4
1+ sin 2 @ = 1/4 ........sin 2 @ = - 3/4,....
I did it this way.........picos@ = pi/4, cos@ = 1/4. pisin@ = pi/4, sin@=1/4. sin2@ = 2sin@cos@
= 2*1/4*1/4 = 1/8.
piecos@=pi/4..cos@=1/4... sin@=1/4
sin2@=2sin@cos@=2*1/4*1/4=1/8
sin ( pie cos@ ) = sin (90+piesin@)
piecos@=90+pisin@
piecos@-pisin@=pi/2
cos@-sin@=1/2
cos^2@+sin^2@-2sin@cos@=1/4
sin2@=1-1/4=3/4
dono shi hai
823. in a right angle triangle, the product of two side is equal to the half of the square of the third
side i.e hypotenuse. One of the acute angle must be ?

Ans- 45 degrees...gud question


824. find greatest ratio a:b ................ 5 - ( 3a - b )^2 how to solve this yar ? aur yai sab R.D
sharma mai nahai hai..SSC ka quest. hai yai... :(
Ans-1:3
3a-b=0 kar do..
825. find the smallest positive value of x for which tan(x+100)=tan(x+50)*tanx*tan(x-50)
Ans- 30
tan(x+100)/tan(x-50)=tan(x+50).tanx
sin(x+100). cos(x-50)/sin(x-50)*cos(x+100) =
ain(x+50).sinx/ cos(x+50).cosx

826.if x+ 1/x = 3, then wht is value of x^5+1/x^5


Ans- 123
take cube nd square nd solve further for result
827.the value of expression..
x^4 -17x^3 + 17x^2 - 17x + 17 at x=16,
options:: a-0 ,b-2, c-3, d-1
plzzzz... also mention ur method....
Ans- did it by unit digit concept
u can take common then solve x^3(x-17) + 17x(x-1) + 17..put x = 16..-1 should be the answer..!
828. cos20 cos40 cos60 cos80 = ?
Ans- 1/16..1/4*cos60*cos60..!
829. sin 2A + sin 2B + sin 2C=
Cos 2A + cos 2B + cos 2C =0
When A,B,C are not equal

830. the interior angle of a polygon are in A.P . the smallest angle is 120 and the common
difference is 5 degree . find the number of sides of polygon.
Ans- the sum of external angles of a polygon will be 360.....so largest external angle = 60 as the
smalest internal angle = 120
then the external angles will be 60,55,50......and its sum = 360
apply the formula of AP, n/2 * ( 2 a + ( n-1) d ) = 360....d = -5....
find n ...it will be 16 or 9
side ka do value hoga 9 aur 16 but 16 possible nahi hai jab t16of ap nikalenge to angle 195 ho
jayega jo ki 180 se jayeda hai so 9 is ans.
831. ABCD is a cyclic quadrileteral whose diagonals intersects at P. If angle DBC = 70* and
angle BAC = 30* find angle BCD ?
Ans- 80
832. If tan A = 1/5...tan B = 1/ 239, then tan ( 4 A - B ) = ?
Ans- 1
find tan2a then tan4a
833. if x=(2rt24)/(rt3+rr2), then value of {(x+rt8)/(x-rt8)+(x+rt12)/(x-rt12) is?
how to do this in quick method
Ans- 2
first of all x=2root 24/root3 +root 2 ko simplify karo then you get 12root2 8root3..........................x+root8/x-root8 applying c and d ...................................x+root8 +xroot8/x+roo8-x+root8=......x/root8 similarly above one x/root 12 then put it.
{(x+rt8)/(x-rt8)+(x+rt12)/(x-rt12} ..............write as x / rt8 + x/ rt12 nw solve
x/rt8 ko componando and divendendo use karne per (x+rt8)/(x-rt8) hoga na.........i
834. in a triangle ABC, AB^2+ AC^2 = 200 cm^2. Meadian AD = 10cm. find BC
Ans- Using apolonius theorem.
200 = 2(100+ BD^2)
confusing then BD comes to be zero means traingle doesn't exist.

835. In a regular polygon if the radius of its circumcircle is equal to its side, then the number of
side of the polygon is ??
1-5
2-6
3-8
4-none
Ans- 6
if you want to make circum radius equal to the side of the polygon then the interior angle must
be 120* which is with hexagon
836. In two circles arcs of equal length subtend angles of 60 degree and 75 at their centres,
then find the ratio of radii.
Ans- 5:4
use theta =l/r

837. sin(x+y)=cos3(x+y) then tan2(x+y)=??


Ans- 1
cosA=Sin(90-A)
90/(3+1) = 22.5 = (x+y)
so tan( 2 * 22.5) = tan 45 = 1
sir.....directly 90/(3+1) kaise likha?
sin (1A) = cos (3A) >>> so 1+ 3 = 4...
sin (1A) = cos (3A) >>> so 1+ 3 = 4...
838. The wheel of a railway carriage is 4ft. in diameter and makes 6 revolutions in a second
how fast is the train going?
Ans- 24.pi units/sec
839. find the value of tan20 + tan72 + tan88
Ans- we al ready know tht.. tan A + tan B + tan C = tan A. tan B. tan C
not always...but here..
its true when
tan(a plus b plus c) equal to 0

840. Assuming the distance of the earth from the moon to be 38400km . and the angle
subtended by the moon at the eye of a person on the earth to be 31' find the diameter of teh
moon.
841. If 12 sin@ - 9 sin^2@ attains its maximum value at @ = A, then value of sin A ?
@=theeta
Explain.......
Ans- dervicative of sinx =
cosx... and sin^2 x = 2.sinx .cosx.. so
differenicating the above eqn and
equating to zero... gives.. 12cosx 18sinx.cosx = 0/.. on solving we get sin
x = 2/3..

842.In a triangle ABC, tanA + tanB + tanC = 6, then cotA cotB cotC = ?
Ans- 1/6
tanA + tanB + tanC = tanA tanBtanC
in a triangle tanA + tanB + tanC = tanA tanBtanC always..and one more useful
formula..cotA/2+cotB/2+cotC/2=cotA/2cotB/2cotC/2 in triangle
843.The maximum value of sin^2(120 + @) + sin^2(120 - @) is?
@=theta.
sin ( 120 + @) = sin 120 cos @ + cos 120 sin @ = rt 3 cos @ / 2 - sin @ / 2.....similarly sin (
120- @ ) = sin 120 cos @ - cos 120 sin @ = rt 3 cos @ / 2 + sin @ / 2...
there fore ...sin^2(120 + @) + sin^2(120 - @) = ( rt 3 cos @ / 2 - sin @ / 2 ) ^2 + ( rt 3 cos @ / 2
+ sin @ / 2 )^2 =
= 2 ( (rt 3 cos @ / 2)^2 + ( sin @ /2)^2)
= 2/4 * ( 3 cos ^2 @ + sin ^2 @)
= 1/2* ( 1 + 2 cos ^2 @ )
for maximum value cos must be 1....so max value = 1/2* ( 1 +2) = 1/2*3 = 3/2

844. Sin^ 2 75-sin^ 2 15=


Ans- root3/2
sin ^2 C - sin ^2 D = sin ( C+D ) sin ( C-D)
and cos ^2 C - sin ^2 D = cos ( C+D) cos ( C-D)

Ans- 2
7 + 4 rt. 3 = (2 + rt.3 )^2
3 + 8 * ( 2 + rt. 3 ) = 19 + 8. rt. 3 = ( 4 + rt.3 ) ^ 2
845. The value of tan20 + tan40 + root3 tan20 tan40 ?
How to solve this?
Ans- rt.3
use tan (a +b)
tan(20+40)=(tan20+tan40)/(1-tan20tan40)==>rt3=(tan20+tan40)/(1-tan20tan40)..so tan20 +
tan40 + root3 tan20 tan40=rt3

846.If tan (A - B) = 1 and sec (A + B) = 2/rt3, then the smallest positive value of B is?
25pie/24
19pie/24
13pie/24
11pie/24
Ans- 19pie/24
a-b=pi/4 cos (a+b)=rt3/2=cospi/6 =cos(2pi-pi/6)=cos11pi/6 ,a+b=11pi/6 solve both equation
847. If cot (A+B) = 0, then sin (A+2B) is equal to?
sinA
cos2B
cosA
sin2A
Ans- sinA
a=b=45
Aaj ek mast walla idea aya hai sabki help chaiye :-)
848. If cos P=1/7 and cos Q=13/14, where both P and Q are acute angle, then the value of P Q is?
Ans-cos(a-b) lagao..!
P-Q = 60..!
849. If sin A = 336/625 where 450deg<A<540deg, then sinA/4 =
a) 3/5 b) -3/5 c) 4/5 d) -4/5
please explain this one.....
Ans- 4/5
Desi--it will be in 2nd quad so b and d cant be possible moreover Sin A= 0.5 smthng and A/4
will be between 112.5 and 135 so value must be greater than 1/rt2 so 3/5 nt possible
850. If x cos@ = y cos(@ + 2pie/3) = z cos(@ + 4pie/3), then value of 1/x + 1/y + 1/z
is?@=theta
Ans- take @=60..xcos60=ycos180=zcos300==>x/2=-y=z/2==>i/y-1/y=0
Jugar @=0 then graph of cos .

851. If A + B - C = pie, and sin^2 A + sin^2 B + sin^2 C = x sinA sinB cosC , then value of x?
Ans- 2
take A=B=90 and C=0......
cos 60

852. cos^2A+cos^2(A+120)+cos^2(A-120) = ??
Ans- 3/2
853. Maximum value of 5 cosx + 3 cos(x +
pie/3) + 3 ?
7
10
4
9
Ans- 10
854. cos^2 45 - sin^2 15 =?
Any short cut except by putting value?
Ans- 1/2-(1-cos30)/2=cos30/2=rt3/4
cos60.cos30=rt3/4
cos(a+B) Cos (a-b)= cos^2a- Sin^2b
855. if tanA= - 1/3 & tanB = - 1/2 thn
find A+B
Ans- tanA + tanB / 1- tanAtanB = [-1/3 + -1/2] / (1-1/6) = -[1/3+1/2]/(5/6) = -[5/6]/[5/6] = -1. Lo
bhai. -1 hi to aa raha hai.
hmmm...to shi to hai yaar...135 hi to aayega....bt srry it will comes in 2nd qnrt....bt answr shld be
135...
(tan^-1)(-1)=135deg
yeah tan inverse(-1) so 135degree

856. If tan A= x+1 and tan B= x - 1, then 2 cot (A - B) = ?


Ans- 2x^2/(x^2-1)
x^2 is the right answer.
Ans=0 if we put x=0,A=45,B=-45
applying dedi method most welcome.
But if u started to apply it everywhere, gradually u will loose the habit of solving any problem
properly and that will hav a negative impact on ur perfection.
857. if @ is a positive angle and 2 sin@+15cos^2@=7 then find the value of 2cot@??
Ans- 2sin@+15(1-sin^2@)=7
2sin@+15-15sin^2@=7
15sin^2@-2sin@-8=0
age solve kru to kaise,,,
bhai aage middle term split karlo 15sin^2@-12sin@+10sin@-8=0..ab common lo aur ho gya..!
15sin^2@-12sin@+10sin@-8=0
3sin@(5sin@-4)+2(5sin@-4)=0, so sin@=4/5 cos@=3/5 cot@=3/4 2cot@ = 3/2
858. the equation (1+n2)x^2 + 2ncx + (c^2-a^2)= 0 will have equal roots, if
a) c^2= 1+a^2 b) c^2= 1- a^2 c) c^2= 1+ n^2 + a^2 d) c^2 = (1=n^2)a^2
Ans- D = 0 ..b^2 = 4ac rakho..
c^2=a^2(1+n^2)
859. tan15 + tan30 + tan15 tan30 =?
any short cut except putting value?
Ans- One desi, in case of tan cot, answer always 1 as (30+15)=45 and tan45/ cot45=1
simple hga yar..tan 15 = 2-3 n tan 30 = 1/3..ho gya khatam..!

tan15=tan(45-30)=1-1/rt3 / 1+1/rt3=rt3-1 / rt3+1 = 2-rt3


tan30=1/rt3
2-rt3+1/rt3 + 2-rt3 /rt3 = 2rt3-3+1+2-rt3 / rt3 = 1
tan(45=30+15)=(tan30+tan15)/(1-tan30tan15) hence ans is 1

860. If tan (A+B) = x and tan (A - B) = y, value of 2A = ?


Explain please......
Ans- Clear tan (A+B+A-B)=tan2A
2a=tan-1x+tan-1y=tan-1( x+y / 1- xy)
tan2A=x+y / 1- xy
861. cos(n + 2 )x.cos(n+1)x + sin(n+2)x. Sin(n+1)x = ??
Ans- cosx..cos(A-B) ban jayega..!
cos(nx+2x-nx-x)=cosx
862. sin163.cos347 + sin73.cos167 = ?
sin(90+73)cos(180+167) + sin 73 cos167 = -cos73cos167+sin73sin167 = -cos (73+167) = -cos
240 =
But given question is wrong. It would be sin167 , not cos167
to kahe bacche ki jan le li,,,ghnte bhr se
863. tan20.tan40.tan60.tan80=?
Ans- 3
864. sinA + cosA=2 has how many soln..?
1) one 2) two 3) zero 4) infinite
Ans- Zero sol
Both sine and cos can never attain value of one together
865. cos(A-B)= 1/2, & sin (A+B)= 1/2,,
find the value of A..?
Ans- 105
a-b=60
a+b=30
b=45
a=105
866. last year mains ques: if 3tanA +4 =0 ; pie/2 <A> pie, tanA= 3/4
find value of 2cotA - 5 cosA + sinA
Ans- 23/10..??shld be -ve same value...

867. least value of 9cosec^2x + 9sec^2x


Ans- 36
sub. x=45
no need to go 4 maxima minima here..simply solve n it'll become 36/sin2x..for least value sin2x
shud be max which will be 1..so 36 will be the answer..!
18 + (3tanx - 3cotx)^2 +18
So min value 36
868. xsin^3A + ycos^3A = sinAcosA,
xsinA=ycosA, cosA is not equal to zero then x^2 + y^2 is
Ans- 1
869. find the rational number whose decimal form is 1.3bar(45).
Ans- 1+345-3/990= 1+342/990=1 57/165
bhai 57/165 = 19/55........if v r talking abt rational the hcf of num n den shud b 1.....
870. If sinA + sinB = a and cosA + cosB = b, then cos (A + B) =?
Ans- b^2-a^2/ b^2+a^2
SinA+ SinB / Cos A+CosB = a/b..................... cotC/2 = tan(a+b)/2= a/b and then cos(A+B)= 1tan^2(a+b)/ 1+ tan^2(a+b)
871. If tan (pie cos@) = cot (pie sin@), then cos [@ - (pie/4)] = ?
@=theeta
Ans- pie cos @ = pi/4
cos@=1/4
pie sin @ = pi/4
sin@ = 1/4
cos(@-pie/4)= cos@cospi/4-sin@sinpi/4= 1/4rt2+1/4rt2=1/2rt2
value of ratios tan and cot are equal. so angle must be pi/4.
872.If tabA - tanB= x and cotB - cotA= y, then cot(A - B) = ?
Ans- (x+y)/xy

873. what should be added in 4913*4914*4915..so that the final sum becomes a prefect cube..
(a)9826 (b)4914 (c)9828 (d)4913
Ans- hehe its very easy ques,one liner,break 4913=4914-1 n 4915=4914+1
4913 * 4914 * 4915..............(4914 - 1) * 4914 * (4914 + 1)
4914 { (4914-1)(4914+1).............4914 *{(4914^2 - 1^2)
4914^3 - 4914........so if we will add 4914 it would yield the required ans. :)
874. a+b+c=1..ab+bc+ca=1/3 then a:b:c=?
Ans- a=1/3,b=1/3,c=1/3..so ans is 1:1:1
875. A B C are three points on a circle. the tangent at A meets BC produced at T.
<BTA=40degree <CAT=44degree. the angle subtended by BC at the centre of the circle is
a) 84
b)92
C)96
d)104
Ans- 104
876. What is the remainder when 1044 * 1047 * 1050 * 1053 is divided by 33?
A. 3
B. 27
C. 30
D. 21
E. 18
Any shortcut method for this???
Ans- do such qs by this
choose multiple of 33 nearest to given qs
33*32=1056
difference of 1044 . 1047 . 1050 . 1053 with 1056 is 12,9,6,3
we can write these directly as,,& further reduce this
12*9*6*3 / 33
54*36/33
21*3/33
63/33
remainder = 30 is ur ans
I got the solution but not shortcut..:)
You can solve this problem if you know one basic rule about remainders.
Let us say a number x, divides the product of A and B.

The remainder that you will get will be the product of the remainders when x divides A and when
x divides B.
Using this information,
The remainder when 33 divides 1044 is 21.
The remainder when 33 divides 1047 is 24
The remainder when 33 divides 1050 is 27 and
The remainder when 33 divides 1053 is 30.
The net remainder is 21*24*27*30.
However, as the value of 21*24*27*30 is more than 33, the final remainder will be the remainder
when 33 divides 21*24*27*30.
When 33 divides 21*24, the remainder is 9.
Similarly when 33 divides 27*30, the remainder is 18.
The final remainder is the remainder when 9*18 is divided by 33 = 30.
877. A sum of Rs 86, 700 is to be divided among A, B and C in such a manner that for every
rupee that A gets, B gets 90 paise and for every rupee that B gets. C gets 100 paise. Bs share
will be
(a) Rs. 26, 010
(b) Rs. 27, 000
(c) Rs. 28, 000
(d) Rs. 28, 090
Ans- question is wrong...the correct question is: A sum of Rs 86, 700 is to be divided among A,
B and C in such a manner that for every rupee that A gets, B gets 90 paise and for every rupee
that B gets. C gets 110 paise. Bs share will be
878. if sec @+tan@=x,then sec@ is
Ans- sec^2 + tan^2+2sectan=x^2
sec^2+sec^2-1+2sectan=x^2
2sec^2+2sectan-1=x^2
take sec common and put x and solve u ll get (x^2+1)/2x
sec^2@-tan^2@=1 so sec@-tan@=1/x-------(1) sec@+tan@=x.....(2) by solving equations we
get 2sec@=x+1/x or sec@=(x^2+1)/2x
879. A man sells an article at a loss of 5%. If he had bought it at 20rs more and sold it for
Rs.115 less, he would have incurred a loss of 40%. Find CP of the article.
a) 500 b) 540 c) 480 d) 520
Ans- for convenience we ca go thru options...... 100---95-----120----72---23----115----100*5=500. .

880. When a number is divided by 8 or 7 it leaves reminder 1 but when the same number is
divided by 9 the reminder is 5. What will be the reminder when it is divided by 72 ????????
Ans- when the number is divided by 8 or 7 it leaves a remainder 1. So the number is in the form
of 56k + 1 (56 being the LCM)
Now given that when the number is divided by 9 the reminder is 5.
So 56K+1-5 should be a perfect multiple of 9.
Substitute values for K by trial and error until you get a multiple of 9 and you will find that the
number is 113.
Divide it by 72.
i just want to add one line to your solution ............So 56K+1-5 should be a perfect multiple of 9.
...............54k + 2k - 4 when divided by 9 leavs no reminder so ............2k - 4 =0 , k=2 ..........so
the no is 56*2 + 1 = 113
881. 3 pipes a b and c can fill cistern in 6 hrs after working together for 2 hrs ,c is closed and a
and b fill in 8 hrs .Then find the time in which the cistern can be filled by pipe c.
Ans- 12
882. A person starts multiplying consecutive positive integers from 20. How many numbers
should he multiply before the will have result that will end with 3 zeroes?
A. 11
B. 10
C. 6
D. 5
Ans- 20-25 = 6 numbers
883. (963 + 476)2 + (963 - 476)2 = ?
(963 x 963 + 476 x 476)
A. 1449 B. 497
C. 2 D. 4
E. None of these
Ans- c.2
(a+b)^2 +(a-b)^2=2(a^2+b^2) ...kaash tier2 mein har question aasaan hai tho! :-)

884. The difference of two numbers is 1365. On dividing the larger number by the smaller, we
get 6 as quotient and the 15 as remainder. What is the smaller number ?
A. 240 B. 270
C. 295 D. 360
Ans- B.270
x-y=1350 , x=6y+15
885. If n is a natural number, then (6n2 + 6n) is always divisible by:
A. 6 only B. 6 and 12 both
C. 12 only D. by 18 only
Ans- b.
considering n=1
886. Diff.of sq of 2 no is 37 than sum of their sq.??
Ans- 19^2 + 18^2 = 361 + 324 = 685....:)
887. Sarah needs to make a cake and some cookies. The cake requires 3/8 cup of sugar and
the cookies require 3/5 cup of sugar. Sarah has 15/16 cups of sugar. Does she have enough
sugar, or how much more does she need?
A. She has enough sugar.
B. She needs 1/8 of a cup of sugar.
C. She needs 3/80 of a cup of sugar.
D. She needs 4/19 of a cup of sugar.
E. She needs 1/9 of a cup of sugar.
Ans- c..cup..size..let..1600
888. ABC is a right triangle right angled at B. AD and CE are the two medians drawn from A and
C respectively. If AC = 5 cm and AD = (3rt5)/2 cm, find the length of CE?
Ans- 2rt5
let ab=y,bc=x....25-x^2=(45-x^2)/4------->x^2=55/3....... so y^2=20/3..now cal CE...
889. What is the value of
sin2 sin 4 sin 6 ...... sin 88/cos 88 cos 86 cos 84 ...... cos 20??
a) 0 b) 1 c) 2 d) 4
Ans- saare cos wale change ho jaega sin me or vice versa...
Sin(90 - @) = cos @
sab cancel ho jaenge so ans 1....

890.

Ans- these formulae are applicable in dis ques???


If 2x - 1/3x =4, then tthe value of (27x^3 - 1/8x^3)
nhi,,iska,,,ye bnta hai...
ax-1/by = c ;
than a^3x^3 - 1/ b^3x^3 = (bc/a)^3+3bc/a
if
ax+1/by = c ;
than a^3x^3+1/b^3x^3 = (bc/a)^3-3bc/a
enjoy....

891. A ship is moving at a speed of 30 km/hr. To know the depth of the ocean beneath it, it
sends a radiowave which travels at a speed 200 m/s. The ship receives the signal after it has
moved 500 m. The depth of the ocean is
(A) root143/2 km
(B) 12 km
(C) root6 km
(D) 8 km
iska ans with sol ???
Ans- A ans ko simplify karo...its just greater than 6km...and d answer which we get =rt(6000^2250^2) imtr is also the same after simlification..
892. If SecX + CosX = 3 then find tan^2X - sin^2X = ?
a.9 b.4 c.5 d.13
ssc ques. i m getting -1 ans?????
Ans- tan^2x-sin^2x=sec^x-1-1+cos^x=sec^2x+cos^2x-2
as sec^2x+cos^x=7.....
ans = 7-2=5
bro by squaring Sec^2x + cos^2x = 7 so Sec^2x =6 + sin^2x now put it in 2nd eqn so tan^2x Sec^2x -6 so ans is 5 bro u know maine -1 kyun nikala q k maine 3 ka square karna bhul gaya
tha silly mistakes

893.

Ans- Sry i guess m wrong.. it shud be "b"..


A is 3/4 at @=45 which is less than A at @= 60 or 30 i.e. 13/16
894. If 3 sinA + 4 cosB = 6 ,
4 sin B + 3 cosA = 1
Then what is the value of angle C ?
Ans- sin(a+b)=1/2..=>a+b=30 or 150....=>c=150 or 30...

895. If x sin3@ + y cos3@ = sin @ cos @ and x sin @ - y cos @ = 0.find the value of x2 + y2 ?
Ans- put y=xtan@ and put it in eq i and then square and add both the equation.
answer toh 1 nikal gya..lekin desi fail ho gya..:(..@=30 lene se answer 1/4 aa rha hai..
..wapas..dekho.@=30 lene se answer..1..aa..rh..h

896. a boy is cycling such that the wheels of the cycle are making 140 revolution per minute.If
the diameter of the wheel is 60 cm ,calculate the speed per hour with which the boy is cycling.
Ans- 2.pi.30.140 in a min ..... speed = 2.22/7*.30*140=264 /60*18/5
=15.84 km/h

897. 40 men can complete a work in 40 days .they started the work together.but at the end of
each 10th day,5 men left the job.THe work would have been completed in
56 2/3 days
53 1/3 ''
52 ''
50 ''
Ans- 40*40=1600=400+350+300+250+200+15*x...=>x=100/15=6
x=100/15=6+2/3...ans 50+6+2/3...
one man can complete in = 1600 days
10 day work= 1/4 ... remaining work = 3/4
10 day work= 10*35/40*40= 7/32 ...remaing =3/4-7/32=17/32
10 day work = 10*30/40*40=3/16..remaining = 17/32-3/16=11/32
10 day work= 10*25/40*40=5/32..remaining = 11/32-5/32=6/32
10 day work= 10*20/40*40=4/32..remaining=6/32-4/32=1/16
one make can do a work in 1600 days than 1/16 of work will be done in x days by rest 20
workers
m1.d1 . w2 = m2.d2.w2
1600*1/16=15*x*1
x=100/15=20/3=6 2/3 days
total days = 50+6 2/3 = 56 2/3 days
898. The line which is parallel to x-axis and crosses the curve y=rootx at an angle of 45' is ?
899. Shyam can do a job in 20 days, Ram in 30 days and Singhal in 60 days. If Shyam is
helped by Ram and Singhal every 3rd day, how long will it take for them to complete the job? (1)
12 days (2) 16 days (3) 15 days (4) 10 day
Ans- 3 days work = 1/5. so 15 days to complete :)
900. aur ek sum jo 1st evening ke paper main aaya tha..
( 1 + 876542 * 876544) / ( 876543 * 876543)
Ans- 1+N/D=1/D+N/D
In this case 1/D is too small approx.0 & N/D will be 1.
1/D+N/D=0+1=1
ye to tuccha qs hai usse ... 1+(a-1)(a+1)/a^2
1+a^2-1/ a^2=a^2/a^2=1...........
evening wale ese hi 120 avg marks nhi la rhe,,,lutiya dubo di ssc ne,,baki sb shifts ki
,,,,grrrrrrrrrrrr

901. The line which is parallel to x-axis and crosses the curve y=rootx at an angle of 45' is ?
902. lo..mera..contri1...:)
1.The difference between 3 years simple interest and compound interest is given by (P * R2
*(300+R))/100^3
903. There is a fan with 3 baldes with 120* to each other whose central circular disc has an area
of 3pie cm^2 and a blade is (20-rt3)cm long. If the tips of the blade are joined so as to form an
equileteral triangle, then what will be its area?
1- 100cm
2- 300rt3
3-(900+9pie)
4-3pie + 300
Ans- Radius of circular disc vl be rt3.... length of median of tr = 2/3of x = 20.. therefore x = 30
now calc legth of each side as a*rt3/2 = 30
u vl get a=20rt3
Area = rt3 *20rt3*20rt3/4 = 300rt3
you can directly put 1/2*20*20*sin120* ..........this is the area of one small triangle .....then
multiply it by 3
904.The average of the runs made by Raju, Shyam and hari is 7 less than that made by shyam,
hari and kishore. if the number of kishore's run is 35, what is raju's run.
Ans- 14
solve the equations.
R + s + H/3 + 7 = S +H +35/3
905. In a kilometre race, A gives B a start of 50 metres and still beats him by 50 metres. On the
other hand, if A gives B a start of 20 seconds, he beats him by 5 seconds. How long does A
take to run the course?
250 seconds
200
180
225
Ans- 225
see... the time rqd for B to cover 900 mtrs is the same as the time req to cover 1000 mts.. so the
ratio of their speed is A:B :: 10 :9..and sin distance is the same... time ratio will be 9:10.. now in
the second statement... its given tht A saves 25 seconds as compared to B ,,,which wud be
25*9 :25 * 10 = : secs

906. A container has 80 L of milk. From this container 8L of milk was taken out and replaced by
water. The process was further repeated twice. The volume of milk in the container after
that...how to this quest.
Ans- 58.32
80*(9/10)^3
ifinitially its x litre....n y litre is replaced with y litre water .....n process is repeted n times the
liquid at last =x(1-y/x)^n

907. side AB of rectangle ABCD is divided into four equal parts by points x, y, z. The ratio of
area of triangle XYC to area of rectangle ABCD.
Ans-1/8
xyz triangle ka base 1/4 AB hai iska area 1/8AB*BC hoga ......AB*BC= area of rectangle
Theorem of triangles and parleogram on same base and betwen same paral lines.

908. if G is the centroid and AD, BE, CF are three medians of triangle ABC with are 72 cm sqr.
then the area of triangle BDG
Ans- 12
simply the concept is lyk 6 equal triangles will be formed. am i r8?
Yes.

909.4x=sec@ and 4/x= tan@ then 8(x^2-1/x^2) is plz explain.....


Ans-
x = sec@/4 ....1/x = tan@/4 ......now put
Square both eqn and subtract one frm other

910. if a = (2+rt3) / (2-rt3) and b = (2-rt3) / (2+rt3) then the value of a^2+b^2+ab ????????
Ans- 195
a=7+4rt3 b=7-4rt3 so put this value in (a+b)^2-ab

911. The value of (3+2rt2)^-3 + (3-2rt2)^-3 is ;


189 180 108 198
Ans- 198 is ans................ye (1/a + 1/b)^3 ..ka equation lagalo .............1/a+1/b = 6 hoga
(a+b)^3 + (a-b)^3 = 2a(a^2+3b^2)
bhai maine a= 3 and b=2rt2 rekha so iska num numerator becomes 2a^3+6a(b^2) and den me
1 so ans 198

Math xprts plz make a file of tricky ques that ssc can give, where we can use such jugaad desi
methods...
it will b better if there r separate files for algebra, trigo..

912. x=997, y=998, z=999 then x"2+y"2+z"2-xy-yz-zx=?


I m not askng d ans here, askng d quickest way to solve dis sums
Ans-Gud ques
Use x, x+1, x-1 and expand
terms cancel ho jaengi
equation can be written as ...............1/2*{(x-y)^2 + (y-z)^2 + (z-x)^2 }............now solve it ans will
be 3
x"2+y"2+z"2+xy+yz+zx agar aisa hoga to it wud be 2*(x^2+y^2+z^2) - 3 ... aur idhar last digit
concept use kar sakte fir
1/2*{(x-y)^2 + (y-z)^2 + (z-x)^2 }............ is also easy as (x-y)^2 in easy to calculate
........equation will be 1/2*{-1^2 + -1^2 + 2^2} = 6/3
913.
If P takes
speed
of Pt time
and R
then
is =Q
will
be
(t+40)
will
=1000/800
take
/ t .............second
t+20=and
5/4 R
.......(t+40)/t
will case
take t+40.......as
When
= 5/4.....
time is
distance
constant
is then
constant
ratioratio
of the
of

Ans- 74

914. P can give Q a start of 20 seconds in a kilometer race. P can give R a start of 200 meters
in the same kilometer race. And Q can give R a start of 20 seconds in the same kilometer race.
How long does P take to run the kilometer?
200 seconds
240 seconds
160 seconds
140 seconds
Ans- 160
If P takes t time then Q will take t+20 and R will take t+40.......as distance is constant ratio of
speed of P and R is = (t+40) / t .............second case When time is constant then ratio of the
speed of P and R will be =1000/800 = 5/4 .......(t+40)/t = 5/4.....
915. If 2-cos^2@=3sin@cos@
and sin@#cos@
find the value of tan@.?
Ans- divide both sides by cos^2@, 2sec^2@-1 = 3tan,. 2sec^2@ - 2 +1 = 3tan@, 2(sec^2@-1)
+ 1 = 3tan@, 2tan^2@ + 1 = 3tan@. now solve this quadratic to get tan@ = 1/2 :)
4-(1+cos2@)=3sin2@
3-cos2@=3sin2@
3=cos2@/1-sin2@
1/3=sec2@-tan2@
1/3=(1+tan^2@-2tan@)/1-tan^2@
=(1+tan@)^2/(1-tan@)(1+tan@)
=> 1+tan @ / 1- tan@ = 1/3
=>3+3tan@=1-tan@
=>tan@=1/2
In such Qs,see options...
Tanx hav been asked..but if options r 1/2,3/4,5/3,3/7 then we cant apply desi in such sums

916. If cosx+cos2x+cos3x=0 find tanx


Ans- x = 45deg.....then tanx = 1......cos45 =1/rt2, cos90=0, cos135 = -sin45 = -1/rt2. so
expression = 0. so tanx = 1....m i right?
only 1 why not -root3

917. A rectangualr sheet of metel, Xcm by Ycm has a square of side Zcm cut from each corner.
The sheet is then bent to form a tray of depth Zcm. The volume of the tray is ?
Ans- (x-2z)*(y-2z)*z

918. Along two perpendicular diameter of a circular disc of radius 5cm , as many circular discs
as possible are cut off with diameter of 2cm. The ration of the area of the original disc to the
perforated disc is ????
Ans- tewari ji aap bhi vahi samjhe jo hum ??? diameter k along ka matlab ??? n perfolated ka
matlab bachi huee ???? 25:9 is ratio in original to remaining....while 25:16 is original to used
one...

919. the radian measure of 63 degree 14 min 51 second


Ans- (63+ 14/60 + 51/3600) *pie/180 .........simplify it this is the ANS

Vous aimerez peut-être aussi